Download as pdf or txt
Download as pdf or txt
You are on page 1of 81

31

10
45

Vol. XXXIII No. 12 December 2015


Corporate Office:
Plot 99, Sector 44 Institutional Area,
Gurgaon -122 003 (HR), Tel : 0124-4951200
e-mail : [email protected] website : www.mtg.in
Regd. Office:
406, Taj Apartment, Near Safdarjung Hospital,
Ring Road, New Delhi - 110029.
Managing Editor : Mahabir Singh
Editor : Anil Ahlawat
26
8
contents 60

74
8 Maths Musing Problem Set - 156
18
10 Jee Work Outs 63
85

18 Ace Your Way CBSe XI


(Series 7) Subscribe online at www.mtg.in

26 Olympiad Corner Individual Subscription Rates

29 Quantitative Aptitude 1 yr. 2 yrs. 3 yrs.


Mathematics Today 330 600 775
31 Concept Boosters (XI) Chemistry Today 330 600 775
Physics For You 330 600 775
45 Concept Boosters (XII) Biology Today 330 600 775

60 Math Archives Combined Subscription Rates

1 yr. 2 yrs. 3 yrs.


63 Ace Your Way CBSe XII PCM 900 1500 1900
(Series 7) PCB 900 1500 1900
PCMB 1000 1800 2300
74 Mock Test Paper Send D.D/M.O in favour of MTG Learning Media (P) Ltd.
Payments should be made directly to : MTG Learning Media (P) Ltd,
WB JEE 2016 Plot 99, Sector 44 Institutional Area, Gurgaon - 122 003, Haryana.
We have not appointed any subscription agent.

84 Math Musing Solutions Owned, Printed and Published by Mahabir Singh from 406, Taj Apartment, New Delhi - 29 and
printed by Personal Graphics and Advertisers (P) Ltd., Okhla Industrial Area, Phase-II, New Delhi.
Readers are advised to make appropriate thorough enquiries before acting upon any advertisements

85 You Ask, We Answer published in this magazine. Focus/Infocus features are marketing incentives MTG does not vouch
or subscribe to the claims and representations made by advertisers. All disputes are subject to
Delhi jurisdiction only.
Editor : Anil Ahlawat
Copyright© MTG Learning Media (P) Ltd.
All rights reserved. Reproduction in any form is prohibited.

mathematics today | december ‘15 7


M aths Musing was started in January 2003 issue of Mathematics Today with the suggestion of Shri Mahabir Singh. The aim of Maths
Musing is to augment the chances of bright students seeking admission into IITs with additional study material.
During the last 10 years there have been several changes in JEE pattern. To suit these changes Maths Musing also adopted the new
pattern by changing the style of problems. Some of the Maths Musing problems have been adapted in JEE benefitting thousand of our
readers. It is heartening that we receive solutions of Maths Musing problems from all over India.
Maths Musing has been receiving tremendous response from candidates preparing for JEE and teachers coaching them. We do hope
that students will continue to use Maths Musing to boost up their ranks in JEE Main and Advanced.
Prof. Dr. Ramanaiah Gundala, Former Dean of Science and Humanities, Anna University, Chennai

Set 156
jee main 7. a2 – b2 is divisible by 5 is
(a) 75 (b) 100 (c) 115 (d) 150
1. Let 0 < a < b be integers such that the H.M. of a and
4 4
b is 2016. The number of pairs (a, b) is 8. a – b is divisible by 5 is
(a) 64 (b) 65 (c) 66 (d) 67 (a) 150 (b) 200 (c) 250 (d) 300
1 1 1 integer match
2. Let 0 < a < b be integers such that + = . 9. Twenty five identical marbles are to be distributed
a b 2016
The number of pairs (a, b) is among 3 children so that each gets atleast 4. The
(a) 80 (b) 81 (c) 82 (d) 85 m
probability that no two get equal number is ,
n
3. The sum of all the roots of the equation reduced fraction, where (n – m) is
2016 matching list
 2 
1 −  = 1 is 10.
5x
Column-I Column-II
(a) 402 (b) 403 (c) 404 (d) 405
P.    1. 173
a = b = c = 1 . The maximum value
4. Let (102016 + 5)2 = 225 N. If S is the sum of the digits
 2  2  2
of N, then the sum of the digits of S is of 4a − 5b + 4b − 5c + 4c − 5a is
(a) 16 (b) 17 (c) 18 (d) 20 Q. 100 3 2. 180
 1 
2016
If x2 + x + 1 = 0, then ∑  x r + 
 xr 
5. ∫ x 5 (2016 − x )6 dx = is r =1

0 R. If y = (x2 – x + 1)5(x – 1)4(x – 2)3 3. 183


2 4 (x – 3)3,
(a) (2016)11 (b) (2016)11 4
11 11 then d y at x = 1 is
2 4 dx 4
(c) (2016)12 (d) (2016)12
11 11 S. If a, b are the roots of x2 – 3x + 5 = 0, 4. 192
jee advanced r, s are the roots of x2 + 5x – 3 = 0 and
6. Let N be the number of right angled triangles with ar + bs, as + br are the roots of
integer sides and inradius r = 2016. Then N is x2 + px + q = 0, then (p + q) is
divisible by 5. 197
(a) 2 (b) 3 (c) 5 (d) 7 P Q R S
comprehension (a) 3 5 4 1
(b) 1 2 3 4
Two numbers a and b are selected from the set of the first (c) 2 3 4 5
25 natural numbers. The number of ways of selecting (d) 1 3 4 5
them such that See Solution set of Maths Musing 155 on page no. 84
Prof. Ramanaiah is the author of MTG JEE(Main & Advanced) Mathematics series

8 mathematics today | December ‘15


PaPer-1

Section-1 7. The number of different hyperbolas represented by


This section contains 8 questions. The answer to each the equation (5x – 5)2 + (5y + 5)2 = mCn(3x + 4y – 2)2
question is a SINGLE DIGIT INTEGER ranging from 0 to where m and n are integers satisfying 1 ≤ n ≤ m ≤ 5 is
9, both inclusive. For each question, darken the bubble
8. Shortest distance between circle (8x – 33)2 + 64y2 = 1
corresponding to the correct integer in the ORS. Marking
scheme : +4 If the bubble corresponding to the answer is 65 − k
darkened, 0 In all other cases.
and parabola 4y2 = x is , where k is
8
1. If the equation of the line(s) through the point of sectioN-2
intersection of the lines x – 3y + 1 = 0, 2x + 5y – 9 = 0 This section contains TEN questions. Each question has FOUR
and whose distance from the origin is 5 is 2x + y = k, options (a), (b), (c) and (d). ONE OR MORE THAN ONE of these
four option (s) is (are) correct. For each question, darken the
then determine k. bubble(s) corresponding to all the correct option(s) in the
2. If two perpendicular tangents PA and PB are drawn ORS. Marking scheme : +4 If only the bubble(s) corresponding
to all the correct option(s) is (are) darkened, 0 If none of
to y2 = 7x then find the minimum length of chord AB.
the bubbles is darkened, – 2 In all other cases.
3. The line x + 2y + a = 0 intersects the circle x2 + 9. Let e(l) be the eccentricity of (x – 3)(y + 2) = l2,
y2 – 4 = 0 at two distinct points A and B. Another line then the value of e(2) – e(3) is
12x – 6y – 41 = 0 intersect the circle x2 + y2 – 4x – 2y + (a) 0 (b) 1 (c) 3 (d) –3
1 = 0 at two distinct points C and D. The value of ‘a’ for 10. P is a point which moves in the xy plane such
which the four points A, B, C and D are concyclic is that the point P is nearer to the centre of a square than
4. The area bounded by the curves x + 2|y| = 1 and any of the sides. The four vertices of the square are
x = 0 is A, then 4A = (± a, ±a). The region in which P will move is bounded
5. If from point P(4, 4) perpendiculars to the straight by parts of parabola of which one has the equation.
(a) y2 = a2 + 2ax (b) x2 = a2 + 2ay
lines 3x + 4y + 5 = 0 and y = mx + 7 meet at Q and R and
(c) y2 + 2ax = a2 (d) x2 = a2 – 2ay
area of triangle PQR is maximum. Then 3m is equal to
11. The straight line segment are P(1, 3) and Q(1, 1).
6. If p is the length of the perpendicular from a
R is a point on the line segment PQ such that
focus upon the tangent at any point P of the ellipse
PR : QR = 1 : l. If R is an interior point of the parabola
x2 y2 y2 = 4x, then l can take value in the interval
+ = 1 and r is the distance of P from the focus,
a2 b2 (a) (0, 1)
 3 
(b)  − , 1
 5 
2a b2 1 3
then − = (c)  ,  (d) none of these
r p2 2 5

By : Vidyalankar Institute, Pearl Centre, Senapati Bapat Marg, Dadar (W), Mumbai - 28. Tel.: (022) 24306367

10 mathematics today | december ‘15


12. If PSQ and PS′R be two focal chords of an ellipse (A) P Q R S T
2 2
x y (B) P Q R S T
+ = 1 and P, Q, R are points (acos q, bsinq),
2
a b2 (C) P Q R S T
(acosq1, bsinq1), (acosq2, bcosq2) respectively, then
q q e −1 q q e2 + 1 (D) P Q R S T
(a) tan tan 1 = (b) tan tan 1 = For each entry in Column I, darken the bubbles of all the
2 2 e +1 2 2 e2 − 1
matching entries. For example, if entry (A) in Column I
q q 1+ e q q 1 + e2 matches with entries (Q) and (R) then darken these two
(c) tan tan 2 = (d) tan tan 2 = bubbles in the ORS. Similarly, for entries (B), (C) and (D).
2 2 1− e 2 2 1 − e2
Marking scheme : For each entry in Column I. +2 If only
13. The locus of middle points of the normal chords of the bubble(s) corresponding to all the correct match(es) is
the rectangular hyperbola x2 – y2 = a2 is (are) darkened, 0 If none of the bubbles is darkened, –1 In
(a) (y2 – x2)3 = 4a2x2y2 (b) (y2 – x2)2 = 4a2x2y2 all other cases.
(c) (y2 + x2)3 = 4a2x2y2 (d) (y2 + x2)2 = 4a2x2y2 19. Match the following:
14. The equation of the tangent to the hyperbola Column I Column II
x2 y2 (A) The circles x2 + y2 + 2x + c = 0 P. c = 1
− = 1 , parallel to the line y = x + 2 is and x2 + y2 + 2y + c = 0 touch
4 3
(a) y = –x + 1 (b) y = x + 1 each other if
(c) y = –x – 1 (d) y = x – 1 (B) The circles x2 + y2 + 2x + 3y + c Q. c < 2
15. Eccentric angle of a point on the ellipse x2 + 3y2 = 6 at = 0 and x2 + y2 – x + 2y + c = 0
a distance of 2 units from the centre of the ellipse is intersect orthogonally if
(a) p/4 (b) p/3 (c) 3p/4 (d) 2p/3 (C) The circle x2 + y2 = 9 contains the R. c = 1/2
16. On the ellipse 4x2 + 9y2 = 1, the points at which the circle x2 + y2 – 2x + 1 – c2 = 0 for
tangents are parallel to the line 8x = 9y are (D) The circle x2 + y2 = 9 is contained S. c>8
2 1  2 1 in the circle x2 + y2 – 6x – 8y +
(a)  ,  (b)  − , 
5 5  5 5
25 – c2 = 0 if
 2 1 2 1
(c)  − , −  (d)  , −  20. Match the following:
 5 5 5 5 Column I Column II
17. Two circles x2 + y2 = 6 and x2 + y2 – 6x + 8 = 0 are (A) The radius of the circle passing P. 2
given. Then the equation of the circle through their through the foci of the ellipse
points of intersection and the point (1, 1) is 3
(a) x2 + y2 – 6x + 4 = 0 (b) x2 + y2 – 3x + 1 = 0 x2 y2
+ = 1 and having centre
(c) x2 + y2 – 4y + 2 = 0 (d) none of these 16 9
18. If p1, p2 denote the lengths of the perpendicular at (0, 3) is
from the point (2, 3) on the lines given by (B) If the length of the major axis Q. 2 2
15x2 + 31xy + 14y2 = 0, then of an ellipse is three times the 3
31 31 12 length of its minor axis, then its
(a) p1 + p2 = (b) | p1 − p2 | = −
eccentricity is
14 74 13
372 15 (C) The eccentricity of hyperbola R. 17
(c) p1 p2 = (d) p1 p2 = whose latus rectum is 8 and
962 14 20
Section-3 conjugate axis is half the distance
This section contains TWO questions. Each question contains between foci, is
two columns, Column I and Column II. Column I has four entries (D) T h e v a l u e o f m f o r w h i c h S. 4
(A), (B), (C) and (D), Column II has four entries (P), (Q), (R) and y = mx + 6 is a tangent to the
(S). Match the entries in Column I with the entries in Column II.
One or more entries in Column I may match with one or more x2 y2
entries in Column II. The ORS contains a 4 × 4 matrix whose hyperbola − = 1 is
100 49
layout will be similar to the one shown below.

12 mathematics today | december ‘15


PaPer-2

Section-1 9. If the tangent at the point (aseca, btana) to the


This section contains EIGHT questions. The answer to each
x2 y2
question is a SINGLE DIGIT INTEGER ranging from 0 to 9, both hyperbola − = 1 meets the transverse axis at T,
inclusive. For each question, darken the bubble corresponding a2 b2
to the correct integer in the ORS. Marking scheme : +4 If the then the distance of T from a focus of the hyperbola is
bubble corresponding to the answer is darkened, 0 In all (a) a(e – cosa) (b) b(e + cosa)
other cases.
(c) a(e + cosa) (d) a2 e 2 + b 2 cot 2 a
1. If the chord y = mx + 1 of the circle x2 + y2 = 1
subtends an angle of measure 45° at the major segment 10. If y = mx be one of the bisectors of the lines
of the circle then value of |m| is 2x2 – 3xy – 4y2 = 0 then
−30 − 41 −3 + 41
2. Find the length of the latus rectum of the conic (a) m = (b) m =
8 8
passing through the origin and having property that
normal at each point (x, y) intersect the x-axis at (c) m = 2 + 5 (d) m = 2 − 5
((x + 1), 0). 11. The coordinates of an end point of the latus rectum
3. If the circles which can be drawn to pass through of the parabola (y – 1)2 = 2(x + 2) are
(1, 0) and (3, 0) and touches the y-axis, intersect at an (a) (–2, 1) (b) (–3/2, 1)
angle q then 4cosq is equal to (c) (–3/2, 2) (d) (–3/2, 0)
4. If m is the slope of a common tangent of the 12. Consider the circle C1 : x2 + y2 = 4 and the circle
parabola y2 = 16x and the circle x2 + y2 = 8, then m2 is C2 : x2 + y2 – 6x – 8y – 24 = 0. Then
equal to (a) C1 and C2 touch each other
(b) C1 completely lies within C2 without a common point
x2 y2 (c) C1 and C2 touch externally
5. If both the foci of the ellipse + = 1 and the
16 b 2 (d) the number of common tangents is one
x2 y2 1 13. The line y = 3x bisects the angle between the lines
hyperbola − = coincide, then find the value
144 81 25 a2x2 + 2axy + y2 = 0 if a is equal to
of b2. (a) –3 (b) 3 (c) –1/3 (d) 1/3
6. Let Pi and Pi′ be the feet of the perpendicular 14. An isosceles triangle ABC is inscribed in a circle
drawn from foci S, S′ on a tangent Ti, for i = 1, 2, ..., 10 x2 + y2 = a2 with the vertex A at (a, 0) and the base angle
to an ellipse whose length of semi major axis is 20, if B and C each equal to 75°, then coordinates of an end
10 point of the base are
∑ (SPi )(S′ Pi ′ ) = 2560 , then find the value of 5e (where
i =1  3a a   3a a 
(a)  ,  (b)  − , 
'e' denotes eccentricity).  2 2 2 2
7. Two rays in the 1st quadrant x + y = |a| and ax – y = 1    3a a
(c)  − 3a , − a  (d)  ,− 
intersect each other in the interval a ∈ (a0, ∞) then the  2  2 2 2
value of a0 is
15. The equation of the pair of straight lines, each of
8. The distance from the centre of the circle which makes an angle a with the line y = x, is
x2 + y2 = 2x to the straight line passing through the (a) x2 + 2xy sec2a + y2 = 0
point of intersection of the circles x2 + y2 + 5x – 8y + 1 = (b) x2 + 2xy cosec2a + y2 = 0
0 and x2 + y2 – 3x + 7y – 25 = 0 is (c) x2 – 2xy cosec2a + y2 = 0
(d) x2 – 2xy sec2a + y2 = 0
sectioN-2
This section contains EIGHT questions. Each question has FOUR 16. Let L1 be a straight line through the origin and L2
options (a), (b), (c) and (d). ONE OR MORE THAN ONE of these be the straight line x + y = 1. If the intercepts made by
four option(s) is(are) correct. For each question, darken the the circle x2 + y2 – x + 3y = 0 and L1 and L2 are equal,
bubble(s) corresponding to all the correct option(s) in the then which of the following equation can represent L1?
ORS. Marking scheme : +4 If only the bubble(s) corresponding (a) x + 7y = 0 (b) x – y = 0
to all the correct option(s) is(are) darkened, 0 If none of the (c) x – 7y = 0 (d) x + 2y = 0
bubbles is darkened, –2 In all other cases.

14 mathematics today | december ‘15


sectioN-3 x y
This section contains TWO paragraphs. Based on each cos q + sin q = 1 and also the equation of tangent to the
a b
paragraph, there will be TWO questions. Each question has
FOUR options (a), (b), (c) and (d). ONE OR MORE THAN ONE of ellipse in terms of slope 'm' is y = mx ± a2m2 + b 2
these four option(s) is(are) correct. For each question, darken
the bubble(s) corresponding to all the correct option(s) in the x2 y2
ORS. Marking scheme : +4 If only the bubble(s) corresponding 19. A tangent to the ellipse + = 1 cuts the axes in
to all the correct option(s) is(are) darkened, 0 If none of the
a2 b2
bubbles is darkened, –2 In all other cases A and B respectively and touches the ellipse at any point
P in the first quadrant. If the point P divides AB in the
Paragraph for Q. No. 17 & 18 ratio 3 : 1, then equation of tangent is
The equation of the normal to the parabola y2 = 4ax
y (a) ax + by = 3ab (b) bx + ay = ab
at the point (x1, y1) is given by y − y1 = − 1 ( x − x1 ). (c) bx + a 3y = 2 ab (d) b 3x + ay = 3ab
2a
Also the equation of normal in parametric form at 20. The equation of the largest circle with centre (1, 0)
(at2, 2at) is y + tx = 2at + at3 and in the slope (m) form that can be inscribed in the ellipse x2 + 4y2 = 16 is
it is y = mx – 2am – am3. The point of intersection (a) x2 + y2 = 22 (b) (x + 1)2 + (y + 1)2 = 3
(T) of normals at any two points on the parabola can 1
be obtained by (c) ( x − 1)2 + ( y − 1)2 =
3
2 2
T ≡ ((2a + a(t1 + t2 + t1t2), –at1t2(t1 + t2)]) 2 2 11
(d) ( x − 1) + y = 3
17. The normal to the parabola y2
= 8x at the point
(2, 4) meets it again at (l, m). The length of the normal anSwer keyS
chord is PAPER-1
(a) 8 (b) 16 2 1. (5) 2. (7) 3. (2) 4. (2) 5. (4)
(c) 8 3 (d) 4 5 6. (1) 7. (6) 8. (8) 9. (a)
10. (a, b, c, d) 11. (a, b, c)
18. If the normal to the parabola y2 = 4ax, makes an 12. (a, c) 13. (a) 14. (b, d) 15. (a, c) 16. (b, d)
angle q with the axis, then it will cut the curve again at 17. (b) 18. (b, c) 19. (A) → (r); (B) → (p); (C) →
an angle (p, q, r); (D) → (s)
1 
(a) sin −1  sin q  (b) tan–1(2q) 20. (A) → (s); (B) → (q); (C) → (p); (D) → (r)
2
PAPER-2
−1  1  1 
(c) tan  tan q  (d) tan −1  sin 2q  1. (1) 2. (2) 3. (2) 4. (1) 5. (7)
2 2
6. (3) 7. (1) 8. (2) 9. (a, c) 10. (c, d)
Paragraph for Q. No. 19 & 20 11. (c, d) 12. (a, d) 13. (a) 14. (b, c) 15. (d)
Suppose the Point (acosq, bsinq) satisfies the equation 16. (a, b) 17. (b) 18. (c) 19. (c) 20. (d)
of the ellipse for any value of 'q'. This 'q' is also an For detailed solution to the Sample Paper,
eccentric angle. Equation of tangent at 'q' is given by visit our website : www. vidyalankar.org.
nn
When Hollywood meets the math genius from India
A movie on a Indian mathematician Ramanujan is going to
hit the big screen. Matthew Brown's biopic-- The Man
Who Knew Infinity - is set for premier on September 17 at the
screenplay was adapted from Robert Kanigel's biography "The
Man Who Knew Infinity: A Life of the Genius Ramanujan". It is
not a typical Hollywood Film and connects deeply with the people.
Toronto Film Festival. The film will help us know more about Ramanujan," film director
A week later, it will be the opening film at the Zurich Film Festival. Matt Brown told HT. "I came across the book 10 years ago and
The film stars Jeremy Irons, Dev Patel, Toby Jones, Stephen Fry, was very inspired by it. I instantly wanted to make a movie. It
Devika Bhise, Jeremy Northam and Kevin McNally. has taken me 10 years to make the film. But I am so happy that
The nearly one hour 40 minutes film tells the true story of I could finally do it," he said. Matt said that he had not brought
Srinivasa Ramanujan (Patel), a self-taught mathematics genius any changes in the film. "I have made sure that the human factor
from India, who leaves behind his young bride, Janaki (Bhise), was at the forefront. Math is difficult to understand and one has
to travel across the world to Cambridge, England. Brown's to connect at a different emotional level."
Courtesy : Hindustan Times

16 mathematics today | december ‘15


Limits and derivatives
limits If ai = 0 ∀ 0 ≤ i ≤ n, then f(x) is called zero function.
Left hand limit Also, lim f (x) = f (a)
x →a
The left hand limit of f at x = a is the expected value
of f at x = a given the values of f near x to the left z A function f(x) is said to be a rational function, if
of a. It is denoted by lim f (x). g (x)
x →a −
f (x) = , where g(x) and h(x) are polynomials
h(x)
Right hand limit
The right hand limit of f at x = a is the expected value in x and h(x) ≠ 0
of f at x = a given the values of f near x to the right lim g (x) g (a)
of a. It is denoted by lim+ f (x). Then lim f (x) = x →a
=
x →a x →a lim h(x) h(a)
x →a
Limit of a function at a Point some important theorems
Limit of a function at x = a is the value of f(x) at
1. If f and g be two real
x = a, i.e., lim f (x).
x →a valued functions such
Note : that f(x) ≤ g(x) ∀ x
lim f (x) exists at x = a iff lim− f (x) = lim+ f (x) lies in the common
x →a x →a x →a
domain of f and g, then
algebra of limits
lim f (x) ≤ lim g (x),
Let f and g be two functions such that both x →a x →a
lim f (x) and lim g (x) exist, then when both lim f (x) and lim g (x) exist at point ‘a’.
x →a x →a x →a x →a
z lim{ f (x) + g (x)} = lim f (x) + lim g (x) 2. Sandwich Theorem : If f(x), g(x) and h(x) are
x →a x →a x →a

lim{ f (x) − g (x)} = lim f (x) − lim g (x) real functions of x such that f(x) ≤ g(x) ≤ h(x) for
z
x →a x →a x →a all x lies in the common domain of f, g and h and
z lim{c ⋅ f (x)} = c ⋅ lim f (x), where c is a constant. lim f (x) = lim h (x) = l (say), for some real number
x →a x →a x →a x →a

{ }{ }
a, then lim g (x) = l.
z lim{ f (x) ⋅ g (x)} = lim f (x) ⋅ lim g (x) x →a
x →a x →a x →a

 f (x)  lim f (x)


z lim   = x →a , provided lim g (x) ≠ 0, g (x) ≠ 0
x →a  g (x)  lim g (x) x →a
x →a
limits of polynomials and rational functions
z A function f(x) is said to be a polynomial
function if f(x) = a0 + a1x + a2x2 + ..... + anxn ,
0 ≤ i ≤ n where ai’s are real numbers.

18 mathematics today | december ‘15


some standard limits algebra of derivative of functions
x n − an Let f and g be two functions such that their derivatives
(i) lim = na n−1, for any positive integer n. are defined in a common domain. Then
x →a x − a
d d d
sin x (i) [ f (x) + g (x)] = f (x) + g (x)
(ii) lim =1 dx dx dx
x →0 x
d d d
1 − cos x (ii) [ f (x) − g (x)] = f (x) − g (x)
(iii) lim =0 dx dx dx
x →0 x
sin(x − a) d d d
=1 (iii) [ f (x) ⋅ g (x)] = g (x) ⋅ f (x) + f (x) ⋅ g (x)
(iv) lim dx dx dx
x →a x −a
(product rule)
tan x d d
(v) lim =1
x →0 x d  f (x)  g (x) dx f (x) − f (x) dx g (x)
(iv) =
tan(x − a) dx  g (x)  ( g (x))2
(vi) lim x − a = 1
x →a (quotient rule)
some additional results based on limits of derivative of a polynomial function
exponential and logarithmic functions Let f(x) = anxn + an–1 xn–1 + an – 2xn–2 + .... +a1x + a0
ex −1 be a polynomial function of degree n in x where ai ∈R,
(i) lim =1
x →0 x i = 0, 1, 2, ...., n and an ≠ 0. Then
log e(1 + x) d
(ii) lim =1 f (x) = nan x n −1 + (n − 1)an −1x n −2 + (n − 2)an −2 x n −3
x →0 x dx
+..........+ 2a2x + a1
ax −1 derivatives of some standard functions
(iii) lim = log e a, a > 1
x →0 x d n
(i) (x ) = nx n−1
(iv) lim(1 + x)1/x = e dx
x →0
d
(ii) dx (sin x) = cos x
derivatives
d
Derivative of a function at a point (iii) dx (cos x) = − sin x
Let f(x) be a real valued function and a be any point in
its domain of definition. The derivative of f(x) at x = a is d  π
(iv) dx (sec x) = sec x tan x,  x ≠ nπ + 2 
f (a + h) − f (a)  
denoted by f ′(a) and defined as f ′(a) = lim , d
h →0 h
(v) (cosec x) = −cosec x cot x, (x ≠ nπ)
provided this limit exists. dx
d
first principle of derivative (vi) (c) = 0, where c is independent of x.
Let f(x) be a function finitely differentiable at every dx
f (x + h) − f (x) d  π
point on its domain. Then, f ′(x) = lim (vii) (tan x) = sec 2 x,  x ≠ nπ + 
h →0 h dx  2
is called the first principle of derivative. This method d 2
(viii) dx (cot x) = − cos ec x, (x ≠ nπ)
is also called ab-initio method or delta method.
mathematicaL reasoning
statement simple and compound statements
A sentence which is either true or false but not both is Simple statement : A statement which has no other
called a statement. statement as its component, is called a simple statement.
negation of a statement Compound statement : It is a combination of two or
Let p be any statement then the statement expressing more simple statements by using the words “and”, “or”
denial of p is called negation of p, which is denoted by etc. In this case such statements are called component
~p and read as ‘not p’. statement.

mathematics today | december ‘15 19


Rule 1 : The compound statement with “AND” is  3 
3. Find the derivative of  9x 2 + + 5 sin x  w.r.t. x.
(a) true, if all its component statements are true.  x 
(b) false, if any or all of its component statements are 4. Evaluate lim θ cosec θ
θ→0
false.
Rule 2 : The compound statement with an “OR” is 5. Which of the following sentences are statements?
(a) true, when one component statement is true or Give reasons for your answer.
both the component statements are true. (i) The square of a number is an even number.
(b) false, when both component statements are false. (ii) All real numbers are complex numbers.
quantifiers 6. Find the component statement of the following
Many mathematical statements contain phrases ‘there compound statements and check whether they are
exists’ and ‘for all’ or ‘for every’. These phrases are true or false.
called quantifiers. (i) The number 3 is prime or it is odd.
implications (ii) All integers are positive or negative.
In Mathematics, we come across many statements of
the form “if p then q”, “p only if q” and “if and only short answer type
if ”, such statements are called implications.
x 5 − a5
z If then implication : A sentence “if p then q” can 7. (i) lim = 405, find all possible values of a.
x →a x − a
be written in the following ways : (ii) Find the derivative of ex using first principle.
(i) p implies q (denoted by p ⇒ q)
(ii) p is sufficient condition for q 8. (i) Given below are two statements:
(iii) q is necessary condition for p p : 25 is a multiple of 5.
(iv) p only if q q : 25 is a multiple of 8.
(v) ~ q implies ~ p Write the compound statement, connecting these
two statements with ‘and’ and ‘or’. In both cases,
• If and only if Implication :
check the validity of the compound statement.
If p and q are two statements, then the compound
statement p ⇒ q and q ⇒ p is called if and only if (ii) If f(x) = mx + c and f(0) = f ′(0) = 1, find the
implication and is denoted by p ⇔ q. value of f(2).
converse statement cos ax − cos bx
9. Evaluate lim
If p and q are two statements, then the converse of the x →0 x2
implication “if p, then q” is “if q, then p”.
sin(x − a)
contrapositive statement 10. (i) Differentiate with respect to x.
cos x
If p and q are two statements, then the contrapositive
of the implication “if p, then q” is “if not q, then not (ii) Differentiate x2 tanx with respect to x.
p” i.e., “if ~q then ~p”. 3 sin x − cos x
11. Evaluate lim
inverse π  π
 x − 6 
x→
6
If p and q are two statements, then the inverse of “if p
then q” is “if ~p then ~q.”
12. Are the following pair of statements negation of
very short answer type each other?
(i) The number x is not a rational number.
4x 2 − 1 The number x is not an irrational number.
1. Evaluate lim
x →1/2 2x − 1 (ii) The number x is not a rational number.
The number x is an irrational number.
2. Evaluate lim 1 − cos 2x
(iii) Two lines are not parallel.
x →0 x2
Two lines does not intersect at a point.

20 mathematics today | december ‘15


Long answer type that all real numbers are complex numbers of the
form a + i 0.
13. (i) Verify by the method of contradiction that 3 is
irrational. 6. (i) p : The number 3 is prime.
q : The number 3 is odd.
 5x − 4, 0 < x ≤ 1
(ii) If f (x) =  3 , show that p and q both are true.
4x − 3x, 1 < x < 2 (ii) p : All integers are positive.
lim f (x) exists. q : All integers are negative.
x→1
tan x + 4 tan 2x − 3 tan 3x p and q both are false.
14. Evaluate lim
x →0 x 2 tan x x 5 − a5
7. (i) Given, lim = 405
x →a x − a
15. Differentiate x2 cos x from first principle.
⇒ 5(a)5–1 = 405 \ 5a4 = 405
dy
16. (i) If y = x tanx, prove that x sin 2 x dx = y 2 + y sin 2 x. ⇒ a4 = 81 ⇒ a = ± 3
(ii) Let f(x) = ex ⇒ f(x + h) = e x + h
x dy f (x + h) − f (x)
(ii) If y = , prove that x = y(1 − y). Then, f ′(x) = lim
x +5 dx h →0 h
17. Using the words “necessary and sufficient” rewrite
e x +h − e x e x ⋅ eh − e x
the statement “The integer m is odd if and only if m3 ⇒ f ′(x) = lim = lim
h →0 h h →0 h
is odd”. Also check whether the statement is true.
e x (e h − 1) x eh − 1 x
 x 8 − 16 x 2 − 9  = lim = e lim = e ⋅1
18. (i) Evaluate lim  4 + h h →0 h
x − 3 
h →0
x →2  x − 4
 eh − 1 
(2x + 4)1/3 − 2  h→0 h = 1
 lim
(ii) Evaluate lim  
x →2 x −2 d x x
Thus, (e ) = e .
dx
soLutions
8. (i) (a) Compound statement with ‘AND’
4x 2 − 1 (2x + 1)(2x − 1) 25 is a multiple of 5 and 8.
1. lim = lim
x →1/2 2 x − 1 x →1/2 2x − 1 This is a false statement since p and q both are
not true.
1
= lim 2x + 1 = 2   + 1 = 2 (b) Compound statement with ‘OR’
x →1/2 2
25 is a multiple of 5 or it is a multiple of 8.
2
1 − cos 2x 2 sin 2 x  sin x  2 This is a true statement.
2. lim = lim = 2 lim   = 2(1) = 2
x →0 x 2 x →0 x2 x →0  x 
(ii) We have,
d 3 f(x) = mx + c ⇒ f ′(x) = m(1) + 0 = m
3. We have (9x 2 + + 5 sin x)
dx x Given f(0) = 1 ⇒ m(0) + c = 1 ⇒ c = 1
d 2 d d Also, given f ′(0) = 1 ⇒ m = 1
= 9⋅ (x ) + 3 ⋅ (x −1) + 5 ⋅ (sin x) Now f(x) = mx + c ⇒ f(x) = x + 1
dx dx dx
⇒ f(2) = 2 + 1 = 3
3
= (9 × 2x) + 3 ⋅ (−1)x −2 + 5 cos x = 18x − + 5 cos x. cos ax − cos bx
x2 9. lim
x →0 x2
θ 1 1
4. lim θ cosec θ = lim = lim= =1 a +b  a −b 
θ→0 sin θ
 sin θ  1 −2 sin  x sin  x
 2   2 
θ→0 θ→0
 θ  = lim
5. (i) The given sentence is not a statement, since the x →0 x2
square of a number may be even or odd, e.g. 22 = a +b  a −b 
4(even), 32 = 9(odd) sin  x sin  x
 2   2 
= −2 lim lim
(ii) The given sentence is a statement, since it is true x →0 x x →0 x

mathematics today | december ‘15 21


a +b  a −b  12. (i) The negation of the statement of “The number
sin  x sin  x
 2  a + b  2  a − b x is not a rational number” is : The number x is a
= −2 lim ⋅ lim ⋅ rational number.
x →0 a +b 2 x →0 a − b 2
x x The second statement is same : “The number x is
2 2
not an irrational number”. Therefore, the given
 a + b   a − b  b2 − a 2
= −2(1)  (1) = statements are negation of each other.
 2   2  2
(ii) The negation of the statement of “The number
10. (i) We have, x is not a rational number” is : The number x
d  sin(x − a)  is a rational number.
dx  cos x 
The second statement is not same : “The number
d  sin x cos a − cos x sin a  x is an irrational number”. Therefore, the given
=
dx  cos x 
 statements are not negation of each other.
(iii) The negation of the statement of “Two lines are
d
=
dx
{tan x cos a − sin a} not parallel” is : The lines are parallel.
The second statement is same : “Two lines does not
d d
= (tan x cos a) − (sin a) intersect at a point”.
dx dx
Therefore, the given statements are negation of each
d d other.
= cos a (tan x) − (sin a)
dx dx
13. (i) Let p be the statement given by
= cosa × sec2 x – 0 = sec2 x cosa
d p : 3 is irrational.
(ii) We have , (x 2 tan x)
dx If possible, let p be not true. Then, p is false.
 d d  ⇒ 3 is rational
=  x 2 ⋅ (tan x) + tan x ⋅ (x 2)
 dx dx  a
⇒ 3 = , where a and b are integers having no
b
= x2sec2x + 2x tanx
common factor.
3 sin x − cos x
11. lim a2
π  π ⇒ 3= ⇒ a2 = 3b2
x→
6  x − 6  b2
⇒ 3 divides a2 ⇒ 3 divides a
 3 1  ⇒ a = 3c for some integer c
2 sin x − cos x 
 2 2 
= lim ⇒ a2 = 9c2 ⇒ 3 b2 = 9c2 ⇒ b2 = 3c2
π  π
x→
6  x − 6  ⇒ 3 divides b2 ⇒ 3 divides b
Hence, our supposition was wrong so, 3 is
 π π irrational.
2  sin x cos − cos x sin 
 6 6 (ii) We have,
= lim
π  π
 x − 6 
x→
6 (L.H.L. of f(x) at x = 1) = lim− f (x)
x →1

 π = lim f (1 − h) = lim 5(1 − h) − 4 = lim(1 − 5h) = 1


sin  x −  h →0 h →0 h →0
 6 sin y
= 2 lim = 2 lim =2
π  π  y → 0 y (R.H.L. of f(x) at x = 1)= lim f (x)
 x − 6 
x→ +
6 x →1

= lim f (1 + h) = lim 4(1 + h)3 − 3(1 + h)


 π h →0 h →0
[putting  x −  = y]
 6 3
= 4(1) – 3(1) = 1

22 mathematics today | december ‘15


mathematics today | december ‘15 23
Clearly, lim f (x) = lim f (x)  h h

x →1 +
x →1 sin  x +  sin
d  2 2
⇒ ( f (x)) = lim − 2x 2
So, lim f (x) exists and is equal to 1. dx h →0 h
x →1
14. We have, + lim 2x cos(x + h) + lim h cos(x + h)
h →0 h →0
tan x + 4 tan 2x − 3 tan 3x
lim h h
x →0 x 2 tan x 
sin  x +  sin
d  2 2
2 tan x  3 tan x − tan 3 x  ⇒ ( f (x)) = −x 2 lim
tan x + 4 − 3 dx h→0 h
1 − tan 2 x  1 − 3 tan 2 x 
2
= lim
x →0 x 2 tan x + lim 2x cos(x + h) + lim h cos(x + h)
h →0 h →0
8  3 − tan x  2
1+ 2 − 3 d
 1 − 3 tan x  ⇒ ( f (x)) = −x 2 sin x + 2x cos x + (0 × cos x)
2
1 − tan x dx
= lim
x →0 x2
d
⇒ ( f (x)) = −x 2 sin x + 2x cos x
(1 − tan 2 x)(1 − 3 tan 2 x) + 8(1 − 3 tan 2 x) dx
−3(3 − tan 2 x)(1 − tan 2 x) 16. (i) We have y = x tanx ...(1)
= lim
x →0 x 2(1 − tan 2 x)(1 − 3 tan 2 x) dy d d
⇒ = x (tan x) + tan x (x)
dx dx dx
1 − 4 tan 2 x + 3 tan 4 x + 8 − 24 tan 2 x − 9
= x sec2 x + tanx·1 = xsec2x + tanx
+12 tan 2 x − 3 tan 4 x
= lim dy
x →0 x 2(1 − tan 2 x)(1 − 3 tan 2 x) \ x sin 2 x = x sin 2 x(x sec 2 x + tan x)
dx
−16 tan 2 x 1
= lim = x 2 sin 2 x ⋅ + x tan x sin 2 x
x →0 x (1 − tan 2 x)(1 − 3 tan 2 x)
2
cos 2 x
 tan x 
2
 1  = x2tan2 x + x tanx sin2x = y2 + y sin2 x
= −16 lim   × lim   [Using (1)]
x →0  x  x →0  1 − tan 2 x 

 1  x
× lim   = −16 (ii) We have, y =
x →0  1 − 3 tan 2 x  x +5
Then
15. Let f(x) = x2 cos x. Then, f(x + h) = (x + h)2cos(x + h) dy (x + 5)1 − x(1) x + 5 − x 5
= = =
dx (x + 5) 2
(x + 5) 2
(x + 5)2
d f (x + h) − f (x)
( f (x)) = lim
dx h→0 h dy 5x ...(1)
Now L.H.S. = x =
2 2 dx (x + 5)2
d (x + h) cos(x + h) − x cos x
⇒ ( f (x)) = lim
dx h →0 h x  x 
R.H.S. = y(1 − y) = 1−
x + 5  x + 5 
d (x 2 + 2hx + h 2)cos(x + h) − x 2 cos x
⇒ ( f (x)) = lim
dx h →0 h x x +5− x 5x ...(2)
= ⋅ =
2 2 x +5 x +5 (x + 5)2
{x cos(x + h) − x cos x}
d +2hx cos(x + h) + h 2 cos(x + h) From (1) and (2),
⇒ ( f (x)) = lim dy
dx h →0 h x = y(1 − y)
dx
d  cos(x + h) − cos x 
⇒ ( f (x)) = lim[x 2   17. The necessary and sufficient condition that the
dx h →0  h  integer m be odd is m3 must be odd. Let p and q
+2x cos(x + h) + h cos(x + h)] denote the statements.

24 mathematics today | december ‘15


p : The integer m is odd. m3 = 8k3. Therefore, m3 is even.
q : m3 is odd. Thus, the given statement is true.
To check the validity of “p if q”, we have to check  x 8 − 16 x 2 − 9  x 8 − 16 x2 − 9
whether “if p then q” and “if q then p” is true. 18. (i) lim  4 + = lim + lim
x →2  x − 4 x − 3  x →2 x 4 − 4 x →2 x − 3
Case 1 : If p, then q
If p, then q is the statement. (x 4 + 4)(x 4 − 4) (x + 3)(x − 3)
= lim + lim
If the integer m is odd, then m3 is odd. We have to x →2 x4 − 4 x →2 x −3
check whether this statement is true. Let us assume
= lim(x 4 + 4) + lim(x + 3)
that m is odd. The m = 2k + 1, where k is an integer. x →2 x →2
Thus 4
= (2 + 4) + (2 + 3) = 20 + 5 = 25
m3 = (2k + 1)3 (ii) Put 2x + 4 = y
= 8k3 + 1 + 6k(2k + 1) = 8k3 + 12k2 + 6k + 1 y−4
⇒ x=
Therefore, m3 is one more than an even number 2
and hence is odd. As x → 2, y → 8
Case 2 : If q, then p
(2x + 4)1/3 − 2 y 1/3 − 2
If q, then p is the statement. \ lim = lim
x →2 x −2 y →8 y − 4
If m is an integer and m3 is odd, then m is odd. −2
2
We have to check whether this statement is true.
We check this by contrapositive method. The y 1/3 − 81/3 y 1/3 − 81/3
= 2 lim = 2 lim
y →8 y − 4 − 4 y →8 y −8
contrapositive of the given statement is :
If m is an even integer, then m3 is an even integer. 1 2 2 1 1
= 2 (8)(1/3)−1 = (23)−2/3 = × =
m is even implies that m = 2k for some k. Then 3 3 3 4 6
nn

CLASSROOM
ATTENTION COACHING INSTITUTES: STUDY MATERIAL

a great offer from MTG

MTG offers “Classroom


Study Material” for
J E E ( M a i n & A d va n c e d ) , A I P M T a n d
FOUNDATION MATERIAL for Class 7, 8, 9,
10, 11 & 12 with YOUR BRAND NAME &
COVER DESIGN.
This study material will save you lots of money
spent on teachers, typing, proof-reading and
printing. Also, you will save enormous time.
Normally, a good study material takes 2 years to
develop. But you can have the material printed
with your logo delivered at your doorstep.
LENT Profit from associating with MTG Brand – the
� EXCEL Y � most popular name in educational publishing
IT
QUAL for JEE (Main & Advanced)/AIPMT/PMT ....

� CON
TENT ER Order sample chapters on Phone/Fax/e-mail.
� PAP Phone : 0124-4951200
� PRIN
TING 09312680856, 09717933372 Your logo
e-mail : [email protected] | www.mtg.in here

mathematics today | december ‘15 25


1. In triangle ABC, the median AD is the geometric Then from (1), it follows that a = 2 (b – c)
mean of AB and AC. from the law of sines
Prove that
sin a = 2(sin b − sin g ) …(2)
1 + cos A = 2 | cos B – cos C|.
We have to show
2. Let r be the inradius of a tetrahedron A1A2A3A4, 1 + cos a = …(3)
2(cos g − cos b).
and let r1,r2,r3,r4 be the inradii of triangles A2A3A4,
A1A3A4, A1A2A4, A1A2A3 respectively. from (2) and (3), it is sufficient to prove that
Prove that sin a sin b − sin g
= ,
1 1 1 1 2 1+ cos a cos g − cos b
+ + + ≤ ,
r12 r22 r32 r42 r 2 a 2 sin[(b − g ) / 2]cos[(b + g ) / 2]
or tan =
with equality if the tetrahedron is regular. 2 2 sin[(b − g ) / 2]sin[(b + g ) / 2]
3. Let a = sin 10°, b = sin 50°, c = sin 70°. Prove that b+g
= cot , and that holds.
(i) a + b = c, (ii) a–1 + b–1 = c–1 + 6. 2
4. find all polynomials f(x) = xn + a1xn–1 + ... + an 2. Let S1, S2,S3, S4 be the areas of the faces A2A3A4,
with the following properties: A1A3A4, A1A2A4, A1A2A3; let a, b, g be the dihedral
(i) all the coefficients a1, a2, .... , an belong to the angles at the edges A2A3, A2A4, A3A4; and let h1 be the
set {–1,1}; altitude from the vertex A1 of the tetrahedron and h1′
(ii) all the roots of the equation f(x) = 0 are real. = A1E be the altitude of the face A1A2A3. Then
5. (a) find all positive integers p ≤ q ≤ r satisfying the 2S4
h1 = h1′ sin a = (1 + cos a)(1 − cos a)
equation A2 A3
p + q + r + pq + qr + rp = pqr + 1.
2
(b) for each such solution (p, q, r), evaluate = (S4 + S4 cos a)(S4 − S4 cos a), …(1)
A2 A3
tan–1(1/p) + tan–1(1/q) + tan–1(1/r).
and analogously
solutions 2
h1 = (S3 + S3 cos b)(S3 − S3 cos b), …(2)
1. Let a, b, c be the sides and a, b, g be the angles of the A2 A4
triangle. We are given that 2
2 2 2 h1 = (S2 + S2 cos g)(S2 − S2 cos g) …(3)
b +c a A3 A4
bc = (AD)2 = − ,
2 4 from (1), (2) and (3), we get
2 2
or a = 2(b – c) …(1) 2
Without loss of generality we may assume b > c, so h1 = .Q …(4)
A2 A3 + A2 A4 + A3 A4
b > g and cos b < cos g.

26 mathematics today | December ‘15


where 3 1
sin350° = sin 50° −
Q = (S4 + S4 cos a)(S4 − S4 cos a) 4 8
+ (S3 + S3 cos b)(S3 − S3 cos b) 3 1
and sin3(–70)° = sin(−70)° − ,
4 8
+ (S2 + S2 cos g )(S2 − S2 cos g ).
i.e. a, b, – c are the three roots of
According to Cauchy’s inequality, we have 3 1
f(t) = t3– t+ .
{ }
1/ 2
Q ≤ (S4 + S4 cos a) + (S3 + S3 cos b) + (S2 + S2 cos g ) 4 8
Therefore a + b + (–c) = 0, …(i)
. {(S4 − S4 cos a) + (S3 − S3 cos b) + (S2 − S2 cos g )}
1/ 2

1
{ } ab(–c) = …(ii)
1/ 2
= S4 + S3 + S2 + (S4 cos a + S3 cos b + S2 cos g )
8
. {S − (S cos a + S cos b + S cos g )}
1/ 2
+ S3 + S2 3
4 4 3 2 and ab + b(–c) + (–c)a = −
= (S4 + S3 + S2 + S1)1/2(S4 + S3 + S2 – S1)1/2 4
Dividing by abc and using (ii) leads to
= S1/2(S – 2S1)1/2 …(5)
 3
where S = S1 + S2 + S3 + S4. from (4) and (5), we get c–1 – a–1 – b–1 =  −  ⋅ 8 , …(iii)
 4
2 S(S − 2S1 ) 4. Let r1, r2, ..., rn represent the roots of f(x). Then
h1 ≤ .
A2 A3 + A2 A4 + A3 A4 2
n
n  …(1)
Therefore [since rS = h1S1 = three times the volume ∑ ri2 =  ∑ ri  − 2 ∑ ri ri = a12 − 2a2 = 1 − 2a2
i =1 i =1 i< j

of the tetrahedron], we obtain

h1S1 2S1 S(S − 2S1 )


r= ≤ .
S A2 A3 + A2 A4 + A3 A4 S
S − 2S1
= r1
S .
In the same manner, we have

S − 2Si i = 1, 2, 3, 4
r ≤ ri ,
S
hence
4
1 1 4 S − 2Si 2
Σ ≤ 2Σ = 2.
i =1 r 2 r i =1 S r
i

1
3. Since, sin 30° = sin 150° = –sin 210° = ,
2
we have
1
sin (3.10)° = sin (3.50)° = sin (3 . –70)° = . …(1)
2
3 1
But sin3x = sin x − sin 3x ,
4 4
so that by (1)
3 1
sin310° = sin 10° − ,
4 8

mathematics today | December ‘15 27


By the Arithmetic Mean-Geometric Mean or 3. When p = 2, (1) becomes (q – 1)(r – 1) =
inequality, we have 2(q + r + 1) or (q – 3)(r – 3) = 10. Thus q – 3 = 1,
r – 3 = 10 or q – 3 = 2, r – 3 = 5, yielding two
∑ ri 2
≥ (∏ ri 2 ) = (a n) = 1
1/n 2 1/n
solutions: (2, 4, 13) and (2, 5, 8). When p = 3,
n
(1) becomes (q – 1) (r – 1) = q + r + 2 or (q – 2)
with equality if and only if ri2 = 1 for all i
(r – 2) = 5 which yields the third solution:
from (1), we obtain 1 – 2a2 ≥ n (3, 3, 7).
This implies
(b) The value is p/4 in all cases. To see this, set
1. n ≤ 3
A = tan–1(1/p), B = tan–1(1/q) and C = tan–1(1/r).
2. If n = 3, ri = ± 1 for all i
Since
3. If n = 2, 3, a2 = –1
1 1 1
This gives the following polynomials: 0 < ≤ ≤ < 1,
r q p
n = 1 f(x) = x + 1 or f(x) = x – 1
n = 2 f(x) = x2 + x – 1 or f(x) = x2 – x – 1 we have 0 < C ≤ B ≤ A < p/4 and thus
n = 3 f(x) = (x – 1)2(x + 1) = x3 + x2 – x – 1 0 < A + B + C < 3p/4.
or f(x) = (x + 1)2(x – 1) = x3 – x2 – x + 1 from the well know formula for tan (x + y) one
The set of polynomials is easily deduces that for all x, y, z with
{x – 1, x + 1, x2 – x – 1, x2 + x – 1, x3 + x2 – x – 1, x + y + z ≠ kp + p/2 (where k denotes an integer).
x3 – x2 – x + 1}. tan (x + y + z)
5. (a) There are exactly 3 solutions, given by tan x + tan y + tan z − tan x tan y tan z
(p, q, r) = (2, 4, 13), (2, 5, 8) and (3, 3, 7). =
1 − (tan x tan y + tan y tan z + tan z tan x )
Note first that
(p – 1) (q – 1) (r – 1) = pqr – (pq + qr + rp) Thus
+ (p + q + r) –1 1 1 1 1
which becomes, using the given equation, + + −
p q r pqr
tan (A + B + C) =
(p – 1) (q – 1) (r – 1) = 2(p + q + r – 1) ...(1)  1 1 1
1−  + +
If p ≥ 4, then 4 ≤ p ≤ q ≤ r implies  pq qr rp 
(p – 1)(q – 1)(r – 1) ≥ 9(r – 1)
pq + qr + rp − 1
and 2(p + q + r – 1) ≤ 2(3r – 1); = =1
pqr − ( p + q + r )
and since
9(r – 1)–2(3r – 1) = 3r – 7 > 0, hence A + B + C = p/4, that is
(1) cannot hold in this case. Thus p < 4. Since p
tan–1(1/p) + tan–1(1/q) + tan–1(1/r) = .
p = 1 clearly does not satisfy (1), we have p = 2 4
nn

Do you want •

NEW arrivals at MTG Book store

yourself to be
Special offers on MTG Books and Magazines
• Exams alerts, instant reply to your queries
updated •

Important questions for forthcoming examinations

about
Study tips, quizzes, flowcharts, learning strategies, news
from science world, and much more…

Like us on Facebook
https://www.facebook.com/pcmbtoday

28 mathematics today | December ‘15


1. A sum of money is to be distributed among A, B, respectively and the train passes them completely in
C and D in the proportion of 5 : 2 : 4 : 3. If C gets 10 seconds and 11 seconds respectively. The speed
` 1000 more than D, what is B’s share? of the train is
(a) ` 500 (b) ` 1500 (a) 24 km/hr (b) 25 km/hr
(c) ` 2000 (d) None of these (c) 27 km/hr (d) 28 km/hr
2. A shopkeeper sold an article offering a discount of 7. A man arranges to pay off a debt of ` 3600 by
5% and earned a profit of 23.5%. What would have 40 annual installments which are in A.P. When
been the percentage of profit earned if no discount
30 of the installments are paid, he dies leaving
was offered?
one-third of the debt unpaid. The value of the 8th
(a) 24.5 (b) 28.5
installment is
(c) 30 (d) None of these
(a) ` 35 (b) ` 50
3. A and B can do a piece of work in 45 days and (c) ` 65 (d) None of these
40 days respectively. They began to do the work
8. The difference between the radii of the smaller
together but A leaves after some days and then
B completed the remaining work in 23 days. The circle and the bigger circle is 7 cm and the difference
number of days after which A left the work was between the areas of the two circles is 1078 cm2.
(a) 6 (b) 8 (c) 9 (d) 12 Radius of the smaller circle is
(a) 17.5 cm (b) 21 cm
4. A, B and C jointly thought of engaging themselves
(c) 28 cm (d) None of these
in a business venture. It was agreed that A would
invest ` 6500 for 6 months, B, ` 8400 for 5 months 9. In the given figure, AOB is a diameter of the circle
and C, ` 10000 for 3 months. A wants to be the and CD || AB. If ∠CAB = 25°, then ∠CAD =
working member for which he was to receive 5% of
the profits. The profit earned was ` 7400. Calculate D
C
the share of B in the profit.
A 25°
(a) ` 1900 (b) ` 2660 B
O
(c) ` 2800 (d) ` 2840
5. If one of the interior angles of a regular polygon
9 (a) 45° (b) 40° (c) 65° (d) 115°
is equal to of each interior angle of a regular
8
hexagon, then the number of sides of the polygon 10. (2 sin2 30° – 3 cos2 45° + tan2 60°) =
is (a) 1 (b) 2 (c) 4 (d) 5
(a) 4 (b) 5 (c) 7 (d) 8 1 2 3 4 5 6
11. 999 + 999 + 999 + 999 + 999 + 999 =
6. A train passes two persons walking in the same 7 7 7 7 7 7
direction in which the train is going. These persons (a) 2997 (b) 5979
are walking at the rate of 3 km/hr and 5 km/hr (c) 5997 (d) 5994

mathematics today | December ‘15 29


12. Simple interest on a certain sum at a certain annual 21. Find the ratio in which rice at ` 7.20/kg be mixed
25 with rice at ` 5.7/kg to produce a mixture worth
rate of interest is of the sum. If the numbers
16 ` 6.3/kg.
representing rate per cent and time in years be (a) 1 : 3 (b) 2 : 3
equal, then rate per cent per annum is (c) 3 : 4 (d) 4 : 5
1 22. A box contains 2 white balls, 3 black balls and 4 red
(a) 8% (b) 11 %
2 balls. In how many ways can 3 balls be drawn from
1 1 the box, if at least one black ball is to be included in
(c) 12 % (d) 12 %
4 2 the draw?
(a) 32 (b) 48 (c) 64 (d) 96
(0.27)2 + (0.21)2 + (0.29)2
13. = 23. If a + b + c = 0, then find the value of
(0.027)2 + (0.021)2 + (0.029)2
(a) 0.1 (b) 0.01 (c) 10 (d) 100 a4 + b4 + c 4
14. A covered wooden box has the inner measures as b2c 2 + c 2a2 + a2b2
115 cm, 75 cm and 35 cm and the thickness of wood (a) 0 (b) 1 (c) 2 (d) 3
is 2.5 cm. The volume of the wood is
(a) 81000 cu. cm (b) 81775 cu. cm 24. A bag contains 4 white, 5 red and 6 blue balls.
(c) 82125 cu. cm (d) None of these Three balls are drawn at random from the bag. The
probability that all of them are red, is
15. The sum of the roots of the equation 3
1 2 2
5x2 + (p + q + r) x + pqr = 0, is equal to zero. What is (a) (b) 22 (c) (d)
22 91 77
the value of (p3 + q3 + r3) ? 25. Three years ago, the age of a father was 7 times that
(a) 2pqr (b) 3pqr (c) 4pqr (d) 5pqr
of his son. Three years hence the father’s age would
16. The points A(1, 2), B(5, 4), C(3, 8) and D(–1, 6) be four times that of his son. The present ages of the
taken in order are the vertices of a father and his son respectively are
(a) rectangle (b) rhombus (a) 45, 9 (b) 40, 5
(c) square (d) None of these (c) 50, 10 (d) 45, 8
x( x + 1)(2 x + 1) answer keys
17. If (12 + 22 + 32 + .... + x2) = , then
(12 + 32 + 52 + .... + 192) = 6 1. (c) 2. (c) 3. (c) 4. (b) 5. (d)
(a) 1330 (b) 2100 (c) 2485 (d) 2500 6. (b) 7. (c) 8. (b) 9. (b) 10. (b)
11. (c) 12. (d) 13. (c) 14. (c) 15. (b)
18. If log10 2 = 0.3010 and log10 7 = 0.8451, then the
value of log10 2.8 is 16. (c) 17. (a) 18. (a) 19. (a) 20. (d)
(a) 0.4471 (b) 1.4471 21. (b) 22. (c) 23. (c) 24. (c) 25. (a)
(c) 2.4471 (d) None of these nn
19. A man is watching from the top of a tower a boat
speeding away from the tower. The boat makes an
angle of depression of 45° with the man’s eye when
at a distance of 60 metres from the tower. After
5 seconds, the angle of depression becomes 30°. Online Test Series
What is the approximate speed of the boat, assuming
Practice Part Syllabus/ Full Syllabus
that it is running in still water?
Mock Test Papers for
(a) 32 km/hr (b) 36 km/hr

JEE Main
(c) 38 km/hr (d) 40 km/hr
20. Three numbers are in the ratio 1 : 2 : 3 and their
HCF is 12. The numbers are
(a) 4, 8, 12 (b) 5, 10, 15 Log on to http://test.pcmbtoday.com
(c) 10, 20, 30 (d) 12, 24, 36

30 mathematics today | December ‘15

Attempt free online test


Log on to http://test.pcmbtoday.com
PERMUTATION AND COMBINATION
*ALOK KUMAR, B.Tech, IIT Kanpur
This column is aimed at Class XI students so that they can prepare for competitive exams such as JEE Main/Advanced, etc. and
be also in command of what is being covered in their school as part of NCERT syllabus. The problems here are a happy blend
of the straight and the twisted, the simple and the difficult and the easy and the challenging.

IntroductIon z Some Important properties


The present chapter can be given the name as (i) n! = n (n – 1) !
counting techniques or counting without actual (ii) (2n)! = 2n ⋅ n! [1 ⋅ 3 ⋅ 5 ⋅ 7... ⋅ (2n – 1)]
counting. For example if we are interested in (iii) 0! = 1! = 1
counting the number of 9 digit numbers which (iv) factorial of negative integers are not defined
are divisible by 9, then it will take a lot of time to z arrangement of n different things
count the number of such numbers, but by using The number of permutations of n different things
the techniques given in this chapter, this will be taken all at a time = nPn = n!
easy enough to find the number.
The basis of all the techniques given in this z meaning and value of nPr
chapter are based on two fundamental principles The number of permutations of n different things,
of counting namely addition and multiplication taken r at a time is denoted by nPr or P (n, r).
principles. Using these principles, formulae for n!
n
Pr = (0 ≤ r ≤ n )
number of ways of arrangements and selections (n − r )!
of different objects are derived. = n (n – 1) (n – 2) ... (n – r + 1).
fundamental prIncIple of countIng note :
multiplication principle: If an operation can be (i) nP0 = 1, nP1 = n and nPn – 1 = nPn = n!.
performed in ‘m’ different ways following which a (ii) nPr = n (n – 1Pr – 1) = n (n – 1) (n – 2Pr – 2)
second operation can be performed in ‘n’ different = n (n – 1) (n – 2) (n – 3 Pr – 3) = ...
ways, then the two operations in succession can be alternative :
performed in m × n ways. This can be extended (Combinational proof) nPr denotes the number
to any finite number of operations. of ways of arranging r-objects out of n-objects
permutatIon in a line. This work can be done in the following
definition : Each of the different arrangements way also.
which can be made by taking some (or all) of a Suppose the objects are a1, a2, ..., an. First we
number of given things is called permutation. find the number of permutations, in which
note : a1 does not appear. Number of such permutations
Factorial Notation : The continued product of first is n – 1Pr. Further we consider those arrangements,
n natural numbers is generally written as n! and in which a1 necessarily appears. Number of such
is read as factorial n i.e., permutation is r ⋅ n – 1Pr – 1, as we can arrange
n! = n (n – 1)(n – 2) ... 3. 2. 1 (r – 1) objects out of (n – 1) objects in n – 1Pr– 1
* Alok Kumar is a winner of INDIAN NAtIoNAl MAtheMAtIcs olyMpIAD (INMo-91).
he trains IIt and olympiad aspirants.
mathematics today | december ‘15 31
ways and then in any such permutation we can meaning and value of nCr
fix the position of a1 in r-ways. The number of combinations of n different things
Now using the principle of addition, the required taken r at a time is denoted by
number is n – 1Pr + r ⋅ n – 1Pr – 1. n 
n
z arrangement when repetition is allowed Cr or C (n, r) or  
The number of permutations of n different things r 
taken r at a time, when each thing may be repeated n n !
Cr = (0 ≤ r ≤ n)
any number of times, is nr. r !(n − r )!
arrangement of thIngS when all are n
Pr n(n − 1)(n − 2). . .(n − r + 1)
not dIStInct = =
r! r (r − 1)(r − 2) . . . 2 ⋅1
The number of permutations of n things taken all
at a time, p are alike of one kind, q are alike of If r > n, then nCr = 0.
second kind and r are alike of a third kind and the note :
n! (i) nCr is a natural number.
rest n – (p + q + r) are all different is
p! q! r ! (ii) nC0 = nCn = 1, nC1 = n
z Some cases (iii) nCr = nCn – r
(a) Arrangement when certain things occur (iv) nCr + nCr – 1 = n + 1Cr
together. (v) nCx = nCy ⇒ x = y or x + y = n
(b) Arrangement when certain things do not occur (vi) n ⋅ n – 1Cr – 1 = (n – r + 1) ⋅ nCr –1
together. (vii) If n is even then the greatest value of nCr is
method : n
Cn/2 .
Required number of ways = Total ways
(without restriction) - Number of ways in (viii) If n is odd then the greatest value of nCr is
n
which they sit together. C n+1 or nC n−1 .
(c) Arrangement when no two of a certain things 2 2
occur together. n
(d) Arrangement when order should not change. (ix) nCr = n –1
Cr – 1
r
If we want to arrange ABC in such a way so
that order of A and C should not change, it z Selection from distinct objects
means we want those arrangement in which The number of ways (or combinations) of n
A comes before C i.e., different things selecting at least one of them is
n
ABC, BAC, ACB C1 + nC2 + nC3 + ... + nCn = 2n – 1.
For this we take A & C as identical objects and z Selection from Identical objects
then ABC can be arranged in 3!/2! = 3 ways. 1. The number of ways of selecting r (r ≤ n)
(e) Rank of the word in the dictionary. objects out of n identical objects is 1.
(f) Sum of numbers formed with the given digits. 2. The number of ways of selecting at least one
method: Suppose we have to find sum of numbers object out of n identical objects is n.
of four digits that can be formed with the given z Selection when both identical & distinct objects
digit, first find the sum at unit place, ten’s place, are present
hundred’s place & thousand’s place. Sum of If out of (p + q + r + t) things, p are alike of one
numbers = (sum at unit place) × 1 + (sum at ten’s kind, q are alike of second kind, r are alike of third
place) × 10 + (sum at hundred’s place) × 100 + kind and t are different, then the total number of
(sum at thousand’s place) × 1000. combinations is (p + 1) (q + 1) (r + 1) 2t – 1.
combInatIon (SelectIon) (a) total number of divisors of a given natural
definition : Each of the different group or number
selection which can be made by taking some (or Every natural number N can always be put
all) number of given things, without reference to in the form N = p1a1 p2a2 ... pkak where p1, p2,
the order of the things in each group, is called a ..., pk are distinct primes and a1, a2, ... ak are
combination. non negative integers.

32 mathematics today | december ‘15


If N = p1a1 p2a2 ... pkak, then number of divisors n
Pn
of N are (a1 + 1) (a2 + 1) ... (ak + 1) which = (n – 1)!, if clockwise and anticlockwise
includes 1 and N also. n
note : All the divisors excluding 1 and N are orders are taken as different.
called proper divisors. z formation of necklace/garland using beads/
(b) Sum of divisors flowers (ring permutation)
The sum of the divisors of N Consider five beads A, B, C, D and E in a necklace
= (1 + p1 + p12 + … + p1a1) (1 + p2 + p22 + … or five flowers A, B, C, D and E in a garland etc.
If the necklace or garland on the left is turned
… + p2a2) … (1 + pk + pk2 + ... + pkak)
over we obtain the arrangement on the right, i.e.,
p a1 +1 − 1 p2a2 +1 − 1 p ak +1 − 1 anticlockwise and clockwise order of arrangements
= 1 . ... k are not different.
p1 − 1 p2 − 1 pk − 1
(c) number of ways in which N can be resolved D C C D
as a product of two factors
The number of ways of putting N as a product
of two natural numbers is E B E
B
1 A A
(a1 + 1) (a2 + 1) ... (ak + 1) if N is not a
2 Thus, the number of circular permutations
perfect square. If N is a perfect square then of ‘n’ different things taken all at a time is
1 1
this is [(a1 + 1) (a2 + 1) ... (ak + 1) + 1]. (n – 1)!, if clockwise and anticlockwise orders
2 2
are taken as not different.
(d) number of ways in which a composite
number N can be resolved into two factors z number of circular permutations of ‘n’ different
which are relatively prime or co-prime. things taken ‘r’ at a time.
Number of ways = 2n–1 where n is the number case I: If clockwise and anticlockwise orders are
of different prime factors in N. taken as different, then the required number of
circular permutation = (nPr) / r.
cIrcular permutatIonS case II: If clockwise and anticlockwise orders
z arrangements round a circular table : are taken as same, then the required number of
Consider five persons A, B, C, D and E to be seated circular permutations = (nPr) / (2r)
on the circumference of a circular table in order
dIvISIon Into groupS
(which has no head). Now, shifting A, B, C, D and
E one position in anticlockwise direction we will z different number of objects in each group
get arrangements as follows: The number of ways in which n distinct objects
E D C can be split into r groups containing respectively
s1, s2, ... , sr objects, where si ≠ sj , for i ≠ j, is given
A D E C D B
n!
by .
s1 ! s2 !. . . sr !
B C A B E A z Same number of objects in some group
If k of the numbers among s 1 , s 2 , ... , s r are
(i) (ii) (iii) equal, then the required number of ways is
B A n!
C A B E s1 ! s2 !. . . sr ! k !
For example if 36 distinct objects are to be divided
D
among 9 groups such that four groups have 2
D E C
objects each, three groups have 5 objects each and
(iv) remaining two groups having 6 and 7 objects, then
(v)
the required number of ways
We see that arrangements in all figures are same.
36 ! .
\ The numb er of circ u l ar p er mut at ions =
of n different things taken all at a time is (2 !) ⋅ 4 !(5 !)3 ⋅ 3 ! ⋅ 6 ! ⋅ 7 !
4

mathematics today | december ‘15 33


dIStrIbutIon among perSonS m
The number of ways in which n distinct objects can Note that if x ∈  Ak , then x belongs to atleast
be distributed among r persons in a required way k =1
= number of ways of dividing n distinct objects one of Ak, 1 ≤ k ≤ m.
in r-groups in the required way × r!.
de-arrangement
For example if 36 distinct objects are to be
As another application of the principle of inclusion
divided among 9 persons such that four of the
and exclusion, number of dearrangement of n
persons are getting 2 objects each, three of
objects (number of ways in which n numbered
the persons are getting 5 objects each and the
balls (from 1 to n) can be placed in n numbered
remaining two persons are getting 6 and 7
boxes (from 1 to n), one in each box, so that no
objects, then the number of ways of doing so is
ball goes to its corresponding numbered box) is
36 !
× 9! .  1 1 1 (−1)n 
(2 !) ⋅ 4 ! (5 !)3 ⋅ 3 ! ⋅ 6 ! ⋅ 7 !
4
given by n! 1 − + − + . . . +
n! 
.
 1! 2 ! 3 !
exponent of prIme number (p) In n!
Let p be a given prime and n is any positive integer, Also Dn = (n – 1) [Dn–1 + Dn–2]
then maximum power of p present in n! is m u lt I n o m Ia l t h e o r e m a n d I t S
n  n   n  applIcatIon
  +  2  +  3  + . .., where [·] denotes the Consider the equation x1 + x2 + ... +xr = n, where
 p   p   p  ai ≤ xi ≤ bi ; ai, bi, xi ∈ I ; i = 1, 2, ..... r. In order to
greatest integer function. The proof of the above find the number of solutions of the given equation
n satisfying the given conditions we observe that the
formula can be obtained using the fact that   number of solutions is the same as the coefficient
m
gives the number of integral multiples of m in 1, 2, of xn in the product
... , n; for any positive integers n and m. The above (x a1 + x a1 +1 + x a1 +2 + ... + x b1 ) ×
formula does not work for composite numbers. (x a2 + x a2 +1 + x a2 +2 + ... + x b2 ) ×
For example, if we have to find the maximum
power of 6, present in 32!, then the answer is not (x a3 + x a3 +1 + x a3 +2 + ... + x b3 ) × …
 32   32  … × (x ar + x ar +1 + x ar +2 + ... + x br )
 6  +  2  + . . . = 5, as 5 is the number of integral
  6  For example, if we have to find the number
multiples of 6 in 1, 2, ... , 32; and 6 can be obtained of non negative integral solutions of x1 + x2 +
on multiplying 2 by 3 also. Hence for the required ... + xr = n, then as above the required number is
number, we find the maximum powers of 2 and the coefficient of xn in
3 (say r and s) present in 32!. Using the above (x 0 + x1 + . . . + x n )(x 0 + x1 + . . . + x n )
formula r = 31 and s = 14. Hence 2 and 3 will be . . .(x 0 + x1 + . . . + x n )(r -brackets)
combined (to form 6) 14 times. Thus maximum
= Coefficient of xn in (1 + x + x2 + ... + xn)r
power of 6 present in 32! is 14.
= Coefficient of xn in (1 + x + x2 + ... )r
prIncIple of IncluSIon and excluSIon = Coefficient of xn in (1 – x)– r
If A1, A2, ... , Am are m sets and n(S) denotes the = Coefficient of xn in
number of elements in the set S, then  (−r ) (−r − 1) 2 (−r ) (−r − 1) (−r − 2) 3
m  m 1 + ( −r ) ( − x ) + (− x ) + (− x ) . . .
n   Ak  = ∑ n( Ak ) − ∑ n( Ai ∩ A j ) + . . .  2! 3!
 k =1  k =1 1≤ i < j ≤ m  (−r ) (−r − 1) 2 (−r ) (−r − 1) (−r − 2) 3 
 s 1+ ( −r ) ( − x ) + (− x ) + (− x ) + ...
s −1  2! 3! 
+ ( −1) ∑ n   Aik  + . . . n
  = Coefficient of x in
i <i1 < i2 ... < is ≤m k =1
[1 + rC1x +r +1 C2 x 2 +r +2 C3 x 3 + . . .]
m−1 m 
+ ( −1) n   Ak 
 k =1  = n+r −1Cn =n+r −1 Cr −1

34 mathematics today | december ‘15


note : If there are l objects of one kind, m objects (a) 14 · 7C2 (b) 14 · 7C3
of second kind, n objects of third kind and so on; (c) 7 · 7C2 (d) 7 ·7 C3
then the number of ways of choosing r objects 5. The number of times the digit 3 will be written
out of these objects is the coefficient of xr in the when listing the integers from 1 to 1000, is
expansion of (a) 269 (b) 300 (c) 271 (d) 302
(1 + x + x2 + x3 + ... + xl) (1 + x + x2 + ... + xm)
(1 + x + x2 + ... + xn) ... . 6. The number of different ordered triplets (a, b, c), a,
Further if one object of each kind is to be included, b, c ∈I such that these can represent sides of a triangle
then the number of ways of choosing r objects whose perimeter is 21, is
out of these objects is the coefficient of xr in the (a) 12 (b) 31 (c) 55 (d) 91
expansion of 7. The number of different permutations of all the
(x + x2 + x3 + ... + xl) (x + x2 + x3 + ... + xm) letters of the word 'PERMUTATION' such that any two
(x + x2 + x3 + ... + xn) ...
consecutive letters in the arrangement are neither both
and the number of possible permutations of r vowels nor both identical is
objects out of these objects is the coefficient of (a) 63 × 6! × 5! (b) 57 × 5! × 5!
xr in (c) 33 × 6! × 5! (d) 7 × 7! × 5!
 x x 2
x  x x
l 2 m
x   x x8. The
2 n  positive integral value of x which satisfies
x least
r! 1 + + + ... +  1 + + + ... +  1 + + the+inequality
...  ... 10
 1! 2 ! l !   1! 2 ! m !   1! 2 ! n!  Cx – 1 > 2 ⋅ 10Cx is
(a) 7 (b) 8 (c) 9 (d) 10
 x x2 xl   x x2 xm   x x2 xn 
1 + + + ...  1 + + + ... +  1 + + + ... +  ... 9. A man moves one unit distance for each step he
 1! 2 ! l !   1! 2 ! m !   1! 2 ! n! 
takes. He always moves forward, backward, up or down
While applying it, we use the coefficient of xr in either parallel to the x-axis or y-axis. He starts from the
(1 – x)–n is n+r–1Cr or n+r–1Cn–1. point (0, 0) and reaches the point (1, 1) at the end of six
(i) Number of non-negative integral solution steps. The number of ways he can do it is
of equation x 1 +x 2 +...+x r =n is n+r–1C n or (a) 280 (b) 300 (c) 360 (d) 420
n+r–1
Cr–1.
(ii) Number of positive integral solution of 10. Number of pairs of positive integers (p, q) whose
equation x1 + x2 + ... + xr = n is n–1Cn–r or L.C.M. (least common multiple) is 8100, is “K”. Then
n–1
Cr–1. number of ways of expressing K as a product of two
coprime numbers is
Problems (a) 2 (b) 6 (c) 4 (d) 8
section-i 11. Five balls of different colours are to be placed in
single correct Answer type three boxes of different sizes. Each box can hold all five
1. Total number of integers ‘n’ such that 2 ≤ n ≤ 2000 balls. In how many ways can we place the balls so that
and H.C.F of ‘n’ and 36 is one, is equal to no box remains empty ?
(a) 666 (b) 667 (c) 665 (d) 668 (a) 20 (b) 100 (c) 150 (d) 200
2. A flight of stairs has 10 steps. A person can go up 12. The number of points (x, y, z) in space, whose
the steps one at a time, two at a time or any combination each coordinate is a negative integer such that
of 1s and 2s. The total number of ways in which the x + y + z + 12 = 0 , is
person can go up the stairs is (a) 55 (b) 110 (c) 75 (d) 100
(a) 75 (b) 79 (c) 85 (d) 89 13. The number of positive terms in the sequence
3. The number of ways of forming an arrangement of 195
n+3
P3
5 letters from the letters of the word “IITJEE” is xn = − , n ∈ N is
n n+1
(a) 60 (b) 96 (c) 120 (d) 180 4 Pn Pn+1
(a) 2 (b) 3 (c) 4 (d) 5
4. Let A = {x1, x2, x3, … , x7}, B = {y1, y2, y3}. The total
number of functions f : A → B that are onto and there 14. 12 small sticks of length 1 cm each are distributed
are exactly three elements x in A such that f (x) = y2, is into three children A, B and C. These children join
equal to the sticks in the form of line segments individually. If

mathematics today | december ‘15 35


‘n’ is the number of ways in which the sticks can be 24. The number of 4-digit numbers that can be formed
distributed to the children so that the line segments by using the digits 1,2,3,4,5,6,7,8 and 9 such that the
joined by them form a triangle then the value of ‘n’ is least digit used is 4, when repetition of digits is allowed,
(a) 10 (b) 9 (c) 8 (d) 12 is
15. A guard of 12 men is formed from a group of ‘n’ (a) 617 (b) 671 (c) 716 (d) 761
soldiers. It is found that 2 particular soldiers A and B section-ii
are 3 times as often together on guard as 3 particulars multiple correct Answer type
soldiers C, D and E. Then n – 24 is equal to 11
(a) 8 (b) 7 (c) 2 (d) 6 25. C10 · 9C1 + 11C9 · 9C2 + ......... + 11C2 · 9C9 =
16. Number of 4 digit numbers having the sum of the (a) 20C11 (b) 20C8 (c) 20C9 – 1
digits equal to 9 is 11Cr then least value of r = (d) Number of different ways of exchanging 11 books
(a) 1 (b) 2 (c) 3 (d) 4 of A with the 9 books of B
17. Let x1x2x3x4x5x4x3x2x1 be a nine digit palindrome 26. Using the elements –3, –2, –1, 0, 1, 2, 3
such that either the sequence (x1, x2, x3, x4, x5) is a (a) The number of 3 × 3 matrices having trace 0 is
strictly ascending or strictly descending. Then the 37(76)
number of such palindromes is (b) The number of 3 × 3 matrices is 79
(a) 9 × 9P4 (b) 3 × 9P5 (c) The number of 3 × 3 skew symmetric matrices is 73
(c) 9 × 9C5 (d) 3 × 9C5 (d) The number of 3× 3 symmetric matrices is 76
18. The number of positive integers ≤ 100000 which 27. Consider the following statements.
contain exactly one 2, one 5 and one 7 in its decimal (i) In a 12 storeyed house, 10 people enter the lift
representation is cabin at ground floor. It is known that they will
(a) 2940 (b) 7350 (c) 2157 (d) 1582 leave lift in groups of particular 2, 3 and 5 people
at different storey. The number of ways this can be
19. If x, y ∈ (0, 30) such that
done if the lift does not stop at first and second
 x   3x   y   3 y  11 5 floors is 720.
3 +  2  + 2 +  4  = 6 x + 4 y (ii) Each of three ladies have brought their one child
       
for admission to a school. The principal wants to
(where [x] denotes greatest integer ≤ x). Then the
interview the six persons one by one, subject to
number of ordered pairs (x,y) is
the condition that no mother is interviewed before
(a) 0 (b) 2 (c) 4 (d) 28
her child. The number of ways in which interviews
3 can be arranged is 90.
n−1  nC  4
20. If ∑  r
 = , then n = (iii) The number of ways in which one can put three
n + nC 
r =0  Cr 5 balls numbered 1, 2, 3 in three boxes labelled a,
r +1
b, c such that atmost one box is empty is equal
(a) 5 (b) 4 (c) 3 (d) 6
to 18.
21. A (1, 2) and B(5, 5) are two points. Starting from A, (iv) A box contains 5 different red balls and 6 different
line segments of unit length are drawn either rightwards white balls. The total number of ways in which 4
or upwards only, in each step, until B is reached. Then, balls can be selected, taking atleast 1 ball of each
the number of ways of connecting A and B in this colour is 310.
manner is (a) statements (i) and (ii) are correct.
(a) 35 (b) 40 (c) 45 (d) 50 (b) statements (ii) and (iv) are correct.
22. Let the product of all the divisors of 1440 be P. If P (c) statements (i) and (iii) are correct.
is divisible by 24x, then the maximum value of x is (d) all statements are correct.
(a) 28 (b) 30 (c) 32 (d) 36 28. A fair coin is tossed n times. Let an denotes the
23. The number of 5-digit numbers which are divisible no. of cases in which no two heads occur consecutively,
by 3 that can be formed by using the digits 1,2,3,4,5,6,7,8 then
and 9, when repetition of digits is allowed, is (a) a1 = 2 (b) a2 = 3
(a) 39 (b) 4⋅38 (c) 5⋅38 (d) 7⋅38 (c) a5 = 14 (d) a8 = 55

36 mathematics today | december ‘15


29. If X = 144, then section-iii
(a) no. of divisors (including 1 and X) of X = 15 comprehension type
(b) sum of divisors (including 1 and X) of X = 403
(c) product of divisors (including 1 and X) of paragraph for Question no. 35 to 37
X = 1215 10-digit numbers are formed by using all the digits
(d) sum of reciprocals of divisors (including 1 and X) 0,1,2,3,4,5,6,7,8 and 9 such that they are divisible by
403 11111.
of X =
144 35. The digit in the ten’s place, in the smallest of such
30. Letters of the word SUDESH can be arranged in numbers, is
(a) 120 ways when two vowels are together (a) 9 (b) 8 (c) 6 (d) 7
(b) 180 ways when two vowels occupy in alphabetical 36. The digit in the unit’s place, in the greatest of such
order numbers, is
(c) 24 ways when vowels and consonants occupy their (a) 2 (b) 3 (c) 4 (d) 1
respective places
37. The total number of such numbers is
(d) 240 ways when vowels do not occur together (a) 3456 (b) 5634 (c) 6543 (d) 4365
31. The number of isosceles triangles with integer sides
paragraph for Question no. 38 to 40
if no side exceeds 2008 is
(a) (1004)2 if equal sides do not exceed 1004. Define a function f : N → N as follows :
(b) 2(1004)2 if equal sides exceed 1004. f(1) = 1, f(P n ) = P n−1 (P − 1) if P is prime and n ∈ N
(c) 3(1004)2 if equal sides have any length ≤ 2008. and f (mn) = f(m)f(n) if m & n are relatively prime
(d) (2008)2 if equal sides have any length ≤ 2008. natural numbers.
32. A contest consists of ranking 10 songs of which 6 38. f(8n + 4) where n ∈ N is equal to
are Indian classic and 4 are western songs. Number of (a) f(4n + 2) (b) f(2n + 1)
ways of ranking so that (c) 2f(2n + 1) (d) 4f(2n + 1)
(a) There are exactly 3 Indian classic songs in top 5 39. The number of natural numbers ‘n’ such that f(n)
is (5!)3. is odd is
(b) Top rank goes to Indian classic song is 6(9!). (a) 1 (b) 2 (c) 3 (d) 4
(c) The ranks of all western songs are consecutive is n
40. If f(7 ) = 2058 where n ∈ N, then the value of n is
4! 7!. (a) 3 (b) 4 (c) 5 (d) 6
(d) The 6 Indian classic songs are in a specified order
is 10P4. paragraph for Question no. 41 to 43
If a set A has n elements then the number of subsets
n
Cr 2r +1 n
of A containing exactly r elements is nCr . The number
33. Let for n ∈ N, f (n) = ∑ (−1)
r
then
r =0 (r + 1)(r + 2) of all subsets of A is 2n. Now answer the following
(a) f(2n) = f(2n + 1) (b) f(n) = f(n + 1) questions. A set A has 7 elements. A subset P of A is
(c) f(2n) = f(2n – 1) (d) f(2011) = f(2012) selected. After noting the elements they are placed back
in A. Again subset Q is selected. Then the number of
34. Which of the following is/are true?
ways of selecting P and Q such that
(a) 56 − 5C1 ⋅ 46 + 5C2 ⋅ 36 − 5C3 ⋅ 26 + 5C4 ⋅16 = 6C2· 5!
41. P, Q have no common element is
(b) 56 − 6C1 ⋅ 55 + 6C2 ⋅ 45 − 6C3 ⋅ 35 + 6C4 ⋅ 25 (a) 2830 (b) 124 (c) 3432 (d) 2187
– 6C1 · 15 = 0
42. P and Q have exactly 3 elements in common is
(c) 66 − 6C1 ⋅ 56 + 6C2 ⋅ 46 − 6C3 ⋅ 36 + 6C4 ⋅ 26 (a) 2835 (b) 125 (c) 3430 (d) 2185
– 6C5 · 16 = 720
43. P and Q have equal number of elements is (P and Q
5 6 5 6 5 6 5 6 5 may be null sets)
(d) 6 − C1 ⋅ 5 + C2 ⋅ 4 − C3 ⋅ 3 + C4 ⋅ 2
– 6C5 · 15 = 5C2 · 6! (a) 2834 (b) 120 (c) 3432 (d) 2180

mathematics today | december ‘15 37


paragraph for Question no. 44 to 46 (B) The number of permutations (q) 11! × 4 !
Two numbers x and y are drawn without replacement in which S occurs in first place
from the set of the first 15 natural numbers. The number 3 ! × (2 !)2
and R occurs in the last place
of ways of drawing them such that is
44. x3 + y3 is divisible by 3 (C) The number of permutations in (r) 11!
(a) 24 (b) 32 (c) 35 (d) 65 which none of the letters S, T, R 3! × 2 !
2 2
45. x – y is divisible by 5 occur in first 7 positions is
(a) 21 (b) 33 (c) 30 (d) 69 (D) The number of permutations in (s) 12 !
which the letters A, S, R occur
(2 !)4
4 4
46. x – y is divisible by 5
in even positions is
(a) 57 (b) 64 (c) 69 (d) 72
49. Match the following
section-iV
matrix-match type column I column II
(A) Nu m b e r o f w a y s o f (p) C3 21
47. Match the following:
There are 2 Indian couples, 2 American couples and selecting 8 balls out of
one unmarried person an unlimited collection
of red, blue, green and
column I column II
yellow balls is
(A) The total number of ways in (p) 22680
(B) N u m b e r o f 4 d i g i t (q) Number of ways
which they can sit in a row
numb ers having t he of distributing
such that an Indian wife and
sum of the digits equal 12 apples to
an American wife are always
to 9 is four people with
on either side of the unmarried
each one getting
person, is
atleast one
(B) The total number of ways in (q) 5760
(C) Nu m b e r o f w a y s o f (r) 6
which they can sit in a row
arranging 3 identical
such that the unmarried person
red balls, 20 identical
always occupy the middle
white balls in a row so
position, is
that no two red balls are
(C) The total number of ways in (r) 40320 together is
which they can sit around a 11
circular table such that an (D) Exponent of 3 in 17! is (s) C3
Indian wife and an American section-V
wife are always on either side of
integer Answer type
the unmarried person, is
(D) If all the nine persons are to be (s) 24320 50. If n > 1 is the smallest integer with the property
interviewed one by one then that n2(n – 1) is divisible by 2009, then the integral part
the total number of ways of n
of is
arranging their interviews such 8
that no wife gives interview 51. The number of onto functions which are non-
before her husband, is decreasing from A = {1, 2, 3, 4, 5} to B = {7, 8, 9} is
48. Match the following: 52. Let S = {1, 2,3, …, n}, If X denote the set of all
Consider all possible permutations of the letters of the subsets of S containing exactly two elements, then the
word MASTERBLASTERS. value of ∑ (min A) is n + 1Cl then l =
column I column II A∈X
53. If one test (on screening paper basis) was
(A) The number of permutations (p) (7 !)2
containing the word RAAT is conducted on Batch A, maximum number of marks is
3 !(2 !)4 (90 × 3) = 270. 4 students get the marks lower than 80.

38 mathematics today | december ‘15


9
Coaching institute decide to inform their guardians, V : When x = 8 and y = 1 C1 = 9(1 1 1 1 1 1 1 1 2)
that is why their result card were sent to their home. VI : When x = 10 and y = 0 1(1 1 1 1 1 1 1 1 1 1)
The number of ways, in which all the letters were put in Hence, total no. of ways
wrong envelopes, is = 1+ 15 + 35 + 28 + 9 + 1 = 89.
100
54. The exponent of 7 in C50 is 3. (d) : Number of arrangements in which two are
55. The unit digit in 1! + 2! + 3! + ….. + 49! is identical of one kind, two are identical of another kind
and one letter different from the remaining two letters
56. Nine hundred distinct n-digit numbers are to be
formed using exactly the three digits 2, 5 and 7. The is 2C1 × 5 ! = 60.
smallest value of n for which this is possible, is (2 !)2
57. The number of different ordered triplets (a, b, c), a, Number of arrangements in which 2 are identical of
b, c ∈I such that these can represent sides of a triangle 5!
one kind and the rest are different is 2C1 × = 120
whose perimeter is 21, is 9k+10,then k is 2!
4. (b) : A = {x1, x2, x3, x4, x5, x6, x7},
58. Out of 5 apples, 10 mangoes and 15 oranges, the
B = {y1, y2, y3}
number of ways of distributing 15 fruits each to two
n f : A → B is onto such that f(x) = y2
persons, is n, then is equal to
22 Since exactly three elements of A has image y2
59. The number of divisors of the form 2n + 1 (n ≥ 1) \ it can be done in 7C3 ways.
of the integer 120 is Remaining four elements of A is associated with 2
elements of B in 24 – 2C1(2 – 1)4 = 14 ways
solutions Total no. of onto functions = 7C3 × 14
2 2
1. (a) : 36 = 2 × 3 5. (b) : Since 3 does not occur in 1000, we have to
From 2 to 2000, number of multiples of 2 are count the number of times 3 occurs when we list the
 2000  integers from 1 to 999. Any number between 1 and
 2  = 1000 999 is of the form xyz, where 0 ≤ x, y, z ≤ 9. Let us
 
From 2 to 2000, number of multiples of 3 are first count the number in which 3 occurs exactly once.
 2000  Since 3 can occur at one place in 3C1 ways, there are
 3  = 666 3
C1 (9 × 9) = 3 × 92 such numbers. Next, 3 can occur
 
in exactly two places in (3C2)(9) = 3 × 9 such numbers.
From 2 to 2000, number of multiples of 6 are
Lastly, 3 can occur in all three digits in one number
 2000 
 6  = 333 only. Hence, the number of times 3 occurs is
  1 × (3 × 92) + 2 × (3 × 9) + (3 × 1) = 300.
Number of possible 'n' are
6. (c) : a + b + c = 21 ⇒ b + c > a
= 1999 – [1000 + 666 – 333] = 666
⇒ a + b + c > 2a ⇒ 2a < 21 ⇒ a ≤ 10.
2. (d) : A flight of stairs has 10 steps. A person can So, 1 ≤ a, b, c ≤ 10.
go up the steps one at a time, two at a time or any The cases when a > b > c are (10, 9, 2), (10, 8, 3), (10, 7, 4),
combination of 1s and 2s. (10, 6, 5), (9, 8, 4), (9, 7, 5) and (8, 7, 6). So, number
Let x + 2y = 10 of cases when a, b, c are all distinct is 7 × 3! = 42.
where x is the number of times he takes single steps The cases when a = b > c or a > b = c are (10, 10, 1),
and y is the number of times he takes two steps. (9, 9, 3), (8, 8, 5) and (9, 6, 6). So, number of cases
cases total no. of ways when two same and 1 different is 4 × (3! /2!) = 12.
5! The cases when a = b = c is (7, 7, 7). The total number
I : When x = 0 and y = 5 = 1(2 2 2 2 2)
5! of ordered triplets = 42+12 + 1 = 55.
6!
II : When x = 2 and y = 4 = 15(11 2 2 2 2 ) 7. (b) : The letters other than vowels are : PRMTTN
2! 4! Number of permutations with no two vowels together
7! 6! 7
III : When x = 4 and y = 3 = 35(1111 2 2 2) is × C5 × 5!
4 !3! 2!
8! Further among these permutations, the number of
IV : When x = 6 and y = 2 = 28(111111 2 2)
2!6! cases in which T's are together is 5! × 6C5 × 5!

mathematics today | december ‘15 39


So, the required number Q xn is positive.
6! 7
= C5 × 5! – 5! × 6C5 × 5! = 57 × (5!)2 171 − 4n2 − 20n
2! \ > 0 ⇒ 4n2 + 20n − 171 < 0
4 ⋅ n!
8. (b) : 10 ≥ x – 1 ⇒ x ≤ 11 and 10 ≥ x
\ x ≤ 10 which is true for n = 1, 2, 3, 4.
10 Hence, the given sequence has 4 positive terms.
C
 10Cx −1 > 2 ⋅10 Cx ⇒ 1 > 2 ⋅ 10 x 14. (a) : Let x, y, z be the number of sticks received
Cx −1 by the children A, B and C. Then the line segments
10 − x + 1 22 formed by them form a triangle iff 1 ≤ x ≤ 5,
⇒ 1> 2⋅ ⇒ x > 22 − 2 x ⇒ x > 1 ≤ y ≤ 5, 1 ≤ z ≤ 5
x 3
\ n is the number of positive integral solutions of
1
⇒ x >7 \ x =8 the equation x + y + z = 12 where x, y, z ≤ 5.
3 ⇒ n = coefficient of x12 in (x + ... + x5)3
9. (b) : Let E, W, N, S stand for one unit movement = coefficient of x9 in (1 + x + x2 + x3 + x4)3
along +ve, –ve x-direction, +ve, –ve y-direction = coefficient of x9 in (1 – x5)3(1 – x)–3
respectively. The sequence of 6 steps are = coefficient of x9 in (1 – 3x5 + 3x10 – x15) (1 – x)–3
6! = 1 × 11C2 – 3 × 6C2 = 55 – 45 = 10
ENNNSS with = 60 ways
3!2 ! 15. (a) : n – 2C12 – 2 = 3 · n – 3C12 – 3
n–2
6! ⇒ C10 = 3 · n – 3C9
EEWNNS with = 180 ways
2!2! n−2
⇒ = 3 ⇒ n = 32
6! 10
EEEWWN with = 60 ways
3!2 ! 16. (c) : x1 + x2 + x3 + x4 = 9, x1 ≠ 0
The desired number = 60 + 180 + 60 = 300 Coefficient of x9 in (x + x2 + … + x9)(1 + x + … + x8)3
10. (a) : L.C.M. (p,q) = 223452 Coefficient of x8 in (1 + x + … + x8)4 = 8 + 4 – 1C8 = 11C3
p = 2a1 3b1 5c1 , q = 2a2 3b2 5c2 17. (d) : Strictly descending → 10C5
Strictly ascending → 9C5 (because zero can’t be at x1)
⇒ max. {a1, a2}= 2 ⇒ 5 ways
10
⇒ max. {b1, b2} = 4 ⇒ 9 ways C5 + 9 C5 = 2 · 9 C4 + 9 C4 = 3 · 9 C4 = 3 · 9 C5
⇒ max. {c1, c2} = 2 ⇒ 5 ways 18. (a) : Take 5 gaps _ _ _ _ _
\ K = 32 · 52 can be expressed as 1·3252 , 32·52 The digit ‘2’ can occupy any of 5 places
 3!  The digit ‘5’ can occupy any of 4 places
11. (c) : 3 1 1 5C3 ⋅ 2C1 ⋅ 1C1   The digit ‘7’ can occupy any of 3 places
 2! 
Remaining 2 places in 7 × 7 ways
 3! 
2 2 1 5C2 ⋅ 3C2 ⋅ 1C1   \ Total ways = 5 × 4 × 3 × 7 × 7 = 2940
 2! 
19. (d) : From the given condition,
12. (a) : x + y + z + 12 = 0, x, y, z are negative integers.
Let x = –a, y = –b, z = –c, a, b, c are positive  x   3x   y   3 y 
 + + + = 0
integers. 3  2  2  4 
Then, required number of points (x, y, z) x 3x y 3y
⇒ , , and must be integers.
= number of positive integral solutions of 3 2 2 4
a + b + c = 12 \ x = 6, 12, 18, 24
= coefficient of x12 in (x + x2 + ...)3 y = 4, 8, 12, 16, 20, 24, 28
11 × 10 \ No. of ordered pairs = 4 × 7 = 28
= coefficient of x9 in (1 – x)–3 = 11C2 = = 55
2 20. (b) : L.H.S.
n+3
195 P 3
13. (c) : We have, xn = n − n+1 3 , n ∈ N 3  n
Cr 
4 Pn Pn+1 n−1  n
Cr  n−1  
= ∑  = ∑n  n
Cr +1  
195 (n + 3)(n + 2)(n + 1) 171 − 4n2 − 20n n n
= − = r =0  Cr + Cr +1  r =0  Cr 1 + n 
4 ⋅ n! (n + 1)! 4 ⋅ n!   Cr  

40 mathematics today | december ‘15


n−1 3 24. (b) : Least digit used = 4
 r +1  n2 (n + 1)2 n2 \ We can use 4,5,6,7,8,9. But remember that atleast
= ∑  = =

r =0 n + 1 4(n + 1)3 4(n + 1) one 4 must be used _ _ _ _ (4 blanks)
1st blank can be filled in 6 ways.
n2 4
Now = ⇒ 5n2 − 16n − 16 = 0 2nd blank can be filled in 6 ways.
4(n + 1) 5
3rd blank can be filled in 6 ways.
4 4th blank can be filled in 6 ways.
n = 4 or n = – ⇒n=4
5 \ 4 blanks can be filled in 64 ways. But out of these,
21. (a) : Given A(1,2) and B(5,5). some may contain no 4 at all. Let us find them.
Difference of x-coordinates = 5 – 1 = 4 _ _ _ _ (4 blanks)
\ Exactly 4 rightward steps are needed. Each blank can be filled in 5 ways (by 5, 6, 7, 8, or 9)
Difference of y-coordinates = 5 – 2 = 3. \ 54 ways (no 4 at all)
\ Exactly 3 upward steps are needed. \ Atleast one 4 = 64 – 54 = 671.
note: Order of the steps is immaterial. 25. (c, d) : (1 + x)11 = 11C0 + 11C1x + 11C2x2 +
Denote each rightward step by R and each upward .... + 11C11x11 … (i)
step by U. (1 + x) = C0 + C1x + C2x + .... + 9C9x9
9 9 9 9 2
… (ii)
The problem is arranging the letters RRRRUUU
On multiply (i) & (ii) and compare coefficient of x11
7! on both sides and put x = 1
No. of arrangements = = 35
4 !3! 20
C11 = 11C119C0 + 11C109C1 + .... + 11C29C9
22. (b) : 1440 = 25⋅32⋅51 \ 20
C9 – 1 = 11C109C1 + ..... + 11C29C9
No. of divisors = (5 + 1)⋅(2 + 1)⋅(1 + 1) = 36
26. (a, b, c, d) : (a) a11 + a22 + a33 = 0, remaining ‘6’
Product of divisors = 1⋅2⋅3……. 480⋅720⋅1440. Here all
the 36 divisors are written in the increasing order. They elements can be filled in 76 ways.
can be clubbed into 18 pairs as shown below. (−3, 0, 3),(−2, 0, 2),(−1, 0,1)
 → 3 !⋅ 5 = 30
(1,1440), (2, 720), (3, 480) …… etc. (−3,1, 2),(3, −1, −2) 
\ Product of divisors = (1440)18 = 290⋅336⋅518
(–2, 1, 1), (2, –1, –1) → 3 · 2 = 6
= (23⋅3)30⋅36⋅518 = 2430⋅36⋅518
which is divisible by 24x (0, 0, 0) → 1
\ Maximum value of x = 30 (b) Each of 9 elements can be filled in 7 ways.
0 − − 
23. (a) : _ _ _ _ _ (5 blanks)  
1st blank can be filled in 9 ways (c)  0 −  , 3 elements can be filled in 73 ways.
2nd blank can be filled in 9 ways (since repetition is  0 
allowed).
(d) a11, a22, a33 filled in 7 ways, also a12 filled in 7 ways
3rd blank can be filled in 9 ways
then a21 filled in one way and so on
4th blank can be filled in 9 ways
Now, we have to fill the 5th blank carefully such that − − −
 
the number is divisible by 3. Add the 4 numbers in  − −  , 6 elements can be filled in 76 ways.
the first 4 blanks.  − 
If their sum is in the form 3n, then fill the last blank
by 3, 6 or 9 so that the sum of all digits is divisible 27. (a, b) : (i) The number of ways = 10C3 × 3!
by 3. = 720
If their sum is in the form 3n+1, then fill the last blank (ii) Each lady and her child can be arranged in a fixed
by 2, 5 or 8. order only.
If their sum is in the form 3n+2, then fill the last blank \ The total number of ways in which interview can
by 1, 4 or 7. 6!
be held = = 90
Therefore, in any case, the last blank can be filled in 2 !2 !2 !
3 ways only. (iii) case I: No box empty
\ Ans = 9 × 9 × 9 × 9 × 3 = 39 Then the number of ways = 3! = 6

mathematics today | december ‘15 41


case II: If one of the boxes is empty, then number of 35 - 37 :
ways = 3C1(23 – 2) = 18. Let a b c d e f g h i j be one of such numbers where
\ total number of ways = 6 + 18= 24 a b c d e f g h i j is some permutation of the digits
(iv) Total – (All red) – (All white) 0,1,2,3,4,5,6,7,8,9 where a ≠ 0.
= 11C4 – 5C4 – 6C4 = 330 – 5 – 15 = 310 Sum of digits of the number = 0+1+2+3+4+5+6+7+8+9=
28. (a, b, d) : The cases for a1 : {H, T} i.e., a1 = 2 45, which is divisible by 9 and hence the number is
The case for a2 : {HT, TH, TT}, a2 = 3 divisible by 9. But it is divisible by 11,111 also and
For n ≥ 3, If the first outcome is H then next just 11,111 is not divisible by 3 or 9. Therefore, the number
T and then an–2. If the first outcome is T then an–1 is divisible by11,111 × 9 = 99,999.
should follow. And a b c d e f g h i j = a b c d e × 105 + f g h i j
So, an = 1 × 1 × an–2 + 1 × an–1 ⇒ an = an–2 + an–1 = a b c d e × (99,999+1) + f g h i j
So, a3 = a1 + a2 = 5, a4 = 3 + 5 = 8 and so on. = a b c d e × 99,999 + a b c d e + f g h i j is divisible
29. (a, b, c, d) : 144 = 24⋅32 by 99,999.
(a) No. of divisors = (4 + 1)⋅(2 + 1) = 15 ⇒ a b c d e + f g h i j is divisible by 99,999.
(b) Sum of divisors = (1+2+22+23+24) (1+3+32) But a b c d e < 99,999
= 403 And f g h i j < 99,999
15 ⇒ a b c d e + f g h i j < 2 × 99,999
(c) Product of divisors = (144) 2 = (12)15 \ a b c d e + f g h i j = 99,999
(d) Sum of reciprocals of divisors ⇒ e+j=d+i=c+h=b+g=a+f=9
sum of divisors 403 35. (c) :
= = a b c d e f g h i j
144 144
5! 6! 1 0 2 3 4 8 9 7 6 5
30. (a, b, c, d) : (a ) (2 !) (b)
2! 2! 2! For smallest number, a must be 1 (since a cannot be 0)
4! (d) 360 – 120
(c) (2 !) and hence f = 8.
2! Then, b = 0 ⇒ g=9
31. (a, b, c) : If the sides are a, a, b, then the triangle c=2 ⇒ h=7
forms only when 2a > b, so for any a ∈ N, b can change d=3 ⇒ i=6
from 1 to 2a –1. When a ≤ 1004 then number of e=4 ⇒ j=5
triangles = 1 + 3 + 5 + … + (2(1004) – 1)= (1004)2 and if \ The smallest of such numbers is 1023489765 and
1005 ≤ a ≤ 2008, b can take any value from 1 to the digit in the ten’s place is 6.
2008. 36. (c) :
But a has 1004 possibilities, hence number of triangles
= 1004 × 2008 = 2(1004)2 a b c d e f g h i j
\ Total number of isosceles triangles = 3(1004)2 9 8 7 6 5 0 1 2 3 4
32. (a, b, c, d) : (a) 6C3·4C2·5!·5! = (5!)3 For greatest number, a = 9 ⇒ f = 0
(b) 6C1·9! (c) (6 + 1)! 4! (d) 10P4 Then, b = 8 ⇒ g = 1
n c=7 ⇒ h=2
1
33. (c, d) : f (n) = ∑ n+2 Cr +2 (−2)r +2
2(n + 1)(n + 2) r =0
d=6 ⇒ i=3
e=5 ⇒ j=4
 1 \ The greatest of such numbers is 9876501234 and
n+2  , if n = odd the digit in the unit place is 4.
(1 − 2) 
− 1 + 2(n + 2) = n + 2
= 
2(n + 1)(n + 2)  1 , if n = even 37. (a) :
 n + 1 a b c d e f g h i j
34. (a, c) : (a) Number of onto functions from a set 9 8 6 4 2 1 1 1 1 1
containing 6 elements to a set containing 5 elements The blank ‘a’ can be filled in 9 ways (except 0).
= 6C2 ⋅ 5 Then blank ‘f ’ can be filled in only one way (by 9–a).
(c) Number of onto functions from a set containing 6 Now, blank ‘b’ can be filled by any of the remaining
elements to a set containing 6 elements = 6 = 720 8 digits.

42 mathematics today | december ‘15


Then blank ‘g’ can be filled in only one way (by 9– b) 44- 46 :
Now, blank ‘c’ can be filled by any of the remaining 44. (c) : The natural numbers are written in rows
6 digits. 1 4 7 10 13
Then blank ‘h’ can be filled in only one way (by 9 – c). 2 5 8 11 14
Now, blank ‘d’ can be filled by any of the remaining 3 6 9 12 15
4 digits. x3 + y3 is divisible by 3 if and only if x + y is divisible
Then blank ‘i’ can be filled in only one way (by 9 – d). by 3. The numbers x and y are taken one from row 1
Now, blank ‘e’ can be filled by any of the remaining and other from row 2 or both from row 3. The desired
2 digits. number is
Then blank ‘j’ can be filled in only one way (by 9 – e). (5C1 × 5C1) + 5C2 = 25 + 10 = 35
\ The total number of such numbers 45. (b) : The numbers are written in rows
= 9 × 1 × 8 × 1 × 6 × 1 × 4 × 1 × 2 × 1 = 3456. 1 6 11
38 - 40 : 2 7 12
38. (c) : f(1) = 1, f(P n ) = P n−1 (P − 1), f(mn) = f(m) ⋅ f(n) 3 8 13
f(8n + 4) = f(4(2n + 1)) = f(4) ⋅ f(2n + 1) 4 9 14
5 10 15
= f(22 ) ⋅ f(2n + 1) = 2 ⋅ f(2n + 1)
x2 – y2 = (x + y)(x – y) is divisible by 5 ⇒ both x and
39. (b) : f(n) is odd. y are from any of these rows or one from row 1 and
⇒ f(Pn) is odd. other from row 4 or one from row 2 and other from
⇒ Pn – 1(P – 1) is odd. row 3 ⇒ desired number is
Q P is prime. The only value P can take is P = 2 5 × 3C2 + 2(3C1)2 = 15 + 18 = 33
\ f(2n)is odd. 46. (c) : x4 – y4 = (x2 + y2)(x – y)(x +y) is divisible by
⇒ 2n – 1(2 – 1) = 2n – 1 is odd. 5 ⇒ Both x and y are from any one row or one from
⇒ n – 1 = 0 ⇒ n = 1 \ f(1) = 1 = f(2) any one row and the other from other row of the first
40. (b) : f(7n) = 2058 4 rows. The desired number is
7n – 1(7 – 1) = 2058 5 × 3C2 + 6(3C1)2 = 15 + 54 = 69
7n – 1 = 343 47. a - r; b - r; c - q; d - p
n–1=3 ⇒ n=4 (A) One Indian wife and one American wife can be
41- 43 : selected in 2C1 × 2C1ways and keeping an unmarried
41. (d) : If P and Q have no common element, then person in between these two wives, the total number
each of the 7 elements have 3 choices. They are of linear arrangements are 2 C1 ×2 C1 × 7 × 2 = 40320
x ∈ P and x ∉ Q or x ∉ P and x ∈ Q or x ∉ P and
(B) Required number of ways = 8 = 40320
x∉Q
(C) Required number of ways
Hence, the number of ways of selecting P and Q
= 37 = 2187 = (7 − 1) × 2 × 2C1 × 2C1 = 5760
42. (a) : As in both sets three elements are common (D) Number of ways in which interviews can be
so, three elements can be choosen in 7C3 ways and arranged
rest of the elements can be chosen in any of the = 9 × 8C2 × 6C2 × 4C2 × 2C2 = 22680
above three ways in 34 ways. So, total number of ways 48. a - r; b - s; c - p; d - p
=7C3 × 34 =2835 AA, SSS, TT, EE, RR, M, B, L
43. (c) : Let P & Q have same number of elements i.e (A) Take RAAT as one unit. Therefore 10 + 1 = 11
r (0 ≤ r ≤ 7) 11!
⇒ Total number of ways in which P & Q have same units can be arranged in ways.
3 !× 2 !
number of elements
(B) After fixing S in first position and R in last position,
7
= ∑ 7 Cr ⋅7 Cr = (7 C0 )2 + (7 C1 )2 + (7 C2 )2 + .... + (7 C7 )2 the remaining 12 letters can be arranged in remaining
r =0 12 !
12 positions in ways.
= 3432 (2 !)4

mathematics today | december ‘15 43


(C) First 7 positions can be filled with A′s, E′s, M, B, L 100 !
7! 54. (0) : 100 C50 =
in ways. The remaining 7 positions can be filled 50 ! 50 !
(2 !)2 Exponent of 7 in 100 ! = 16
7!
with S′s, T′s, R′s in ways.  50   50 
3 !× (2 !)2 Exponent of 7 in 50! =   +  2  = 8
 7  7 
(D) 7 even positions can be filled with A′s, S′s, R′s
Exponent of 7 in (50!)2 = 16
7!
in ways. 7 odd positions can be filled with \ Exponent of 7 in 100C50 = 16 – 16 = 0
(2 !)2 × 3 ! 55. (3) : 1! +2! + 3! + 4! = 33
7! 5! = 120, 6! = 720, 7! = 5040
T′s, E′s, M, B, L in ways.
(2 !)2 8! = 40320, 9! = 362880
49. a - q,s; b - q,s; c - p; d - r Thus the unit digit of 1! + 2! + …. + 9! = 3
(A) Non-negative integral solutions of Also note that n! is divisible by 100 for all n ≥ 10.
\ Unit digit of 10! + 11! + ….. + 49! = 0
R + B + G + Y = 8 is 8 + 4 – 1C4 – 1 = 11C3
\ Unit digit of 1! + 2! + ……+ 49! = 3.
(B) D1 + D2 + D3 + D4 = 9, D1 ≠ 0
D1 – 1 + D2 + D3 + D4 = 8 56. (7) : For n = 6
8+4–1 3 × 3 × 3 × 3 × 3 × 3 = 729 < 900
C4 – 1 = 11C3
For n = 7
(C) Select ‘3’ places out of 21 places = 21C3 3 × 3 × 3 × 3× 3 × 3 × 3 = 2187 > 900
17   17  For n = 8
(D)   +  2  = 5 + 1 = 6
 3  3  Number of n-digits formed > 900
50. (5) : Since 2009 = 7 × 7 × 41. Therefore, 41 must Since the least n is required
divide n2(n – 1), which implies that 41 is a factor of \ n = 7.
either n or n – 1 . In particular, n ≥ 41. For n = 41, 57. (5) : a + b + c = 21
neither n nor (n – 1) is divisible by 7. For n = 42, ⇒ b + c > a ⇒ a + b + c > 2a
n2 is divisible by 72, since n is divisible by 7. Therefore, ⇒ 2a < 21 ⇒ a ≤ 10.
n = 42 is the smallest integer. So, 1 ≤ a, b, c ≤ 10
51. (6) : Images of 1,2,3,4,5 are respectively. The cases when a > b > c are (10, 9, 2), (10, 8, 3),
7,7,7,8,9; 7,7,8,8,9; 7,7,8,9,9 (10, 7, 4), (10, 6, 5), (9, 8, 4), (9, 7, 5) and (8, 7, 6).
7,8,8,8,9; 7,8,8,9,9; 7,8,9,9,9 So, number of cases when a, b, c are all distinct is
\ There are six functions. 7 × 3! = 42.
The cases when a = b > c or a > b = c are (10, 10, 1),
52. (3) : There are exactly (n – 1) subsets of S containing (9, 9, 3), (8, 8, 5) and (9, 6, 6). So number of cases
two elements having 1 as least element; exactly (n – 2) when two same and 1 different is 4 × (3! /2!) = 12.
subsets of S having 2 as least element and so on. The cases when a = b = c is (7, 7, 7). The total number
Thus of ordered triplets = 42 + 12 + 1 = 55.
Thus ∑ min( A) = (1)(n − 1) + 2(n − 2) + .... + (n − 1)(1) 58. (3) : x1+ x2 + x3 = 15
A∈X 0 ≤ x1 ≤ 5, 0 ≤ x2 ≤ 10, 0 ≤ x3 ≤ 15
n−1 n = coeff. of x15 in (1 – x6) (1 – x11) (1 – x16) (1 – x)–3
= ∑ r (n − r ) =n+1C3 n = 66
r =1 n
⇒ =3
53. (9) : The number of ways in which all the letters 22
are in wrong envelopes 59. (4) : Number of the form (2n+1), n ∈ I are all odd
 1 1 1 1 natural numbers.
= 4 ! 1 − + − + 
 1! 2 ! 3 ! 4 !  Thus we have to find all odd numbers dividing 120.
These numbers are 1, 3, 5, 15.
1 1 1  Hence, number of divisors = 4.
= 4 !  − +  = 12 − 4 + 1 = 9
 2 6 24  nn

44 mathematics today | december ‘15


PROBABILITY
*ALOK KUMAR, B.Tech, IIT Kanpur
This column is aimed at Class XII students so that they can prepare for competitive exams such as JEE Main/Advanced, etc. and
be also in command of what is being covered in their school as part of NCERT syllabus. The problems here are a happy blend
of the straight and the twisted, the simple and the difficult and the easy and the challenging.

terms and definitions used in probability z event


z random experiment An event is a subset of the sample space.
An experiment is called random if z simple event
(i) all the outcomes of the experiment are known An event is called a simple event, if it is a singleton
in advance subset of the sample space S.
(ii) the exact outcome of any specific performance z Compound event
of the experiment is unpredictable A subset of the sample space S which contains more
i.e. not known in advance. than one element is called a compound event.
z sample space z equally likely events
A set whose elements represent all possible A set of events is said to be equally likely if taking
outcomes of a random experiment is called the into consideration all the relevant factors there is no
sample space and is usually represented by ‘S’. reason to expect one of them in preference to the
Consider the experiment of tossing a die. If we others. For example when a fair coin is tossed, the
are interested in the number on the top face, then occurrence of a tail or a head are equally likely.
sample space would be S1 = {1, 2, 3, 4, 5, 6}. z mutually exclusive events
If we are interested only in whether the number A set of events is said to be mutually exclusive
is even or odd, then sample space is S2 = {even, if the occurrence of one of them precludes the
odd}. Clearly more than one sample space can be occurrence of any of the remaining events. For
used to describe the outcomes of an experiment. example, when we throw a pair of dice, the
In this case ‘S1’ provides more information than
events “a sum of 5 occurs”, “a sum of 7 occurs”
‘S2’. If we know which element in S1 occurs, we
and “a sum of 9 occurs” are mutually exclusive.
can tell which outcome in S2 occurs; however,
knowledge of what happens in S2 in no way helps In set theoretic notation, events A1, A2, ...., Am
us to know which element in S1 occurs. are mutually exclusive if Ai ∩ Aj = f for i ≠ j and
In general it is desirable to use a sample space that 1 ≤ i, j ≤ m.
gives the maximum information concerning the z exhaustive events
outcomes of the experiment. A system of events is said to be exhaustive if on
z sample point each performance of the experiment at least one
Each element of the sample space is called a of the events of the system is must to occur. In
sample point. set theoretic notation, events A1, A2, .... , Am are
* Alok Kumar is a winner of INDIAN NAtIoNAl MAtheMAtIcs olyMpIAD (INMo-91).
he trains IIt and olympiad aspirants.
mathematics today | december ‘15 45
m (i) If A and B are any two events, then
exhaustive if  Ai = S . For example on throwing
i =1 P(A ∪ B) = P(A) + P(B) – P(A ∩ B).
of a die, the events {1, 2}, {2, 3, 4}, {5} and (ii) If A and B are mutually exclusive, then
{4, 5, 6} form an exhaustive system of events. P(A ∪ B) = P(A) + P(B)
(iii) If A is any event, then
axiomatiC definition of probability
P ( A) = P(A′) = 1 – P(A)
z probability with discrete sample space containing
(iv) P(A ∩ B′) = P(A) – P(A ∩ B)
equally likely points
(v) P(exactly one of A and B occurs)
If there are n exhaustive mutually exclusive and
= P(A ∩ B′) + P(A′ ∩ B)
equally likely outcomes of an experiment and m
= P(A) + P(B) – 2P(A ∩ B)
of them are favourable to an event A, then the
= P(A ∪ B) – P(A ∩ B)
probability of the happening of A is equal to
m some theorems
and it is denoted by P(A). Clearly P(A) is a (i) If A ⊆ B, then
n
(a) P(A) ≤ P(B) and
non-negative number not greater than unity. So
(b) P(B – A) = P(B) – P(A)
0 ≤ P(A) ≤ 1.
If probability of happening of an event A is 1, n  n
(ii) P   Ai  ≤ ∑ P ( Ai ) and equality holds if and
then A is called certain event and if probability of  i =1  i =1
happening of an event A is zero, then A is called only if events Ai, i = 1, 2, ..., n are exclusive.
impossible event. n 
(iii) P   Ai  ≤ 1 and equality holds if and only
z odds in favour and odds against an event  i =1 
As a result of an experiment, if p of the outcomes if events Ai, i = 1, 2, ...., n are exhaustive.
are favourable to an event E and q of the outcomes n
are against it, then we say that odds are p to q in (iv) ∑ P ( Ai ) = 1 if events Ai, i = 1, 2, ..., n are
favour of E or odds are q to p against E. i =1
exclusive and exhaustive.
\ Odds in favour of an event E
note : If events A1, A2, ..., An are exclusive, then
number of favourable cases p
= = n  n
number of unfavourable cases q P   Ai  = ∑ P ( Ai ). This is called the rule of sum.
 i =1  i =1
and odds against an event E
number of unfavourable cases q some useful formulae
= = (i) for two events A and B
number of favourable cases p
(a) P(atleast one out of them)
If odds in favour of an event E are p : q then = P(A) + P(B) – P(A ∩ B)
the probability of the occurrence of that event (b) P(exactly one out of them)
p = P(A) + P(B) – 2P(A ∩ B)
is .
p+q (ii) for three events A, B and C
Similarly, the probability of the non-occurrence (a) P(atleast one out of them)
q = P(A) + P(B) + P(C) – P(A ∩ B)
of that event is . – P(B ∩ C) – P(C ∩ A) + P(A ∩ B ∩ C)
p+q
(b) P(atleast two out of them)
set theoretiC prinCiples = P(B ∩ C) + P(C ∩ A) + P(A ∩ B)
If ‘A’ and ‘B’ be any two events of the sample – 2P(A ∩ B ∩ C)
space, then A ∪ B would stand for occurrence (c) P(exactly two out of them)
of atleast one of them and A ∩ B stands for = P(B ∩ C) + P(C ∩ A) + P(A ∩ B)
simultaneous occurrence of A and B. A (or A ′) – 3P(A ∩ B ∩ C)
stands for non–occurrence of A. A ∩ B or (d) P(exactly one out of them)
(A′ ∩ B′) stands for non–occurrence of both = P(A) + P(B) + P(C) – 2P(B ∩ C)
A and B. – 2P(C ∩ A) – 2P(A ∩ B) + 3P(A ∩ B ∩ C)

46 mathematics today | december ‘15


independent experiments Thus required probability = nCr pr qn – r
If two random experiments are performed 4
1 2
1
10
separately i.e., their outcomes are independent = 5C4     =
 3   3  243
of each other, then the experiments are called
independent experiments. Conditional probability
for example The probability of occurrence of an event B
(i) Consider the tossing of a coin twice, clearly when it is known that some event A has already
the outcome for the second toss is not effected occurred, is called the conditional probability and is
by the result of the first toss. So the two tosses denoted by P(B | A). The symbol P(B | A) is usually
are independent random experiments. read as probability of B, given A. Consider two
(ii) In throwing of a die and a coin together events A and B. When it is known that event ‘A’
or one after the other are independent has occurred, it means that sample space would
experiments. reduce to that sample points representing event
remarks: A. Now for P(B | A), we must look for the sample
(i) In drawing of two cards, without replacement, points representing the simultaneous occurrence of
from a well-shuffled ordinar y pack of A and B i.e., sample points in A ∩ B.
52-playing cards, the two draws are not n( A ∩ B)
independent experiments. n( A ∩ B) n(S) P ( A ∩ B)
Now , P (B | A) = = =
(ii) Let E1 and E2 be two independent random n( A) n( A) P ( A)
experiments. Let A be an event of experiment n(S)
E 1 only and B be an event of experiment P ( A ∩ B)
Thus, P (B | A) = , where 0 < P(A) ≤ 1.
E2 only. Then P ( A)
P(A occurs in E 1 and B occurs in E 2 ) = Similarly, P ( A | B) = P ( A ∩ B) , where 0 < P(B) ≤ 1
P(A) ⋅ P(B). P ( B)
For example, if a coin and a die are thrown together
P ( A) ⋅ P (B | A), P ( A) > 0
and A = {H}, B = {1, 2, 3, 4}, then P (head on coin Hence, P ( A ∩ B) = 
and a number ≤ 4 on die) = P(A) · P(B)  P (B) ⋅ P ( A | B), P (B) > 0
1 4 1 independent events
= × = . pairwise independent events
2 6 3 z

z binomial distribution for successive events Two events A and B are said to be independent
If an experiment is repeated ‘n’ times and if occurrence or non–occurrence of one does not
probability of occurrence of an event of one trial affect the occurrence or non–occurrence of the
is p, then other.
(i) Probability of occurring the event exactly ‘r’ i.e., P(B | A) = P(B), P(A) ≠ 0.
times is nCr pr (1 – p)n – r. P ( A ∩ B)
⇒ P (B | A) = = P ( B)
(ii) Probability of occurring the event atleast ‘r’ P ( A)
n
times is ∑ nCk pk (1− p)n− k . ⇒ P(A ∩ B) = P(A) · P(B)
k =r If the events are not independent, then they are
(iii) Probability of occurring the event atmost ‘r’ said to be dependent.
r z mutually independent events
times is ∑ nCk pk (1 − p)n− k . Three events A, B and C are said to be mutually
k =0
independent if,
For example if a die is thrown five times and we
P(A ∩ B) = P(A) · P(B), P(A ∩ C) = P(A) · P(C),
want the probability of occurrence of a composite
P(B ∩ C) = P(B) · P(C)
number four times, then we have
and P(A ∩ B ∩ C) = P(A) · P(B) · P(C)
2 1 1 2 These events would said to b e pair–wise
p = = , q = 1 − = and n = 5, r = 4.
6 3 3 3 independent if,

mathematics today | december ‘15 47


P(A ∩ B) = P(A) · P(B), P(B ∩ C) = P(B) · P(C) 4. A man throws a die until he gets a number bigger
and P(A ∩ C) = P(A) · P(C). than 3. The probability that he gets 5 in the last throw is
Thus, mutually independent events are pair–wise (a) 1/3 (b) 1/4 (c) 1/6 (d) 1/36
independent but the converse may not be true. 5. A randomly selected year is containing 53 Mondays
z rule of multiplication then probability that it is a leap year, is
Suppose A1, A2, ..., An be n events such that (a) 2/5 (b) 3/5 (c) 4/5 (d) 1/5
P(A1 ∩ A2 ∩ .... ∩ An) ≠ 0, then 6. A and B are events such that P(A) = 0.3, P(A ∪ B)
n  = 0.8. If A and B are independent then P(B) =
P   Ai  = P ( A1 ) ⋅ P ( A2 | A1 ) ⋅ P ( A3 |( A1 ∩ A2 )) (a) 1/7 (b) 3/7 (c) 5/7 (d) 6/7
 i =1 
... P(An | (A1 ∩ A2 ∩ .... ∩ An – 1)) 7. If the letters of the word MATHEMATICS are
arranged arbitrarily, the probability that C comes before
total probability theorem
E, E before H, H before I and I before S is
Suppose A1, A2 ..., An are mutually exclusive and
1 1 1 1
exhaustive events, then for any event B, we can (a) (b) (c) (d)
n n 75 24 120 720
write  (B ∩ Ai ) ⇒ P (B) = ∑ P (B ∩ Ai ) 8. A signal which can be green or red with probability
i =1 i =1
as events B ∩ Ai, 4 1
i = 1, 2, ..., n are exclusive.   and   respectively is received by the station A
5 5
n and transmitted to B. The probability that each station
⇒ P (B) = ∑ P (B | Ai ) ⋅ P ( Ai )
i =1
receive signal correctly = 3/4. If signal received by B is
green. The probability that original signal was green is
bayes’ theorem (a) 3/5 (b) 6/7 (c) 20/23 (d) 9/20
Suppose A1, A2, ...., An are mutually exclusive and
exhaustive set of events. 9. Four identical oranges and six distinct apples (each
a different variety) are distributed randomly into five
Then the conditional probability that Aj happens
distinct boxes. The probability that each box gets a total
(given that B has happened) is given by
of two objects is
P ( A j ∩ B) P ( A j ) ⋅ P (B | A j ) 813 162
P ( A j | B) = = n , (a) (b)
P ( B) 109375 21875
∑ P ( A j ) ⋅ P (B | A j )
j =1 323 151
where j = 1, 2, ....., n. (c) (d)
43750 21875
Problems
10. A bag contains two red balls and two green balls. A
SECTION-I person randomly pulls out a ball, replacing it with a red
Single Correct Answer Type ball regardless of the colour.What is the probability that
1. If a and b are chosen randomly from the set all the balls are red after three such replacement ?
(a) 3/8 (b) 7/16 (c) 5/32 (d) 9/32
consisting of numbers 1, 2, 3, 4, 5, 6 with replacement,
2/ x 11. Three fair coins are tossed simultaneously. Let E be
 ax + bx  the event of getting three heads or three tails, F be the
then probability that lim   = 6 is
x →0  2  event of atleast two heads and G be the event of atmost
two heads then which of the following is true?
(a) 1/3 (b) 1/4 (c) 1/9 (d) 2/9
(a) P(E ∩ F) = P(E) · P(F)
2. A coin is tossed 7 times. Then the probability that (b) P(E ∩ G) = P(E) · P(G)
at least 4 consecutive heads appear is (c) P(F ∩ G) = P(F) · P(G)
(a) 3/16 (b) 5/32 (c) 5/16 (d) 1/8 (d) none of these
3. A fair coin is tossed 5 times then probability that 12. Consider all the 3 digit numbers abc (where a ≠ 0).
two heads do not occur consecutively (No two heads If a number is selected at random then the probability
come together) is that the number is such that a + b + c = 6 is
1 15 13 7 2 7 7 7
(a) (b) (c) (d) (a) (b) (c) (d)
16 32 32 16 15 75 600 300

48 mathematics today | december ‘15


13. If three numbers are chosen randomly from the set drawn. The probability that exactly 3 balls are common
{1, 3, 32, ..., 3n} without replacement then the probability to the drawings is
that they form an increasing geometric progression is (a) 8/21 (b) 6/19 (c) 5/24 (d) 9/22
3 3 22. The probability that in a family of 5 members,
(a) , if n is odd (b) , if n is even
2n 2n exactly two members have birthday on Sunday is
3n 3n 12 × 53 10 × 63
(c) 2 , if n is even (d) , if n is odd (a) (b)
n −1 2(n2 − 1) 75 57
14. A fair coin is tossed until one of the two sides 12 × 62 10 × 63
occurs twice in a row. The probability that the number (c) (d)
57 75
of tosses required is even is
(a) 1/3 (b) 2/3 (c) 1/4 (d) 3/4 23. If a is an integer lying in the closed interval
[–5, 30], then the probability that the graph of
15. A determinant is chosen at random from the set
y = x2 + 2(a + 4)x – 5a + 64 is strictly above the x-axis
of all determinants of order 2 with elements 1 and 0
is
only. The probability that the value of the determinant
(a) 2/9 (b) 1/6 (c) 1/2 (d) 5/9
is positive is
(a) 1/8 (b) 3/16 (c) 5/16 (d) 1/16 24. The probabilities of A, B, C solving a problem
16. Two numbers x and y are chosen at random from 1 1 1
independently are respectively , , . If 21 such
the set {1, 2, 3, ...., 3n}. The probability that x2 – y2 is 4 5 6
divisible by 3 is problems are given to A, B and C, then the probability
5n − 3 2n that atleast 11 problems can be solved by them is
(a) (b) 11
3(3n − 1) 3(3n − 1) 1 1
(a) 21 C11   (b)
5n n 2 2
(c) (d)
3(3n − 1) 3n − 1 11
1 21 211
17. A and B throw alternatively with a pair of dice. (c)   (d) C11
2 321
A wins if he throws a sum 6 before B throws 7 and B
wins if he throws a sum 7 before A throws sum 6. If A 25. In a test student either guesses or copies or knows
starts the game, his chance of winning is the answer of a multiple choice questions with four
30 31 15 60 choices in which exactly one choice is correct. The
(a) (b) (c) (d) probability that he makes a guess is 1/3; The probability
61 61 61 61
that he copies the answer is 1/6. The probability that
18. An unbiased coin is tossed 12 times .The probability his answer is correct given that he copied it is 1/8. Find
that at least 7 consecutive heads show up is the probability that he knew the answer to the question
1 1 9 7 given that he correctly answered it is
(a) (b) (c) (d)
128 64 256 256
29 24 1 1
19. 10 apples are distributed at random among 6 (a) (b) (c) (d)
35 29 7 9
persons. The probability that atleast one of them will
receive none is SECTION-II
14 Multiple Correct Answer Type
6 C4 137 143
(a) (b) (c) (d)
143 15
C5 143 137 26. Let us define the events A and B as
A : An year chosen at random contains 29 February.
20. A certain type of missile hits its target with B : An year chosen at random has 52 Fridays.
probability 0.3. The number of missiles that should be If P(E) denotes the probability of happening of event
fired so that there is atleast an 80% probability of hitting E, then
a target is 2 23
(a) 3 (b) 4 (c) 5 (d) 6 (a) P (B ) = (b) P (B) =
7 28
21. From a bag containing 10 distinct balls, 6 balls are 2 5
drawn simultaneously and replaced. Then 4 balls are (c) P ( A | B ) = (d) P ( A | B) =
5 23

mathematics today | december ‘15 49


27. A, B are two events of a random experiment such (b) Probability that exactly one blue ball is drawn
that P ( A) = 0.3, P (B) = 0.4 and P ( A ∩ B ) = 0.5. Then is 0.2
(a) P(A ∪ B) = 0.9 (b) P (B ∩ A) = 0.2 (c) Probability that all the drawn balls are red given
that all the drawn balls are of the same colour
(c) P ( A ∪ B ) = 0.8 (d) P (B |( A ∪ B )) = 0.25
is 0.2
28. Contents of the two urns is as given in this table. A (d) Probability that atleast one red ball is drawn
fair die is tossed. If the face 1, 2, 4 or 5 comes, a marble is 0.6
is drawn from the urn A otherwise a marble is chosen
33. The probability that exactly one of the independent
from the urn B.
events A and B occurs is equal to
urn red White blue (a) P(A) + P(B) – 2P(A ∩ B)
marbles marbles marbles (b) P(A) + P(B) – P(A ∩ B)
A 5 3 8 (c) P ( A) + P (B ) − 2P ( A ∩ B )
B 3 5 0 (d) P(A) + P(B) – 3P(A ∩ B)
Let E1 : Denote the event that a red marble is chosen. 34. If P(A) = 3/5 and P(B) = 2/3, then
E2 : Denote the event that a white marble is chosen. 2 4 3
E3 : Denote the event that a blue marble is chosen. (a) P ( A ∪ B) ≥ (b) ≤ P ( A ∩ B) ≤
3 15 5
Then, 1 3
(a) the event E1, E2 and E3 are equiprobable. (c) P ( A ∩ B ) ≤ (d) P ( A ∪ B) ≥
3 5
(b) P(E1), P(E2), P(E3) are in A.P.
SECTION-III
(c) if the marble drawn is red, the probability that it
came from the urn A is 1/2. Comprehension Type
(d) if the marble drawn is white, the probability that paragraph for Question no. 35 to 37
the face 5 appeared on the die is 3/32. A lot contains 10 defective and 10 non-defective
29. A random variable x takes values 0, 1, 2, 3, ... with bulbs. 2 bulbs are drawn at random, one at a time
x with replacement. We define the events A, B and C
1 as follows:
probability proportions to (x + 1)   , then
5 A = {The first bulb is defective}.
16 112 B = {The second bulb is non-defective}.
(a) P (x = 0) = (b) P (x ≤ 1) =
25 125 C = {B oth bulbs are either defective or non-
9 defective}.
(c) P (x ≥ 1) = (d) none of these
25 35. P(A) will be equal to
30. If A and B are two independent events such that (a) 1/4 (b) 3/4 (c) 1/2 (d) 1/3
P(A′ ∩ B) = 2/15 and P(A ∩ B′) = 1/6, then P(B) can 36. P(B)·P(C) will be equal to
be (a) 1/4 (b) 1/2 (c) 1/16 (d) 1/8
(a) 1/5 (b) 1/6 (c) 4/5 (d) 5/6
37. P(A ∩ B ∩ C) will be equal to
31. If M and N are any two events, the probability that (a) 0 (b) 1/2 (c) 1/4 (d) 1/8
the exactly one of them occurs is
(a) P(M) + P(N) – 2P(M ∩ N) paragraph for Question no. 38 to 40
Let S be the set of first 18 natural numbers. Then
(b) P(M) + P(N) – P(M ∩ N)
(c) P(Mc) + P(Nc) – 2P(Mc ∩ Nc) 38. The probability of choosing {x, y} ⊆ S such that
(d) P(M ∩ Nc) + P(Mc ∩ N) x3 + y3 is divisible by 3 is
(a) 1/3 (b) 1/6 (c) 1/5 (d) 1/4
32. A bag initially contains one red ball and two blue
balls. An experiment consists of selecting a ball at 39. The probability of choosing {x, y, z} ⊆ S such that x,
random, noting its colour and replacing it together with y, z are in A.P. is
an additional ball of the same colour. If three such trials (a) 1/17 (b) 2/17 (c) 5/34 (d) 3/34
are made, then 40. The probability of choosing {x, y, z} ⊆ S such that
(a) Probability that atleast one blue ball is drawn no two of the numbers x, y, z are consecutive is
is 0.9 (a) 35/51 (b) 2/17 (c) 4/17 (d) 6/17

50 mathematics today | december ‘15


SECTION-IV 44. Five unbiased cubical dice are rolled simultaneously.
Matrix-Match Type
Let m and n be the smallest and the largest number
appearing on the upper faces of the dice, then match the
41. ‘n’ whole numbers are randomly chosen and following event given in Column I with their respective
multiplied. probabilities given in Column II.
Column-i Column-ii
Column-i Column-ii
The probability that the 8n − 4n 5
A. last digit is 1, 3, 7 or p. 2
A. m = 3 p.  
9 is 10n 3
The probability that the 5n − 4n 5
2 1 1
5 4
B. last digit is 2, 4, 6, 8 is q. B. n = 4 q.   +   −  
10n 3 3 2
The probability that the 4n 5 5
C. last digit is 5 is r. 5 1
10n C. 2 ≤ m ≤ 4 r.   −  
6 3
The probability that the 10n − 8n − 5n + 4n 5 5
D. last digit is zero is s. 2 1
10 n D. m = 2 and n = 5 s.   −  
3 2
42. Match the functions given in Column I with their
SECTION-V
domain in Column II. A is a set containing n elements.
A subset P of A is chosen at random. The set A is Integer Answer Type
reconstructed by replacing the elements of the subset 45. If n(X) = (k + 1), then the probability of selecting 2
P. A subset Q of A is again chosen at random, then the subsets A and B of the set ‘X’ such that B = Ac is equal to
probability that 1
, where m – k is equal to
Column-i Column-ii 2m−1

n(3n−1 ) 46. A bag contains 10 different balls. Five balls are


A. P ∩ Q = f p. drawn simultaneously and then replaced and then
4n
seven balls are drawn. The probability that exactly three
B. P ∩ Q is a singleton q. (3/4)n balls are common to the two drawn is p, then the value
2n of 12p is
Cn
C. P ∩ Q contain 2 elements r.
4n 47. If the sides of a triangle are decided by throwing a
die thrice, the probability that the triangle is isosceles or
D. |P| = |Q|, where |X| = number 3n−2 (n − 1)n equilateral is 1/k, then k =
s.
of elements in X 2(4n ) 48. Four identical dice are rolled once, the probability
43. There are 10 pairs of shoes in a cupboard from L
that all the numbers on them are primes is , then
which 4 shoes are taken at random. If P(E) denotes the L= 8L + 2
probability of the event E .
Match the following: 49. If {x, y} is a subset of the first 30 natural numbers,
then the probability that x3 + y3 is divisible by 3, is S/9
Column-i Column-ii then S =
99
A. P (getting no pair) p. 50. An unbiased coin is tossed. If the result is a head, a
323 pair of unbiased dice is rolled and the number obtained
96 by adding the numbers on the two faces is noted. If
B. P (getting atleast one pair) q.
323 the result is a tail, a card from a well shuffled pack of
224 eleven cards numbered 2, 3, 4, 5, ...., 12 is picked and
C. P (getting exactly two pairs) r. the number on the card is noted. The probability that
323
3 P
D. P (getting exactly one pair) s. the noted number is either 7 or 8 is , then the digit
323 792
in tens place of P is

mathematics today | december ‘15 51


51. 1, a1, a2, a3, ..., ak are divisors of number 4 1
N = 2n – 1(2n – 1), where 2n – 1 is a prime number and \ Required probability = =
6×6 9
1 < a1 < a2 < a3 < .... < ak, then value of
 1 1 1  2. (b) : Let H denote the head, T the tail.
1 + a + a + .... + a  is * Any of the head or tail
1 2 k 1 1
52. If the integers m and n are chosen at random from P (H ) = , P (T ) = , P (*) = 1
2 2
{1, 2, 3, ....., 100} then the probability that a number of 4
the form 7m + 7n is divisible by 5 is equal to 1/k. The 1 1
HHHH * ** =   × 1 =
numerical value of k is 2 16
5
53. A man parks his car among n cars standing in a 1 1
THHHH * * =   × 1 =
row, his car not being parked at an end, on his return 2 32
he finds that exactly m of the n cars are still there, 5
1 1
probability that both the cars parked on two sides of his *THHHH * =   × 1 =
2 32
(n − m)(n − m − 1) 5
car, have left is , then A + B is 1 1
(n − A)(n − B) * * THHHH =   × 1 =
2 32
54. A special die is so constructed that the probabilities 5
of throwing 1, 2, 3, 4, 5 and 6 are (1 – k)/6, (1 + 2k)/6, \ The required probability = 32 .
(1 – k)/6, (1 + k)/6, (1 – 2k)/6 and (1 + k)/6 respectively.
 E   E   E   E 
If two such dice are thrown and the probability of 3. (c) : P  +P +P +P ,
 no heads   1 head   2 heads   3 heads 
1 2
getting a sum equal to 9 lies in  ,  . Then find the
9 9  E   E   E   E 
P
  + P   + P   + P  ,
number of integral solutions of k. no heads 1 head 2 heads 3 heads 
where E : getting no two consecutive heads.
55. Two distinct numbers are chosen at random from
1 5 6 1 13
the set {1, 2, ……. 3n}. The probability that x2 – y2 is = + + + =
pn + q 32 32 32 32 32
divisible by 3 is , then p + q + r = 4. (a) : P(getting a number bigger than 3) = 1/2
r (3n − 1)
P(getting 5 in throw) = 1/6
56. 3 numbers are chosen at random without
Let E : getting 5 in last throw when he gets a number
replacement from {1, 2, 3, ...., 14}.
bigger than 3.
Let A = {min of chosen number is 5}
1 1 1 1 1 1
B = {max of chosen number is 11} \ P (E ) = + ⋅ + ⋅ ⋅ + .... ∞
K +11 6 2 6 2 2 6
P ( A ∪ B) = , then K = _____ 1 1 1
91 = × =
6 1 3
57. In a bag there are 15 balls of either red or green 1−
colour. Let Gk be the event that it contains exactly k 2
5. (a) : Selected year may be a non leap year with a
green balls and its probability is proportional to k2.
probability 3/4.
Now a ball is drawn at random. Let P(A) be the
Selected year may leap year with a probability 1/4.
probability that the ball drawn is green. If P(A) = p/q in
Let E be an event that randomly selected year contains
lowest form, then q – p is _____.
53 Mondays
solutions 2
3 1 1 2 5 ⇒  leap year  2
 ax + bx 
2/ x
P (E ) = × + × = P  = 28 =
4 7 4 7 28  E  5 5
1. (c) : lim   =6
x →0  2  28
6. (c) : P(A ∩ B) = P(A) · P(B)
 a x −1   b x −1 
lim   +   P(A ∪ B) = P(A) + P(B) – P(A) P(B)
 x  = 6 = eloga + logb = 6
= e x →0 x 0.8 = 0.3 + P(B) (1 – 0.3)
⇒ ab = 6 5
(a, b) = (1, 6), (6, 1), (2, 3), (3, 2) 0.5 = P (B)(0.7) ⇒ P (B) =
7

52 mathematics today | december ‘15


7. (c) : The total numbers of arrangements is 1 1 1
11! 11! In case III = ⋅ ⋅1 =
= 2 4 8
2! 2! 2! 8 1 3 1 9
The required probability = + + =
The number of arrangements in which C, E, H, I, S 16 32 8 32
appear in that order required
2 1 4 1 7
11 6 ! 11! 11. (a) : P (E ) = = , P (F ) = = , P (G) =
=  = 8 4 8 2 8
 5  2 ! 2 ! 2 ! 8 ⋅ 5!
11! 11! 1 1 1 3 1
\ Required probability = . ÷ = = P ( E ∩ F ) = , P (F ∩ G ) = , P ( E ∩ G ) =
8 5 ! 8 ! 5 ! 120 8 8 8
12. (d) : Since a + b + c = 6, the possible digit selections
8. (c) : G = Original signal was green. A = A receive
are (1, 2, 3), (1, 1, 4), (2, 2, 2), (0, 1, 5), (0, 2, 4),
correct signal, B = B receive correct signal and E =
(0, 3, 3), (0, 0, 6)
signal received by B is green.
The required number of ways
 G  P (G ∩ E )
P = = 6 + 3 + 1 + 4 + 4 + 2 + 1 = 21
E P (E) 21 7
P (E ) = P (GAB) + P (GAB ) + P (GAB ) + P (GAB) Required probability = =
9 × 10 × 10 300
4 3 3 4 1 1 1 3 1 1 1 3 46
= ⋅ ⋅ + ⋅ ⋅ + ⋅ ⋅ + ⋅ ⋅ = 13. (a) : Number of triplets (3r, 3r + 1, 3r + 2) (0 ≤ r ≤ n)
5 4 4 5 4 4 5 4 4 5 4 4 80 is n – 1
40 Number of triplets (3r, 3r + 2, 3r + 4) (0 ≤ r ≤ n) is n – 3
P (G ∩ E ) = P (GAB) + P (GAB ) =
80 ----------------------------------------------------------
9. (b) : The total number of ways to put four identical  n−1 
r+
 r 2 r +n−1 
oranges and six distinct apples into five distinct boxes is Number of triplets  3 , 3 , 3  (n odd) is 2
(5 + 4 – 1C4)·56 = 70 × 56  r+
n 
To satisfy the criteria that each box contains two object And number of triplets  3 , 3 2 , 3r +n  (n even) is 1.
 r
we make three cases \ If n is odd, the number of favourable outcomes
(1) Two oranges in each of the two boxes and no orange
in the other three boxes n2 − 1
= (n – 1) + (n – 3) + ... + 4 + 2 =
6! 4
Number of ways = 5C2 × = 900 And if n is even, the number of favourable outcomes
(2 !)3
(2) Two oranges in one box, one orange in each of the n n n2
= (n − 1) + (n − 3) + ... + 3 + 1 = × =
6! 2 2 4
two other boxes = 5 × (4 C2 ) × = 5400
(2 !)2 (n2 − 1) / 43
Probability = =if n is odd
(n+1) 2n
(3) One orange in each of the four boxes C3
6! n2 / 4 3n
= 5 ⋅ = 1800 = = if n is even.
(n+1)
2! C3 2(n2 − 1)
The total number of ways that each box contains two
objects = 900 + 5400 + 1800 = 8100 14. (b) : A = {HH, HTHH, HTHTHH, ....}
8100 162 and B = {TT, THTT, THTHTT, ...}
\ Required probability = = 1 1 1 1
6 21875 P ( A) = + + ... = and P (B) =
70 × 5 2 4
2 2 3 3
10. (d) : In order that all balls are red after 3 1 1 2
replacements, two of the three balls selected must have \ Required probability = + =
3 3 3
green. There could be three cases.
15. (b) : No. of determinants formed = n(S) = 24 = 16
I : Red, Green, Green; II : Green, red, Green.
III : Green,Green,Red. 1 0 1 0 1 1
Let E : the determinants , , are positive
The probabilities is 0 1 1 1 0 1
1 1 1 1 1 3 1 3 3
In case I = ⋅ ⋅ = , In case II = ⋅ ⋅ = ⇒ n(E) = 3 \ P (E ) =
2 2 4 16 2 4 4 32 16

mathematics today | december ‘15 53


16. (a) : G1 = 1 4 7 ---- 3n – 2 20. (c) : Let n be the required number.
G2 = 2 5 8 ---- 3n – 1 The probability that ‘n’ missiles miss the target is
G3 = 3 6 9 ---- 3n (0.7)n. We require 1 – (0.7)n ≥ 0.8
We have n(S) = 3nC2, for n(E) both numbers can be i.e., (0.7)n ≤ 0.2. The least value of ‘n’ satisfying the
selected from same group or one number from G1 and inequality is 5.
one from G2 3 ⋅ nC2 + n2 21. (a) : Let S be the sample space of the composite
n n
\ n(E) = 3 ⋅ C2 + C1 × C1 n \ P ( E ) = experiment of drawing 6 balls in the first draw and
3n
C2
then 4 in second draw then |S| = 10C6 × 10C4
5
17. (a) : A’s chance of winning in a throw = , 10
C6 × 6C3 × 4C1
1 36 \ Required probability = 10
B’s chance of winning in a throw = C6 × 10C4
6
31 80 × 24 8
A’s chance of losing in a throw = , = =
36 10 × 9 × 8 × 7 21
5 22. (d) : Required probability
B’s chance of losing in a throw = 5
6 C2 × 6 × 6 × 6 10 × 63
A’s chance of winning the game = =
7×7×7×7×7 75
5 31 5 5 31 5 31 5 5 2
23. (c) : (a + 4) + 5a – 64 < 0 ⇒ –16 < a < 3
= + × × + × × × × + ....
36 36 6 36 36 6 36 6 36 18 1
\ Required probability = =
5  155  155   30
2
36 2
= 1 + +  + .... = 24. (b) : No. of trials n = 21
36  216  216   61
Let success of solving the problem be p.
18. (d) : The sequence of consecutive heads may start
\ p = P ( A ∪ B ∪ C ) = 1 − P ( A)P (B )P (C )
with 1st toss or 2nd toss or 3rd toss .... or at 6th toss.
In any case, if it starts with rth throw, the first (r – 2) 3 4 5 1 1
=1− × × = , q =
throws may be head or tail but (r – 1)st throw must 4 5 6 2 2
be tail, after which again a head or tail can show up: Find P (X ≥ 11)
HHH------H × × × × × 25. (b) : Let ‘A’ be the event of guessing the correct
7 times answer. ‘B’ be the event of copying the correct answer.
or × × × ----- × T HHH ------ H × × ----- ‘C’ be the event of knowing the correct answer.
(r – 2) ‘D’ be the event that his answer is correct
(r – 1)st 1 1 1 
1 1 1 1 1 1 1 P ( A) = , P (B) = , P (C ) = 
\ Probability = + ⋅ + ⋅ + .... + ⋅ 3 6 2 
7 2 7 2 7 7 
2 
2 2 22 D 1 D 1 D 
P   = , P   = , P   =1
5 times  B  8  A  4  C  
1  5 7 D
= 1 +  = P (C ) ⋅ P  
27  2  28 C  C  24
P = =
19. (c) : The required probability = 1 – probability of  D  P (D ∩ A) + P (D ∩ B) + P (D ∩ C ) 29
n(E )
each receiving atleast one = 1− . 26. (b, c, d) : P(A) = 1/4, P(B | A) = 5/7,
n(S) P (B | A) = 6 / 7 , P (B) = P ( A)P (B | A) + P ( A)P (B | A)
Now, the number of integral solutions of = (1/4 × 5/7) + (3/4 × 6/7) = 23/28
x1 + x2 + x3 + x4 + x5 + x6 = 10
1 5
Such that x1 ≥ 1, x2 ≥ 1, ..., x6 ≥ 1 gives n(E) and the ×
P ( A ∩ B) 4 7 5
number of integral solutions of x1 + x2 + x3 + x4 + x5 + P ( A | B) = = =
x6 = 10 such that x1 ≥ 0, x2 ≥ 0, ..., x6 ≥ 0 gives n(S) P ( B) 1 5 3 6 23
× + ×
10 −1 4 7 4 7
C
\ The required probability = 1 − 10+6−1 6−1 1 2
C6−1 ×
P(A ∩ B) 4 7 2
9 P(A | B) = = =
C5 137 P (B ) 23 5
= 1− = 1−
15 28
C5 143

54 mathematics today | december ‘15


27. (a, b, c, d) : P(A) = 0.7; P(B) = 0.4 30. (b, c) : Since A and B are independent events,
P(A – B) = P(A) – P(AB) 2
⇒ P(AB) = 0.2 ⇒ P(A + B) = 0.9 = P(A′ ∩ B) = P(A′) P(B)
15
⇒ P(B – A) = 0.2 = [1 – P(A)] P(B) ...(1)
1
⇒ P ( A ∪ B ) = 1 − P ( AB) = 0.8 and = P(A ∩ B′) = P(A) P(B′)
6
P ( A ∩ B) 1 = P(A)[1 – P(B)] ... (2)
⇒ P (B | A ∪ B ) = = Subtracting (1) from (2), we get
P(A ∪ B) 4
28. (a, b, d) : P(A) – P(B) = 1/30 or P(A) = P(B) + 1/30
Putting this value in (2), we get
urn red White blue
[P(B) + 1/30][1 – P(B)] = 1/6
marbles marbles marbles
⇒ 30[P(B)]2 – 29 P(B) + 4 = 0
A 5 3 8 ⇒ P(B) = 1/6, 4/5
B 3 5 0 31. (a, c, d) : The required probability
= prob. that M occurs and N does not occur or N
 2   5   1   3  10 6 1
P (E1 ) = P (R) =     +     = + = occurs and M does not occur.
 3   16   3   8  48 48 3
= P(M ∩ Nc) + P(Mc ∩ N) [This is (d)]
 2   3   1   5  6 10 1 = P(M) – P(M ∩ N) + P(N) – P(M ∩ N)
P (E2 ) = P (W ) =     +     = + =
 3   16   3   8  48 48 3 = P(M) + P(N) – 2P(M ∩ N) [This is (a)]
= 1 – P(Mc) + 1 – P(Nc) – 2[1 – P(M ∩ N)c]
2 8  1
P (E3 ) = P (B) =     = = 2P(Mc ∪ Nc) – P(Mc) – P(Nc)
 3   16  3
[By De-Morgan’s law]
(c) Let A : event that urn A is chosen = 2[P(Mc) + P(Nc) – P(Mc ∩ Nc] – P(Mc) – P(Nc)]
2 5  = P(Mc) + P(Nc) – 2P(Mc ∩ Nc) [This is (c)]
P ( A ∩ R)  3   16   10  5 32. (a, b, c, d) : Probability that atleast one blue ball
P ( A | R) = = =   (3) =
P (R) 1  48  8 is drawn
3 = 1 – Probability that all the balls drawn are red.
⇒ (c) is incorrect. 1 1 3 1 9
2 3  = 1 − ⋅ ⋅ = 1 − = = 0. 9
3 2 5 10 10
P ( A ∩ W )  3   16   6  3
(d) P ( A | W ) = = =   (3) = Probability that exactly one blue ball is drawn
P (W ) 1  48  8 1 1 2 2 1 2 1 2 2
3 = ⋅ ⋅ + ⋅ ⋅ + ⋅ ⋅ = 0. 2
31 3 3 2 5 3 4 5 3 4 5
P (face five | W ) =     = Probability that all the drawn balls are red given that
 8   4  32
⇒ (d) is correct. all the drawn balls are of the same colour
x 1
1 1
29. (a, b, c) : We have, P ( X = x ) ∝ (x + 1)   = 10 = = 0. 2

5 1 4 5
since, ∑ P ( X = x ) = 1 +
x =0
10 10
Probability that atleast one red ball is drawn
 
2 3
1 1 1
⇒ k 1 + 2   + 3   + 4   + ... ∞ = 1 2 3 4
5 5 5 = 1 −  ⋅ ⋅  = 0. 6
  3 4 5
16 33. (a, c) : The probability of exactly one of A and B
⇒ k=
25 0 to occur = P ( AB ) + P ( AB)
1 16
(a) P ( X = 0) = k(0 + 1)   = k = = P ( A)P (B ) + P ( A)P (B) = P ( A){1 − P (B)} + {1 − P ( A)}P (B)
5 25
112 = P(A) + P(B) – 2P(A)P(B)
(b) P(X ≤ 1) = P(X = 0) + P(X = 1) = = P(A) + P(B) – 2P(A ∩ B)
125
16 9 = 1 − P ( A) + 1 − P (B ) − 2{1 − P ( A)}{1 − P (B )}
(c) P(X ≥ 1) = 1 – P (X = 0) = 1 – k = 1 − =
25 25 = P ( A) + P (B ) − 2P ( A) ⋅ P (B )

mathematics today | december ‘15 55


34. (a, b, c, d) : P(A ∪ B) ≥ P(A), P(A ∪ B) ≥ P(B) (C) P ∩ Q contain 2 elements
3 2 n(E) = nC2(1)2 3n – 2
P ( A ∪ B) ≤ 1 ⇒ + − 1 ≤ P ( A ∩ B) (D) n(P) = n(Q)
5 3
n
⇒ P ( A ∩ B) ≥
4 C0 nC0 + nC1 nC1 + ... + nCn nCn
15 ⇒ P (E ) =
2n ⋅ 2n
But P(A ∩ B) ≤ P(A), P(A ∩ B) ≤ P(B)
43. a → r; b → p; C → s; d → q
3 4 1
P ( A ∩ B ) = P ( A) − P ( A ∩ B) ≤ − ≤ 20 18 16 14 224
5 15 3 (A) P (no pair) = ⋅ ⋅ ⋅ =
20 19 18 17 323
35. (c) 36. (a) 37. (a)
224 99
38. (a) : 1 4 7 10 13 16 (B) P (atleast one pair) = 1 − =
323 323
2 5 8 11 14 17 10
C2 3
3 6 9 12 15 18 (C) P (exactly two pairs) = =
20 323
C4
6 6 2
C2 + ( C1 ) 1
Probability = =  224 3  96
18 (D) P (exactly one pair) = 1 −  + =
C2 3  323 323  323
39. (d) : 1 3 5 7 9 11 13 15 17 44. a → s; b → s; C → r; d → q
2 4 6 8 10 12 14 16 18 (A) The number appearing on upper face of any dice
2(9 C2 ) 3 can be 3, 4, 5 or 6 i.e. maximum 4 cases.
Probability = 18 = P(m = 3) = P(m ≥ 3) – P(m ≥ 4)
C3 34
5 5
(18 −3+1) 45 − 35 2 1
C3 35 = =  − 
40. (a) : Probability = 18
= 65 3 2
C3 51
(B) The number appearing on upper face of any dice
41. a → r; b → p: C → q; d → s can be 1, 2, 3 or 4 i.e. maximum 4 cases.
(A) The required event will occur if last digit in all P(n = 4) = P(n ≤ 4) – P(n ≤ 3)
chosen numbers is 1, 5, 7 or 9. 5 5
n 45 − 35 2 1
4 = =  − 
Required probability =   65 3 2
 10 
(C) P(2 ≤ m ≤ 4) = P(m ≥ 2) – P(m ≥ 5)
(B) The required probability is equal to the probability
5 5
that last digit is 2, 4, 6, 8 55 − 25 5 1
8n − 4n = =  − 
P (1, 2, 3, 4, 5, 6, 7, 8, 9) − P (1, 3, 7, 9) = 65 6 3
10n
n
5 −4 n (D) P(m = 2, n = 5) = P(2, 3, 4 or 5) – P(2, 3 or 4)
(C) P (1, 3, 5, 7, 9) − P (1, 3, 7, 9) = – P(3, 4 or 5) + P(3 or 4)
10n 5 4 5
45 − 2 × 35 + 25 2 1 1
(10n − 8n ) − (5n − 4n ) = =  −  + 
(D) P(0, 5) – P(5) = 65 3 2 3
10n
10n − 8n − 5n + 4n 45. (2) : n(X) = k + 1
= No. of ways to construct A = 2k + 1
10n No. of ways to construct B = 2k + 1
42. a → q; b → p; C → s; d → r \ Total ways to construct A and B = 2k + 1 × 2k + 1
If xi ∈ A then xi ∈ P, xi ∈ Q Favourable ways to construct A = 2k + 1
or xi ∉ P, xi ∉ Q or xi ∉ P, xi ∈ Q or xi ∈ P, xi ∉ Q Favourable ways to construct B such that B = Ac is 1
(A) P ∩ Q = f ⇒ xi ∈ P, xi ∉ Q \ Favourable ways = 2k + 1 × 1
or xi ∉ P, xi ∉ Q or xi ∉ P, xi ∈ Q or n(E) = 3n, n(S) = 4n
2k +1 1
(B) P ∩ Q is a singleton set. Required Probability = =
k +1 2 k +1
⇒ xi ∈ P, xi ∈ Q (2 ) 2
n(E) = nC1 (1)3n – 1, n(S) = 4n ⇒ m–1=k+1⇒m–k=2

56 mathematics today | december ‘15


46. (5) : The no. of ways of drawing 5 balls = 10C5 Total number of ways = 3 + 6 + 3 + 3 = 15
For each set of 7 balls of the second draw, 3 must be 15 5
common to the set of 5 balls of the draw, i.e., 2 other \ Probability = =
126 42
balls can be drawn in 3C2 ways thus, for each set of 7
49. (3) : x3 + y3 is divisible by 3 ⇒ x + y is divisible by 3
balls of the second draw, there are 7C3 × 3C2 ways of
⇒ x, y are multiples of 3 or one leaves remainder 1
making the first draw so that there are 3 balls common.
and the other 2 when divided by 3.
Hence, the probability of having three balls in common 3, 6, 9, ... , 30 are multiple of 3; 1, 4, 7, ..., 28 leave
7
C3 × 3C2 5 . remainder 1; 2, 5, 8, ..., 29 leave remainder 2
= 10
=
C5 12 10  10  10 
47. (8) : Let the sides be a, b, c  1   1  +  2 
145 1
a = b = 1, c = 1 \ Probability = = =
 30  15 × 29 3
a = b = 2, c = 1, 2, 3
 2 
a = b = 3, c = 1, 2, 3, 4, 5
a = b = 4, c = 1, 2, 3, 4, 5, 6 50. (9) : Let E1 = the toss result is a head
a = b = 5, c = 1, 2, 3, 4, 5, 6 E2 = the toss result is a tail
a = b = 6, c = 1, 2, 3, 4, 5, 6 A = noted number is 7 or 8
The number of these triangles is \ P(A) = P(E1)·P(A | E1) + P(E2)·P(A | E2)
1 + 3 + 5 + (3 × 6) = 27 1 11 1 2 193 P 193
27 1 = × + × = \ =
Probability = = 2 36 2 11 792 792 792
63 8 \ P = 193
48. (5) : The total number of outcomes: 51. (2) : Divisors of N = 2n – 1(2n – 1) are
6 1, 2, 22 , ..., 2n – 1, 2n – 1, 2(2n – 1), 22(2n – 1), ...
aaaa appear in   = 6 ways
1 ..., 2n – 1(2n – 1)
1 1 1 1 1 1
6 1+ + + ... + = 1 + + .... + +
aaab appear in 2   = 30 ways a1 a2 ak 2 n −1 n
2 2 −1
2
1 1
6 + + ..... +
n n−1
aabb appear in   = 15 ways 2(2 − 1) 2 (2n − 1)
2  1 1  1  1 1 
= 1 + + ... + + 1 + 2 + ... + n−1 
6  2 n−1  n
2 2 −1 2
aabc appear in 3   = 60 ways
3  1 1  1 
= 1 + + ... +  1 + 
6  2 2n−1   2n − 1 
abcd appear in   = 15 ways   1 n 
4 1 1 −     n 
 2  2 2n
Total number of ways = 6 + 30 + 15 + 60 + 15 = 126 =   = =2
1  2n − 1  2n−1
The number of ways of primes appearing 1−
2
3
aaaa appear in   = 3 ways 52. (4) : Total ways of choosing m and n is n(S)
1 = 100 × 100
3 Now, 71 = 7 = 5k + 2, 72 = 49 = 5k + 4,
aaab appear in 2   = 6 ways 73 = 343 = 5k + 3, 74 = 2401 = 5k + 1
2
The same sequence will repeat for next four powers
3 75 = 5k + 2, 76 = 5k + 4, 77 = 5k + 3, 78 = 5k + 1
aabb appear in   = 3 ways
2 \ 71, 75, 79, ..., 797 are of the type 5k + 2
72 , 76, 710, ...., 798 are of the type 5k + 4
3 73, 77, 711, ...., 799 are of the type 5k + 3
aabc appear in 3   = 3 ways number
3 & 74, 78, 712 , ..., 7100 are of the type 5k + 1

58 mathematics today | december ‘15


There will be only four favourable combinations in (x2 – y2) is divisible by 3. If both x, y should come from
which 7m + 7n will be divisible by 5. A0 or A1 or A2 or one is from A1 and other from A2.
7m : 5k + 2, 5k + 4, 5k + 3, 5k + 1 n
7n : 5k + 3, 5k + 1, 5k + 2, 5k + 4 n(E ) = 3 nC2 + nC1 × nC1 = (5n − 3)
2
\ Number of favourable cases is 5n − 3
P (E ) =
n(E) = 25 × 25 + 25 × 25 + 25 × 25 + 25 × 25 3(3n − 1)
= 4 × 625 9 10
n(E ) 4 × 625 1 C2 9 C2 45
\ Required probability = = = 56. (8) : P ( A) = = , P ( B) = =
n(S) 100 × 100 4 14
C3 91 14
C3 364
5
53. (3) : Number of ways in which m – 1 remaining cars C1 5 19
P ( A ∩ B) = = , P ( A ∪ B) =
can take their places (excluding the car of a man) 14
C3 364 91
= n – 1Cm – 1
No. of ways in which remaining (m – 1) cars can take 57. (7) : P(Gk) ∝ k2 ⇒ P(Gk) = lk2
n
places keeping the two places on two sides of his car ∑ P (Gk ) = 1 (as these are mutually exclusive and
vacant = n – 3Cm – 1 k =0
n−3 exhaustive events)
C (n − m)(n − m − 1)
Probability = n−1 m−1 = n 6
Cm−1 (n − 1)(n − 2) ⇒ l ∑ k2 = 1 ⇒ l =
k =0 n(n + 1)(2n + 1)
⇒ A = 1, B = 2 ⇒ A + B = 3 n
54. (1) : Let E1, E2, E3 E4, E5 and E6 be the events of P ( A) = ∑ P (Gk ) P ( A / Gk )
occurrence of 1, 2, 3, 4, 5 and 6 on the dice respectively, k =0
and let E be the event
n k l n2 (n + 1)2 3(n + 1)
= ∑ lk 2 ⋅ = ⋅ =
1− k 1 + 2k 1− k k =0 n n 4 2(2n + 1)
\ P (E1 ) = ; P (E2 ) = ; P (E3 ) = Take n = 15.
6 6 6
nn
1+ k 1 − 2k 1+ k
P ( E4 ) = ; P (E5 ) = ; P (E6 ) =
6 6 6
1 2
and ≤ P (E ) ≤
9 9
Then, E ≡ {(3, 6), (6, 3), (4, 5), (5, 4)}
Hence,
P(E) = P(E3E6) + P(E6E3) + P(E4E5) + P(E5E4)
Since E1, E2, E3, E4, E5 and E6 are independent
\ P(E) = P(E3)P(E6) + P(E6)P(E3) + P(E4)P(E5) +
P(E5)P(E4)
= 2P(E3)P(E6) + 2P(E4)P(E5)
 1 − k   1 + k   1 + k   1 − 2k 
= 2 +2
 6   6   6   6 
1
= [2 − k − 3k 2 ]
18
1 2 1
Since, ≤ P (E ) ≤ \ − 3 ≤ k ≤ 0
9 9
\ Set of integral value of k = {0}
\ Number of integral solution of k is 1
55. (5) : n(S) = 3nC2
Let A0 = {3, 6, 9, ... 3n}, A1 = {1, 4, 7, ..., 3n – 2}
A2 = {5, 8, 11, ..., 3n – 1}

mathematics today | december ‘15 59


10 Best Proble
M th
rchives 10 Best Problems Prof. Shyam Bhu

math Archives, as the title itself suggests, is a collection of various challenging problems related to the topics of
Jee (main & Advanced) Syllabus. This section is basically aimed at providing an extra insight and knowledge to the candidates preparing
for Jee (main & Advanced). In every issue of mT, challenging problems are offered with detailed solution. The readers' comments and
suggestions regarding the problems and solutions offered are always welcome.

7. If f : R – {0} → R is a differentiable function such


d e sin x
1. Let ( f (x )) = , x > 0. xy x y
dx x that ∫ f (t ) dt = y ∫ f (t ) dt + x ∫ f (t ) dt , ∀ x , y ∈ R − {0},
2 1 1 1
4 2e sin x
If ∫ dx = f (k) − f (1), then one of the possible f (1) = 1, then find f (x).
1 x
8. Evaluate :
value of k is (sin q − cos q) dq
(a) –4 (b) 0 (c) 2 (d) 16 I=∫ .
2 2
(cos q + sin q) sin q ⋅ cos q + sin q ⋅ cos q
−1 −1
2. ∫ x − 3 (sin (ln x ) + cos (ln x )) dx , is equal to
π 9. If f is successively differentiable function, then
(a) (x − 3)3/2 + c (b) 0 prove that
3
(c) does not exist (d) none of these 1 1
f (x ) = f (0) + xf ′(0) + x 2 f ′′(0) + x 3 f ′′′(0)
2! 3!
dx
3. ∫ equals 1 x 3
+ ∫ f ′′′′(t )(x − t ) dt
(x − 1) x 2 − 1 3! 0

(a) −
x −1
+c (b)
x −1
+c x2 y2
10. Prove that in the ellipse = 1, the distance
+
x +1 x +1 a 2 b2
between the center and any normal does not exceed
x +1 x +1
(c) +c (d) − +c |a – b|. Find a point on the ellipse normal at which is at
x −1 x −1
a distance of |a – b| from centre.
4. The degree of the differential equation of the family
soLUtioNs
of lines given by y = cx + c is
(a) 1 (b) 2 d e sin x
1. (d) : Given ( f (x )) = ,x > 0
(c) 4 (d) not defined dx x
5. The order of the differential equation of the family 2
4 2e sin x
of rectangular hyperbolas having fixed centre, is Let I = ∫ dx
1 x
(a) 1 (b) 2 (c) 3 (d) 4
Put z = x2 ⇒ dz = 2xdx
6. Let f be a differentiable function such that 2
x 4 2 xe sin x 16 e sin z
2 −t \ I=∫ dx = ∫ dz
f (x ) = x + ∫ e f (x − t ) dt , then prove that
0 1 x2 1 z
= [ f (z )] 1 = f (16) − f (1)
16
⇒ f (k) = f(16)
x3
f (x ) = + x2 .
3 \ One possible value of k is 16.
By : Prof. Shyam Bhushan, Director, Narayana IIT Academy, Jamshedpur. Mob. : 09334870021
60 mathematics today | december ‘15
2. (c) : For x > e, lnx > 1 Then put x = 0 in (4), we get f (0) = C ⇒ C = 0.
⇒ sin–1(lnx) + cos–1(lnx) is not defined. x3
Hence, the given integral does not exist. \ f (x ) = + x2
3
dx
3. (d) : Let I = ∫ 7. Differentiating partially w.r.t. y, we get
1/2
 x +1
(x − 1)2 .  x
 x − 1  f (xy ) ⋅ x = ∫ f (t ) dt + xf ( y )
1
2
x +1 1  1  Putting y = 1, we get
Put u = ⇒ − du =  dx
x −1 2  x − 1  x x
xf (x ) = ∫ f (t ) dt + xf (1) = ∫ f (t ) dt + x , as f (1) = 1.
1 du x +1 1 1
\ I=− ∫ = − u +c = − +c
2 u x −1 Differentiating w.r.t. x, we get
dy f (x) + x f ′(x) = f (x) + 1
4. (b) : We have, y = cx + c ⇒ =c 1
dx ⇒ f ′(x ) = ⇒ f (x ) = ln | x | + C
2
x
dy dy dy  dy  Now f(1) = 1 ⇒ C = 1 ⇒ f (x) = 1 + ln|x|.
⇒ y=x + ⇒ = y−x 
dx dx dx  dx  8. On multiplying the numerator and denominator
⇒ degree is 2. by (sin q + cos q), we get
5. (b) : We need one parameter to fix the direction (sin q + cos q)(sin q − cos q) dq
of the axes and one parameter for fixing the length of I=∫
transverse axis of the hyperbola (note that eccentricity (cos q + sin q)2 sin q ⋅ cos q + sin2 q ⋅ cos2 q
is 2 ). Hence, required order is 2. −(cos2 q − sin2 q) dq
x \ I=∫
6. Given, f (x ) = x 2 + ∫ e −t f (x − t ) dt sin 2q sin2 2q
0
(1 + sin 2q) +
2 4
x
\ f (x ) = x 2 + ∫ e −( x −t ) f (t ) dt − cos 2q dq
=∫
0 1 1
sin 2q + sin2 2q
(1 + sin 2q)
 b b  2 4
 ∫ f (x ) dx = ∫ f (a + b − x ) dx 
 a a  Put 1 + sin2q = x ⇒ 2cos2q dq = dx
x dx
or f (x ) = x 2 + e − x ∫ et f (t ) dt … (1) −
0 \ I=∫ 2
Differentiating (1) w.r.t. x, we get (x − 1) (x − 1)2
x⋅ +
x 2 4
f ′(x ) = 2 x + e − x [1 ⋅ e x f (x ) − 0] + e − x (−1) ∫ et f (t ) dt
0 −dx −dx
=∫ =∫
x 2
\ f ′(x ) = 2 x + f (x ) − e − x ∫ et f (t ) dt …(2) x 2(x − 1) + (x − 2 x + 1) x x2 − 1
0 = cosec–1x + c
Adding (1) and (2), we get = cosec–1(1 + sin2q) + c
f ′(x) + f (x) = x2 + 2x + f (x) x x
∫ f ′′′′(t )(x − t ) dt = (x − t ) f ′′′(t )
3 3
\ f ′(x) = x2 + 2x …(3) 9.
0 0
x3
Integrating, we get f (x ) = + x2 + C x
3 +3 ∫ (x − t )2 f ′′′(t ) dt
0
x3
\ f (x ) = + x2 + C …(4)
 x x 
3 = − x 3 f ′′′(0) + 3 (x − t )2 f ′′(t ) + 2 ∫ (x − t ) f ′′(t ) dt 
Now, putting x = 0 in (1), we get f (0) = 0  0 0 

mathematics today | december ‘15 61


x For fixed a and b, D is maximum when
= − x 3 f ′′′(0) + 3[− x 2 f ′′(0)] + 6 ∫ (x − t ) f ′′(t ) dt S = b2 cosec2q + a2 sec2q is minimum. This happens
0
when
 x x  dS
= − x 3 f ′′′(0) − 3x 2 f ′′(0) + 6 (x − t ) f ′(t ) + ∫ f ′(t ) dt  = −2b2cosec q(cosec q cot q) + 2a2 sec q(sec q tan q)
0
 0  dq
x
2b2 cos q 2a2 sin q 2 (b2 cos 4 q − a2 sin 4 q)
= − x 3 f ′′′(0) − 3x 2 f ′′(0) + 6[− x f ′(0)] + 6 ∫ f ′(t ) dt =− + =− =0
0 sin3 q cos3 q sin3 q cos3 q
= − x 3 f ′′′(0) − 3x 2 f ′′(0) − 6 xf ′(0) + 6 [ f (t )] 0
x i.e., when tan2q = b/a.
dS
x Differentiating , we get
⇒ ∫ f ′′′′(t )(x − t )3 dt = − x 3 f ′′′(0) − 3x 2 f ′′(0) dq
0
d2S 2b2 sin q 6b2 cos2 q 2a2 cos q 6a2 sin2 q
− 6 xf ′(0) + 6 [ f (x ) − f (0)] = + + +
dq2 sin3 q sin 4 q cos3 q cos 4 q
1 x
\ f (x ) − f (0) = [ ∫ f ′′′′(t )(x − t )3 dt + x 3 f ′′′(0) 2b2 6b2 cos2 q 2a2 6a2 sin2 q
6 0 = + + + > 0, ∀q
sin2 q sin 4 q cos2 q cos 4 q
+3x 2 f ′′(0) + 6 x f ′(0)]
Therefore tan2q = b/a corresponds to the minimum
1 2 1 value of S given by
\ f (x ) = f (0) + xf ′(0) + x f ′′(0) + x 3 f ′′′(0)
2! 3!
Smin = b2 (1 + cot 2 q) + a2 (1 + tan2 q) b
tan2 q=
1x a
+ ∫ f ′′′′(t )(x − t )3 dt
3! 0
 a  b
= b2 1 +  + a2 1 +  = (a + b)2
10. Let P(a cosq, b sinq) represents any point on the  b  a
given ellipse and the slope of the tangent at this point is Hence, the maximum value of the required distance
given by is
2x  2 y  dy dy b2 x b cot q b2 − a 2
+  =0⇒ =− =− , at P . Dmax = = |b − a |
2  2  dx dx 2 a
a b a y a +b
Therefore, the slope of the normal at the point P is Hence D ≤ |b – a|.
(a tanq)/b, so that the equation of the normal is
b b
a tan q Equality occurs when tan2 q = ⇒ q = tan −1
y − b sin q = (x − a cos q) a a
b
b a
 a tan q   a2 sin q  ⇒ sin q = , cos q =
⇒ − x + y +  −b sin q + =0 …(1) a +b a +b
 b   b 
 a b 
The distance from (0, 0) to the line (1) is ⇒ P ≡ a ,b
 a +b a + b 
a2 sin q nn
b sin q − (b2 − a2 )sin q
D= b =
a2 tan2 q 2 b2 + a2 tan2 q
+1
b2

b2 − a 2
=
b2 cosec2 q + a2 sec2 q

62 mathematics today | december ‘15


VectoR aLGeBRa
VECTOR DiRECTiOn RaTiOs
z Vector quantities are those which have both • Any three numbers a, b, c proportional to the
magnitude and direction. direction cosines l, m, n respectively of a line, are

z A vector, denoted by PQ , is determined by two called direction ratios of the line.
points P, Q such that the magnitude of the vector is l m n
the length of the straight line PQ and its direction \ = =
Note : a b c
is that from P to Q. The point P is called the initial
(i) The direction cosines of a line are
 and Q is called the terminal point of vector
point
a b
PQ . l=± , m=± ,

z The magnitude of a vector a is the positive number a 2 + b2 + c 2 a 2 + b2 + c 2
which is the measure of its length and is denoted c
 n=± ,
by | a | .
a + b2 + c 2
2
Position Vector where a, b, c are direction ratios.
z If P be a point in (ii) l2 + m2 + n2 = 1
a space, having co-
TypEs Of VECTORs
ordinates (x, y, z)
(i) Zero or Null vector : A vector whose initial and
with respect to the or-
terminal points are coincident is called the zero
igin O(0, 0, 0). Then
 or the null vector. 
the magnitude of OP The null vector is denoted by 0 .
is given by (ii) Unit Vector : A vector whose modulus is unity,

OP = x 2 + y 2 + z 2 . is called a unit vector. The unit vector in the direction

of a vector a is denoted by a^ , read as ‘a cap’. Thus,
DiRECTiOn COsinEs | a^ | = 1.
 (iii) Like and Unlike Vectors : Vectors are said to be
z If a vector a makes
angles, a, b, g with like when they have the same sense of direction
the positive direction and unlike when they have opposite direction.
of X, Y and Z axes (iv) Collinear or Parallel Vectors : Vectors having the same
or parallel supports are called collinear vectors.
r e s p e c t i v e l y, t h e n
(v) Co-initial Vectors : Vectors having the same initial
cos a, cos b, cos g are
point are called co-initial vectors. 
called direction cosines 
 (vi) Equal vectors: Two vectors a and b are said to
of vector a and are
be equal, if they have the same magnitude and
usually denoted by
direction irrespective of the positions of their
l, m, n.  
initial points and written as a = b .

mathematics today | december ‘15 63


(vii) Negative of a Vector : The vector which has Properties of Vector Addition
   
the same magnitude as the vector a but For any three vectors, a , b , c , we have the following
opposite direction is called the negative of properties :
       
a andis denoted by – a . Thus, if PQ = a , Commutative law a + b = b + a

then QP = – a .      
Associative law a + (b + c ) = (a + b ) + c
(viii) Free Vectors : Vectors whose initial point is not     
Additive identity a + 0 = a = 0 + a
specified are called free vectors.   
Additive inverse a + (−a ) = 0
  
aDDiTiOn Of VECTORs Multiplication by m (a ) = (a ) m = ma ,
scalar   
Triangle law: If two m (na ) = n (ma ) = (mn) a ,
z   
(m + n)a = ma + na ,
vectors are represen-  
m(a + b ) = ma + mb ,
ted in magnitude and
where m and n are any two scalars.
direction by the two
sides of a triangle COmpOnEnTs Of a VECTOR
^^ ^
taken in order, then • If P(x, y, z) is a point in space and i , j, k are unit
t heir sum c an b e vectors along X, Y and Z-axes respectively, then
represented in mag- the vector components of OP along X, Y and
^ ^ ^
nitude and direction Z-axes respectively are xi , y j and z k . Also,
 x, y
by the third side of the triangle taken in reverse and z are called scalar components of OP .
 ^ ^ ^
order. OP = x i + y j + z k

Thus in the given figure, ⇒ | OP | = x 2 + y 2 + z 2
    ^ ^ ^  ^ ^
BC + CA = BA F o r a n y t w o v e c t o r s a = a1 i + a2 j + a3 k and b = b1 i + b2 j +
     ^ ^ ^ ^ ^ ^
⇒ a +b = a +b a = a1 i + a2 j + a3 k and b = b1 i + b2 j + b3 k , we define
        ^ ^ ^
(a) a + b = (a1 + b1 ) i + (a2 + b2 ) j + (a3 + b3 ) k
Similarly AB + BC = AC , CA + AB = CB   ^ ^ ^
(b) a − b = (a1 − b1 ) i + (a2 − b2 ) j + (a3 − b3 ) k
z Parallelogram law of
 ^ ^ ^
addition : If two vectors (c) ma = (ma1 ) i + (ma2 ) j + (ma3 ) k , where m is
are represented in magni- a scalar
 
tude and direction by (d) a = b ⇔ a1 = b1 , a2 = b2 and a3 = b3
two adjacent sides of VECTOR JOining TwO pOinTs
a parallelogram, then If P ≡ (x1, y1, z1) and Q ≡ (x2, y2, z2) be any two points,
their sum or resultant then the vector joining P and Q is given by
   ^ ^ ^
will be represented in PQ = OQ − OP = (x2 − x1 ) i + ( y2 − y1 ) j + (z2 − z1 ) k
magnitude and d i re c t i on by t h e diagonal of 
Also, PQ = | PQ | = (x2 − x1 )2 + ( y2 − y1 )2 + (z2 − z1 )2
the parallelogram passing through the point of
intersection of the two adjacent sides. sECTiOn fORmula
    Internal Division
Thus, if a and b are represented by AB and AD
  z Let A and B be any
respectively, then a + b will be represented
 two points with posi-
by AC. 
t ion vectors a and b
Multiplication of a Vector by a Scalar respectively, and let C
 be a point dividing AB
Let a be any vector and m be any scalar. Then the inter-nally in the ratio

product m a is defined as a vector whose magnitude m : n. Then the position

is | m | times that of a and the direction is (i) same vector of C is given by

as that of a if m is positive and (ii) opposite to that  m b + n a
 OC =
of a if m is negative. m+n
64 mathematics today | december ‘15
External Division ^ ^
(xii) If a and b are unit vectors, then
Let A and
 B be any two points with position vectors ^
| a^ | = 1, | b | = 1 \ a^⋅ ^b = 1 ⋅1 ⋅ cos q = cos q
z

a and b respectively and let C be a point dividing  ^ ^ ^
AB externally in the ratio m : n. Then, the position (xiii) If a = a1 i + a2 j + a3 k
 ^ ^ ^
vector of C is given by and b = b1 i + b2 j + b3 k
  
 mb − na Then, a ⋅ b = a1b1 + a2b2 + a3b3
OC =  ^ ^ ^ 
m−n (xiv) If a = a1 i + a2 j + a3 k , b = b1 ^i + b2 ^j + b3 k^ and q
 
be the angle between two vectors a and b then

 a ⋅ b 
–1
q = cos 
 ab 
 
–1 
a1b1 + a2b2 + a3b3
\ q = cos 
 a2 + a2 + a2 b12 + b22 + b32 
1 2 3

pROJECTiOn Of a VECTOR On a linE


 
• Projection of a vector a on the other vector b , is
Mid Point formula
given by 
z If C is the mid point of AB, then it divides AB in the
 ˆ  b  1  
ratio 1 : 1. So, position vector of C is given by a ⋅ b or a ⋅    or  (a ⋅ b )
    b  b
1 ⋅ a + 1⋅ b a + b 
= • If q = 0, then the projection vector of AB will be
1+1 2 
ABitself and ifq = p, then the projection vector
DOT OR sCalaR pRODuCT Of TwO VECTORs 
  of AB will be BA.
Dot or scalar product of two vectors a and b inclined
 p 3p
at an angle q is ab cos q. It is denoted by a ⋅ b • If q = or q = , then the projection vector of
   
Thus, a ⋅ b = | a || b | cos q = ab cos q  2 2
AB will be zero vector.
Properties of Scalar Product
  VECTOR OR CROss pRODuCT Of
(i) a ⋅ b is a real number.
    TwO VECTORs
(ii) a ⋅ b = b ⋅ a (Commutative law)
      • The vector product of
(iii) (x a ) ⋅ b = x(a ⋅ b ) = a ⋅ (xb ), where x is a scalar 
t w o v e c t o r s a and b
(Associative law) 
is denoted by a × b and
       
read as a cross b. Vector
(iv) a ⋅ (b + c ) = a ⋅ b + a ⋅ c (Distributive law)
  product is also called
a ⋅b 
(v) cos q =   cross product. Thus, a × b
| a || b |  
    |sinqqn^n^, where n^
==||aa||||bb|sin
(vi) If a is perpendicular to b, then a ⋅ b = 0 is the unit vector along
   
(vii) a ⋅ b < 0, if and only if the angle between a × b.

a and b is obtuse. Properties of Cross Product
       
(viii) a ⋅ b = 0 ⇒ a ⇒ 0 or b = 0 or a ⊥ b . (i) a × b is a vector.
   
(ii) If a and b be any two non zero vectors, then a × b = 0
(ix) ^i ⋅ ^i = 1, ^j . ^j = 1, ^k . ^k = 1, i^. ^j = 0, ^j .^k = 0, i^.^k = 0    
⇔ a || b or angle between a and b i.e., q = 0 or p.
(x) If the two vectors have same direction, then p    
    (iii) If q = then | a × b | = | a || b |.
q = 0 \ cos q = 1 \ a ⋅ b = | a || b | 2
(xi) If the two vectors have opposite direction, (iv) For mutually perpendicular unit vectors iˆ , jˆ and kˆ ,
then q = p we have

    iˆ × iˆ = jˆ × jˆ = kˆ × kˆ = 0
\ cos q = –1 \ a ⋅ b = − | a || b |
iˆ × jˆ = kˆ , jˆ × kˆ = iˆ , kˆ × iˆ = jˆ

mathematics today | december ‘15 65


    
(v) The angle between two vectors a and b is given as (ix) If a , b and c are any three vectors and l be a
 
|a × b | scalar, then
sin q =  
| a || b |       
(a) a × (b + c ) = a × b + a × c
(vi) The vector product is not commutative, as [Distributivity of vector product over addition]
   
aa××bb==−−bb××aa..      
^ ^ ^
(b) l(a × b ) = (la ) × b = a × (lb )
⇒ j × i = − k , k^ × ^j = − ^i and ^i × k^ = − ^j  ^ ^ ^  ^ ^ ^
  (x) If a = a1 i + a2 j + a3 k and b = b1 i + b2 j + b3 k ,
(vii) If a and b represent the adjacent sides of a triangle ^ ^ ^
1    
i j k
then its area is given by | a × b |. then a × b = a a
  2 1 2 a3
(viii) If a and b represent the adjacent sides of a b1 b2 b3
 
parallelogram, then its area is given by | a × b |.
3D-GEoMETry
DIrECTIoN CoSINES oF A LINE PASSING So, the direction cosines of the line segment joining
ThroUGh Two PoINTS the points P(x1, y1, z1) and Q(x2, y2, z2) are
The direction ratios of the line segment joining x2 − x1 y2 − y1 z2 − z1
P(x1, y1, z1) and Q(x2, y2, z), is , , , where
PQ PQ PQ
x2 – x1, y2 – y1, z2 – z1 or x1 – x2, y1 – y2, z1 – z2
PQ = (x2 − x1 )2 + ( y2 − y1 )2 + (z2 − z1 )2

Equation Vector Form Cartesian form Figure


E qu at i on of r = a + lb
  x − x1 y − y1 z − z1 .
= =
a line passing a b c
through a given If the direction cosines <l, m, n> is
point and given, then equation of the line is
parallel to a x − x1 y − y1 z − z1
 = =
given vector b l m n


Equation of a r = a + l(b − a), l ∈R xx −− xx11 yy −− yy11 zz −− zz11
line passing == ==
xx22 −− xx11 yy22 −− yy11 zz22 −− zz11
through two
given points

Angle between Let the vector equations Let the cartesian equations of two
two lines of two lines L1 and L2 x − x1 y − y1 z − z1
  
  lines be = =
be r = a1 + l b1 and r = a2 + m b2 a1 b1 c1
      and
r = a1 + l b1 and r = a2 + m b2 x − x2 y − y2 z − z 2
  = =
b1 ⋅ b2 a2 b2 c2
Then, cos q =   Then,
| b1 || b2 |
a1a2 + b1b2 + c1c2
cos q =
a12 + b12 + c12 a22 + b22 + c22

66 mathematics today | december ‘15


  
Shortest distan L e t r = a1 + lb1 a n d The shortest distance (d) between the
-ce between   
r = a2 + lb2 represents lines, x − x1 = y − y1 = z − z1
two lines two skew lines, then a1 b1 c1
z Dist ance the magnitude of the and x − x2 y − y2 z − z2 is
= =
b e t w e e n shortest distance (d) is a2 b2 c2
    x2 − x1 y2 − y1 z2 − z1
two skew (b × b ) . (a − a )
1 2
  2 1 a1 b1 c1
lines | b1 × b2 | a2 b2 c2
d=
(b1c2 − b2c1 )2 + (c1a2 − c2a1 )2 + (a1b2 − a2b1 )2

• Dist ance   
b × (a2 − a1 )
between 
|b |
two para-
llel lines
  x2 − x1 y2 − y1 z2 − z1
• Condition (a − a ). (b × b ) = 0
2 1 1 2 a1 b1 c1 = 0
for Copla-
narity of a2 b2 c2
two lines
Equation of a
plane in
• N o r m a l r . n^ = d lx + my + nz = d
form
• Intercept x y z
form + + =1
a b c

E qu at i on of Let r be the position a(x – x1) + b(y – y1) + c(z – z1) = 0
a plane per- vector of any point
pendicular to P(x, y, z) in the plane,

a given vector then (r − a ) . n = 0
and p assing
through a given
point

Equation of a (r − a ) .[(b − a ) × (c − a )] = 0 x − x y − y1 z − z1


1
plane passing
x2 − x1 y2 − y1 z2 − z1 = 0
through three
x3 − x1 y3 − y1 z3 − z1
non-collinear
points

mathematics today | december ‘15 67


Equation of the r . (n1 + ln2 ) = d1 + ld2 (a1x + b1y + c1z – d1) + l(a2 x + b2 y +
plane passing c2 z – d2) = 0
t hroug h t he
inters e c t ion
of two given
planes

Angle between  
n1 . n2
two planes cos q =   a1a2 + b1b2 + c1c2
| n1 || n2 | cos q =
a12 + b12 + c12 a22 + b22 + c22
planes to be
(i) perpendicular if
cos q = a1a2 + b1b2 + c1c2 = 0
a b c
(ii) parallel if 1 = 1 = 1
a2 b2 c2
Distance of a  
| a .n − d |  ax1 + by1 + cz1 − d
point from a  , where n is
|n |
plane a 2 + b2 + c 2
normal to the plane.

Angle between  
−1 b .n
a line and a q = sin  
plane | b || n |

Very short answer type 5. Find the direction cosines of the line segment
1. Find the values of x and y so that the vectors joining the points A(7, –5, 9) and B(5, –3, 8).
^ ^ ^ ^
2 i + 3 j and x i + y j are equal.
short answer type
2. Find the scalar and vector components of the vector  ^ ^  ^ ^ ^  ^ ^ ^
6. Let a = 2 i + k , b = i + j + k and c = 4 i − 3 j + 7 k
with initial point A(2, 1) and terminal point B(–5, 7). 
be three vectors. Find a vector r which satisfies
3. Find the value of p for which the vectors      
 ^ ^ ^  ^ ^ ^ r × b = c × b and r ⋅ a = 0.
a = 3 i + 2 j + 9 k and b = i + p j + 3 k are perpendi-
cular. 7. The projection of a vector on the coordinate axes
are 6, –3, 2. Find its length and direction cosines.
4. Find the equation of a line passing through
^ ^ ^
a point (2, –1, 3) and parallel to the line 8. Find the distance of the point (2 i − j − 4 k ) from the
 ^ ^ plane r ⋅ (3 ^i − 4 ^j + 12 k^ ) = 9.
^ ^ ^
r = (i + j) + l(2 i + j − 2 k ).

68 mathematics today | december ‘15


9. Find a unit vector perpendicular to the plane ABC ^ ^
4. The given line is parallel to the vector 2 i + j − 2 k
^
where A, B, C are the points (3, –1, 2), (1, –1, –3), and the required line is parallel to the given line.
(4, –3, 1) respectively. ^ ^ ^
So, required line is parallel to the vector 2 i + j − 2 k .
10. Find the equation of a plane through the intersection Thus, the equation of the required line passing through
 ^ ^ ^  ^ ^ ^ (2, –1, 3) is
of the planes r ⋅ (i + 3 j − k ) = 5 and r ⋅ (2 i − j + k ) = 3
and passing through the point (2, 1, –2).  ^ ^ ^ ^ ^ ^
r = (2 i − j + 3 k) + m(2 i + j − 2 k)
11. Find the direction cosines of the perpendicular
5. Direction ratios of the line segment AB are
from the origin to the plane r ⋅ (6 ^i − 3 ^j − 2 k^ ) + 3 = 0.
(5 – 7), –3 – (–5), (8 –9), i.e., –2, 2, –1.
Long answer type \ its d.c.’s are
−2 2 −1
12. (i) Find the vector equation of the plane passing , ,
^ ^ ^ ^ ^ ^
through the points i + j − 2 k , 2 i − j + k and (−2)2 + 22 + (−1)2 (−2)2 + 22 + (−1)2 (−2)2 + 22 + (−1)2
^ ^ ^ −2 2 −1
i + 2 j + k. , ,
 ^ ^ ^(−2)2 ^+ 2^2 + (^−1)2 (−2)2 + 22 + (−1)2 (−2)2 + 22 + (−1)2
(ii) Find the angle between the line r = (i + 2 j − k) + l(i − j + k)
 ^ ^ ^ ^ ^ ^  ^ ^ ^
r = (i + 2 j − k) + l(i − j + k) and the plane r ⋅ (2 i − j + k ) = 4. −2 2 −1
i.e., , , .
13. Find the shortest distance and the vector equation 3 3 3
of the line of shortest distance between the lines 6. We have,
   
given by r = 3 ^i + 8 ^j + 3 k^ + l(3 ^i − ^j + k^ ) and r ×b = c ×b
 ^ ^ ^ ^ ^ ^     
r = −3 i − 7 j + 6 k + m(−3 i + 2 j + 4 k) ⇒ r × b − c × b = 0
  
⇒ (r − c ) × b = 0 
14. (i) The vertices of a triangle are A(5, 4, 6), B(1, –1, 3)  
⇒ r − c is parallel  to b
and C(4, 3, 2). The internal bisector of ∠BAC  
meets BC in D. Find AD. ⇒ r − c = lb for some scalar l.
 
(ii) Find a vector of magnitude 15, which is perpendi- ⇒ r = c + lb
 
^ ^ ^ ^ ^
cular to both the vectors 4 i − j + 8 k and − j + k . Now, r ⋅ a = 0
 
⇒ (c + lb ) ⋅ a= 0
15. Find the image of the point having position vector   
^ ^ ^  ^ ^ ^ ⇒ (c ⋅ a ) + l(b ⋅ a ) = 0
i + 3 j + 4 k in the plane r ⋅ (2 i − j + k ) + 3 = 0.  
⇒ l = −  c ⋅ a 
16. Show that the line joining the points (6, –4, 4)  b ⋅a 
and (0, 0, –4) intersects the line joining the points
(–1, –2, –3) and (1, 2, –5).  8+7  15
⇒ l = − = − = −5
 2 + 0 + 1 3
soLUtioNs 
... r = c − 5b = (4 ^i − 3 ^j + 7 k^ )
1. We know that, ^ ^ ^ ^ ^ ^
^ ^ ^ ^ –5(i + j + k) = − i − 8 j + 2 k
a1 i + b1 j = a2 i + b2 j
7. Let l, m, n be the direction cosines of the given vector
⇔ a1 = a2 and b1 = b2 
r (say). Then, its projections on the coordinate axes
.. . 2 ^i + 3 ^j = x ^i + y ^j ⇒ x = 2, y = 3   
are l | r |, m | r |, n | r |.

2. Since AB = (−5 ^i + 7 ^j) − (2 ^i + 1^j) \ l | r | = 6, m | r | = −3, n | r | = 2 ...(i)
   
⇒ AB = −7 ^i + 6 ^j ⇒ {l | r |}2 + {m | r |}2 + {n | r |}2 = 62 + (−3)2 + (2)2
 
So, scalar components of AB along OX and OY ⇒ | r |2 (l 2 + m2 + n2 ) = 36 + 9 + 4

are
 –7 and^6 respectively. The vector components of ⇒ | r |2 = 49 [... l2 + m2 + n2=1]
^
AB are −7 i and 6 j along OX and OY respectively. 
⇒ |r | = 7
    
3. If vectors a and b are perpendicular, then a ⋅ b = 0 Putting | r | = 7 in (i), we get
^ ^ ^ ^ ^ ^
⇒ (3 i + 2 j + 9 k) ⋅ (i + p j + 3 k) = 0 6 3 2
l = ,m= − ,n=
⇒ 3 + 2p + 27 = 0 ⇒ p = –15 7 7 7

mathematics today | december ‘15 69


8. We know that the perpendicular distance of a point 11. The given equation may be written as
    ^ ^ ^
with position vector a from the plane r ⋅ n = d is r ⋅ (6 i − 3 j − 2k ) = −3
given by
  ⇒ r ⋅ (−6 ^i + 3 ^j + 2 k^ ) = 3
| a ⋅n − d |    ^ ^ ^
p=  ⇒ r ⋅ n = 3, where n = (− 6 i + 3 j + 2 k)
|n | 
 ^   n 3 
^ ^ ^ ^ ^
Here, a = 2 i − j − 4 k, n = 3 i − 4 j + 12 k and d = 9. ⇒ r ⋅  =  , where | n | = (− 6)2 + 32 + 22 = 7
|n | |n |
... the required distance is given by ^ ^ ^
^ ^ ^ ^ ^ ^ ⇒ r ⋅ (−6 i + 3 j + 2 k ) = 3
|(2 i − j − 4 k) ⋅ (3 i − 4 j + 12 k) − 9 | 7 7
p=
| 32 + (−4)2 + (12)2 |  
⇒ r ⋅ − 6 ^i + 3 ^j + 2 k^ = 3
 7 7 7  7
|(6 + 4 − 48) − 9 | 47
= = units.
| 169 | 13 Hence, the direction cosines of the normal to the
   6 3 2
9. The
 vector AB × AC is perpendicular to the vectors plane are  − , ,  .
 7 7 7
AB and AC .
  12. (i) Let A, B, C be the points with position vectors
.. . Required vector = AB × AC ^ ^ ^ ^ ^ ^ ^ ^ ^
  i + j − 2 k, 2 i − j + k and i + 2 j + k respectively.
| AB × AC | 
 Then, AB = P.V. of B – P.V. of A
Now, AB = P.V. of B – P.V. of A ^ ^ ^ ^ ^ ^ ^ ^ ^
^ ^ ^ ^ ^ ^ ^ ^ ^
= (2 i − j + k ) − (i + j − 2 k ) = i − 2 j + 3 k
= (i − j − 3 k ) − (3 i − j + 2 k ) = −2 i + 0 j − 5 k 
 and BC = P.V. of C – P.V. of B
and AC = P.V. of C – P.V. of A ^ ^ ^ ^ ^ ^ ^ ^
= (i + 2 j + k) − (2 i − j + k) = –i + 3 j + 0 k
^
^ ^ ^ ^ ^ ^ ^ ^ ^
=(4 i − 3 j + k) − (3 i − j + 2 k) = i − 2 j − k
A vector normal to the plane containing points
i j k
^ ^ ^ A, B, and C is
 
\ AB × AC = −2 0 −5 i j k
^ ^ ^

1 −2 −1    = 1 −2 3 = ^ ^ ^
n = AB × BC −9 i − 3 j + k
^ ^ ^ ^ ^ ^ −1 3 0
= (0 – 10) i − (2 + 5) j + (4 − 0) k = −10 i − 7 j + 4 k
 
⇒ | AB × AC | = (−10)2 + (−7)2 + 42 = 165 The required plane passes through the point
 ^ ^ ^
Hence, required vector = having position vector a = i + j − 2 k and is
  ^ ^ ^
AB × AC 1 ^ ^ ^ normal to the vector −9 i − 3 j + k . So, its vector
  = (−10 i − 7 j + 4 k)
| AB × AC | 165 equation is
10. Any plane through the intersection of the two given   
(r − a ) ⋅ n = 0
planes is    
 ^ ^ ^  ^ ^ ^ ⇒ r ⋅n = a ⋅n
[r ⋅ (i + 3 j − k ) − 5] + l[r ⋅ (2 i − j + k ) − 3] = 0
^ ^ ^ ^ ⇒ r ⋅ (−9 ^i − 3 ^j + k^ ) = (^i + ^j − 2 k^ ) ⋅ (−9 ^i − 3 ^j + k^ )
⇒ r⋅[(1 + 2l) i + (3 − l) j + (l − 1) k] − (5 + 3l) = 0
... (1) 
⇒ r ⋅ (−9 ^i − 3 ^j + k^ ) = −9 − 3 − 2
 ^ ^ ^
If (1) passes through (2, 1, –2), then r = (2 i + j − 2 k)
⇒ r ⋅ (9 i + 3 j − k) = 14
^ ^ ^
should satisfy it.
^ ^ ^ ^ ^ ^
... (2 i + j − 2 k) ⋅[(1 + 2l) i + (3 − l) j + (l − 1) k] − (5 + 3l) = 0 (ii) We know that the angle q between the line
   
– (5 +3l) = 0 r = a + lb and the plane r ⋅ n = d is given by
 
⇔ 2(1 + 2l) + (3 – l) –2(l – 1) – (5 + 3 l) = 0 b ⋅n
⇔ 2l = 2 ⇔ l = 1. sin q =  
| b || n |
Putting l = 1 in (1), we get the required equation of  ^ ^ ^
 ^ ^  ^ ^ ^
the plane as r ⋅ (3 i + 2 j) = 8. Here, b = i − j + k and n = 2 i − j + k

70 mathematics today | december ‘15


^ ^ ^ ^ ^ ^
(i − j + k) ⋅ (2 i − j + k )
... sin q =
12 + (−12 ) + 12 22 + (−12 ) + 12
2 +1+1 4 2 2
= = =
3 6 3 2 3
2 2
⇒ q = sin–1  
 3  ... D divides BC internally in the ratio 5 : 3
13. Given lines are  5 × 4 + 3 × 1 5 × 3 + 3(−1) 5 × 2 + 3 × 3 
 ^ ^ ^ ^ ^ ^ ... D≡  , ,
r = 3 i + 8 j + 3 k + l(3 i − j + k) ...(1)  5+3 5+3 5 + 3 
 ^ ^ ^ ^ ^ ^
and r = −3 i − 7 j + 6 k + m(−3 i + 2 j + 4 k ) ...(2)
or D ≡  23 , 12 , 19 
Equations of lines (1) and (2) in cartesian form are  8 8 8
x −3 y −8 z −3 ...(3) 2 2 2
AB : = = =l  23   12   19 
3 −1 1 ... AD =  5 − 8  +  4 − 8  +  6 − 8 
x +3 y +7 z −6
and CD : = = =m ...(4)
−3 2 4
172 + 202 + 292 1530
Let L ≡ (3l + 3, –l +8, l + 3) = 2
= units.
8 8
and M ≡ (–3m – 3, 2m – 7, 4m + 6)  ^ ^ ^  ^ ^
(ii) Let a = 4 i − j + 8 k and b = − j + k .
 
A unit vector perpendicular to both a and b
 
a ×b
=   ...(1)
|a × b |
^ ^ ^
i j k
Direction ratios of LM are 
Now, a × b = 4 −1 8
3l + 3m + 6, – l – 2m + 15, l – 4m – 3
Since LM ⊥ AB 0 −1 1
... 3(3l + 3m + 6) –1(–l –2m + 15) + 1(l –4m – 3) = 0 ^ ^ ^
= (−1 + 8) i − (4 − 0) j + (−4 − 0) k
or 11l + 7m = 0 ...(5) ^ ^ ^
= 7 i − 4 j− 4 k
Again LM ⊥ CD  
\ | a × b | = 72 + (−4)2 + (−4)2 = 81 = 9
... –3(3l + 3m + 6) +2(–l –2m + 15)  
+ 4(l –4m – 3) = 0 From (1), a unit vector perpendicular to both a and b
  ^ ^ ^
or –7l – 29m = 0 ...(6) (a × b ) 7 i − 4 j − 4 k
=   =
Solving (5) and (6), we get l = 0, m = 0 |a × b | 9 ^ ^ ^
15(7 i − 4 j − 4 k )
\ L ≡ (3, 8, 3), M ≡ (–3, –7, 6) Hence, the required vector =
9
Hence, shortest distance LM 5 ^ ^ ^
= (7 i − 4 j − 4 k ).
= (3 + 3)2 + (8 + 7)2 + (3 − 6)2 3 ^ ^ ^
= 270 = 3 30 units 15. Let Q be the image of the point P (i + 3 j + 4 k) in the
 ^ ^ ^
Vector equation of LM is plane r ⋅ (2 i − j + k ) + 3 = 0. Then, PQ is normal to
 ^ ^ ^ ^ ^ ^ the plane. Since PQ passes through P and is normal
r = 3 i + 8 j + 3 k + t (6 i + 15 j − 3 k ) to the given plane, therefore equation of line PQ is
14. (i) AB =  ^ ^ ^ ^ ^ ^
42 + 52 + 32 = 5 2 r = (i + 3 j + 4 k) + l(2 i − j + k )
AC = 12 + 12 + 42 = 3 2 Since Q lies on line PQ, so let the position
vector of Q be
Since AD is the internal bisector of ∠BAC ^ ^ ^ ^ ^ ^
BD AB 5 (i + 3 j + 4 k) + l(2 i − j + k )
... = = ^ ^ ^
= (1 + 2l) i + (3 − l) j + (4 + l) k
DC AC 3
mathematics today | december ‘15 71
equation of plane through any three of them, say P,
Q, R. Equation of any plane through P(6, –4, 4) is
A(x – 6) + B(y + 4) + C(z – 4) = 0 ... (1)
Since it passes throught Q(0, 0, –4) and R(–1, –2, –3)
... –6A + 4B – 8C = 0 ...(2)
and –7A + 2B – 7C = 0 ...(3)
Solving (2) and (3) by cross-multiplication, we get
A B C A B C
= = or = =
−28 + 16 56 − 42 −12 + 28 −12 14 16
Since R is the mid-point of PQ. Therefore, position
vector of R is A B C
^ ^ ^ ^ ^ ^
or = = = k ⇒ A = 6k, B = –7k and C = –8k
[(1 + 2l) i + (3 − l) j + (4 + l) k] + [i + 3 j + 4 k] 6 −7 −8
Substituting the values of A, B and C in (1), we get
2
^  l ^  l ^ 6(x – 6) –7(y + 4) –8(z – 4) = 0
= (l + 1) i +  3 −  j +  4 +  k ⇒ 6x – 7y –8z – 32 = 0 ...(4)
 2  2
which is the equation of a plane through P, Q and R.
 ^ ^ ^
Since R lies on the plane r ⋅ (2 i − j + k ) + 3 = 0 The four points will be coplanar if the fourth point
... (l + 1) ^i +  3 − l  ^j +  4 + l  k^  . S(1, 2, –5) lies on (4) i.e. if 6(1) –7(2) –8(–5) – 32 = 0
 2  
 2   or if 0 = 0 which is true.
^ ^ ^ So, the four points are coplanar.
(2 i − j + k ) + 3 = 0
l l Now, two coplanar lines are either parallel to
⇒ 2l + 2 − 3 + +4+ +3=0 intersecting.
2 2
⇒ l = –2. d.r.'s of PQ are (0 – 6, 0 + 4, –4 –4)
Thus, the position vector of Q is i.e. (–6, 4, –8) or (–3, 2, –4)
^ ^ ^ ^ ^ ^ ^ ^ ^ d.r.'s of RS are (1 + 1, 2 + 2, –5 + 3)
(i + 3 j + 4 k) − 2(2 i − j + k ) = −3 i + 5 j + 2 k . i.e. (2, 4, –2) or (1, 2, –1)
16. The given points are P(6, –4, 4), Q(0, 0,–4), Since the d.r.'s of PQ and RS are not proportional,
R(–1, –2, –3) and S(1, 2, –5). Let us first find the So, PQ and RS are not parallel. Hence, PQ intersects
RS.
nn

72 mathematics today | december ‘15


UNIT-VI : STATISTICS The entire syllabus of Mathematics of WB-JEE is being divided into six units, on
each unit there will be a Mock Test Paper (MTP) which will be published in the
AND PROBABILITY subsequent issues.

cateGoRy i
2n + 1 n(2n + 1)
For each correct answer one mark will be awarded, (c) (d)
3 6
whereas, for each wrong answer, 25% of total marks
(1/4) will be deducted. If candidates mark more than 6. The mean deviation about median of the numbers
one answer, negative marking will be done. a, 2a, ..., 50a is 50, then |a| =
(a) 4 (b) 5 (c) 2 (d) 3
1. The algebraic sum of deviations of 16 observations
measured from 25 is 4. The mean of the set is 7. All the students of a class performed poorly in
(a) 4 (b) 404 (c) 25 (d) 25.25 mathematics. The teacher decided to give grace
marks of 10 to each student. Which of the following
2. The marks obtained by 300 candidates in statistics
statistical measures will not change even after the
are 46. The mean of the top 100 of them was found
grace marks given?
to be 70 and the mean of the last 100 was known
(a) mean (b) median
to be 20. What is the mean of the remaining 100
(c) mode (d) variance
students?
(a) 48 (b) 46 8. The variance of first 50 even natural numbers is
(c) 70 (d) none of these. 833 437
(a) (b) 833 (c) 437 (d)
3. The sum of the algebraic deviations of a set of values 4 4
measured from 50 is –10. While from 40 is 70. The 9. The standard deviation of 50 values of a variable
number of values is x is 15. If each value of the variable is divided by
(a) 8 (b) 7 (c) 6 (d) 5 –3, then the standard deviation of the new set of
4. The mean weekly salary paid to all employees of a 50 values of x will be
company was Rs. 5000. The mean weekly salaries (a) 15 (b) –5 (c) 5 (d) –15
paid to male and female employees were Rs. 5200 10. The mean deviation about mean of the set of
and Rs. 4200 respectively. The percentage of male numbers 7, 9, 24, 14 and 26 is
and female respectively employees in the company (a) 7.5 (b) 8 (c) 7.2 (d) 7
is 11. If the sum of the squares of deviations of 25
(a) 60, 40 (b) 80, 20 observations taken from the mean 40 is 900, then
(c) 20, 80 (d) 70, 30 the co-efficient of variation is
5. The weighted arithmetic mean of first n (a) 20% (b) 12.5% (c) 15% (d) 18%
natural numbers when weights are equal to the 2
12. If in a distribution, n = 10, ∑ x = 20, ∑ x = 200,
corresponding numbers is equal to then the value of standard deviation of the
2n + 1 n(n + 1)(2n + 1) distribution is
(a) (b)
2 6 (a) 2 (b) 16 (c) 6 (d) 4
By : Sankar Ghosh, HOD(Math), Takshyashila. Mob : 09831244397

74 mathematics today | december ‘15


13. Two variables x and y are related by y = 8 + 2x; if the 22. The probability that at least one of A and B occurs is
s.d. of x is 3, then the s.d. of y will be 0.6. If A and B occur simultaneously with probability
(a) 10 (b) 14 (c) 11 (d) 6 0.3, then P(A′) + P(B′) is
14. If the mean and co-efficient of mean deviation about (a) 0.9 (b) 0.15 (c) 1.1 (d) 1.2.
mean of a distribution be 40 and 30% respectively, 23. An urn contains 8 red and 5 white balls. Three balls
then the mean deviation about mean will be are drawn at random. Then the probability that
(a) 12 (b) 15 (c) 20 (d) 24
balls of both colours are drawn is
15. The median of a distribution is 60 and mean
40 70 3 10
deviation about median is 24, then the co-efficient (a) (b) (c) (d)
of mean deviation of the distribution is 143 143 13 13
(a) 30% (b) 36% (c) 32% (d) 40% 3
24. For two events A and B, if P ( A ∪ B) = ,
4
16. 4 boys and 2 girls occupy seats in a row at random. 1 2 c
Then probability that the two girls occupy side by P ( A ∩ B) = and P ( A) = then P(A ∩ B) =
4 3
side is
(a) 1/2 (b) 1/4 (c) 1/3 (d) 1/6 5 3 1 3
(a) (b) (c) (d)
12 8 12 12
17. Each of a and b can take values 1 or 2 with equal
probability. The probability that the equation 5
25. If P ( A ∩ B) = , then P(Ac ∪ Bc) =
ax2 + bx + 1 = 0 has real roots, is equal to 13
(a) 1/2 (b) 1/4 (c) 1/8 (d) 1/16 4 6 7 8
(a) (b) (c) (d)
18. A fair six faced die is rolled 12 times. The probability 13 13 13 13
that each face turns up twice is equal to
26. Three numbers are chosen at random without
12 ! 212 replacement from {1, 2, ..., 8}. The probability that
(a) (b)
6 ! 6 ! 612 26612 their minimum is 3, given that their maximum is 6,
12 ! 12 ! is
(c) (d) (a) 1/4 (b) 2/5 (c) 3/8 (d) 1/5
26612 62612
19. An urn contains nine balls of which three are 1
27. Let A and B be two events such that P ( A ∪ B)c = ,
red, four are blue and two are green. Three balls 6
1 1
are drawn at random without replacement from P ( A ∩ B) = and P ( Ac ) = , then the events A
the urn. The probability that the three balls have 4 4
and B are
different colour is
(a) mutually exclusive and independent
1 2 1 2
(a) (b) (c) (d) (b) equally likely but not independent
3 7 21 23
(c) independent but not equally likely
20. Two decks of playing cards are well shuffled and 26 (d) independent and equally likely.
cards are randomly distributed to a player. Then the
28. If C and D are two events such that C ⊆ D and D ≠ 0,
probability that the player gets all distinct cards is
52 then the correct statement among the following is
C26 2 × 52C26
(a) (b) P (D )
104 104 (a) P(C|D) < P(C) (b) P (C | D) =
C26 C26
P (C )
213 × 52C26 226 × 52C26 (c) P(C|D) = P(C) (d) P(C|D) ≥ P(C)
(c) (d)
104 104
C26 C26 29. A problem in mathematics is given to three students
21. Two dice are tossed once. The probability of getting A, B and C and their respective probability of solving
an even number at the first die or a total of 8 is the problem is 1/2, 1/3, and 1/4. Then probability
1 3 11 5 that the problem is solved is
(a) (b) (c) (d)
36 36 36 9 (a) 3/4 (b) 1/2 (c) 2/3 (d) 7/8

mathematics today | december ‘15 75


30. The probability that a regularly scheduled flight cateGoRy iii
departs on time is 0.9. The probability that it arrives In this section more than 1 answer can be correct.
on time is 0.8 and probability that it departs and Candidates will have to mark all the correct answers,
arrive on time is 0.7. Then the probability that a
for which 2 marks will be awarded. If, candidates
plane arrives on time, given that it departs on time is
mark one correct and one incorrect answer, then
(a) 0.72 (b) 8/9 (c) 7/9 (d) 0.56
no marks will be awarded. But if, candidate makes
cateGoRy ii only correct, without marking any incorrect, formula
Every correct answer will yield 2 marks. For incorrect below will be used to allot marks.
response, 25% of full mark (1/2) would be deducted. 2 × (no. of correct response/total no. of correct
If candidates mark more than one answer, negative options)
marking will be done.
36. For two events A and B, let P(A) = 0.7 and
31. In a bolt factory three machines A, B and C P(B) = 0.6. The necessarily false statement(s) is/are
manufacture respectively 2000, 2500 and 4000 bolts (a) P(A ∩ B) = 0.35 (b) P(A ∩ B) = 0.45
everyday. Of their outputs 3%, 4% and 2.5% are (c) P(A ∩ B) = 0.65 (d) P(A ∩ B) = 0.28
defective bolts. One bolt is drawn at random from a
3 5
day’s production and it is found to be defective. Then 37. If A and B are two events and P ( A) = , P (B) = ,
the probability that it was produced by machine C 4 8
then
is 3 1
3 (a) P ( A ∪ B) ≥ (b) P ( Ac ∩ B) ≤
4 5 7 4 4
(a) (b) (c) (d)
13 13 13 13 3 5
(c) ≤ P ( A ∩ B) ≤ (d) none of these.
32. A student answers a multiple choice question with 8 8
5 alternatives, of which exactly one is correct. The 38. For two events A and B, the correct statement
probability that he knows the correct answer is p, is/are
0 < p < 1. If he does not know the correct answer, (a) P ( A ∩ B) ≥ P ( A) + P (B) −1
he randomly ticks one answer. Given that he has (b) P ( A ∩ B) ≤ P ( A)
answered the question correctly, the probability (c) P ( Ac ∩ Bc ) ≥ P ( Ac ) + P (Bc ) −1
that he did not tick the answer randomly, is
(d) P(A ∩ B) = P(A) P(B)
3p 5p
(a) (b) 39. A and B are two independent events such that
4p + 3 3p + 2
1 1
5p 4p P ( A ∩ B) = and P(either A or B) = then
(c) (d) 12 2
4p +1 3p +1
1
33. There are two coins, one unbiased with probability (a) P ( A ∪ B) =
2
1/2 of getting heads and the other one is biased with 1
c c
probability 3/4 of getting heads. A coin is selected (b) P ( A ∩ B ) =
7
at random and tossed. If it shows heads up, then the
1 1
probability that the unbiased coin was selected is (c) P ( A) = , P (B) =
3 4
(a) 2/3 (b) 3/5 (c) 1/2 (d) 2/5
1 1
34. Let A and B be two events with P(Ac) = 0.3, (d) P ( A) = , P (B) =
P(B) = 0.4 and P(A ∩ Bc) = 0.5 then P(B | (A ∪ Bc)) = 4 3
(a) 1/4 (b) 1/3 (c) 1/2 (d) 2/3 40. If A and B are two events then probability that
exactly one of them occurs is
2 1 1
35. If P ( A) = , P (B) = and P ( A ∩ B) = then find (a) P(A ∩ Bc) + P(Ac ∩ B)
3 2 6
(b) P(A ∪ B) – P(A ∩ B)
the value of P(A ∩ B′) + P(A ∪ B) is
(c) P(A) + P(B) – 2P(A ∩ B)
(a) 3/2 (b) 1/2
(d) P(Ac) + P(Bc) – 2P(Ac ∩ B)
(c) 1 (d) none of these

76 mathematics today | december ‘15


soLUtioNs n n(n + 1) n n(n + 1)(2n + 1)
∑ fi = , ∑ xi f i =
16 i =1 2 i =1 6
1. (d) : Given that ∑ (xi − 25) = 4
i =1 \ Required mean
16 1 16 1 n(n + 1)(2n + 1) 2 2n + 1
⇒ ∑ xi − 400 = 4 ⇒ ∑ xi = (404) = × =
i =1 16 i =1 16 6 n(n + 1) 3
\ x = 25.25 6. (a) : The median of the numbers a, 2a, ..., 50a is
the mean of 25th and 26th observations.
2. (a) : Total marks obtained = 300 × 46 = 13800
1 1
Total marks of top 100 = 100 × 70 = 7000 \ Median = (25a + 26a) = ⋅ 51a = 25.5a
Total marks of last 100 = 100 × 20 = 2000 2 2
\ Total marks of remaining 100 candidates \ Mean deviation about median
= 13800 – 7000 – 2000 = 4800 1 50
= ∑ | x − 25.5a |
4800 50 i =1 i
\ Required mean = = 48
100 1
= (| a − 25.5a | + | 2a − 25.5a | +...+ | 50a − 25.5a |)
3. (a) : Let the number of values be n. 50
n n 2 2
\ ∑ (xi − 50) = −10 and ∑ (xi − 40) = 70 = (24.5a + 23.5a + ... + 0.5a) = (312.5a)
i =1 i =1 50 50
⇒ n(x − 50) = −10 …(i) 2
Given, (312.5a) = 50 ⇒ 625a = 2500
and n(x − 40) = 70 …(ii) 50
x − 50 −10 1 100
Dividing (i) and (ii), = =− ⇒ a= =4
x − 40 70 7 25
⇒ 7 x − 350 = − x + 40
7. (d) : We know that variance is independent of
\ 8 x = 390 ⇒ x = 48.75 change of origin. Therefore, after giving grace marks to
Also, from (ii), putting x = 48.75 all the students of a class, the variance of their marks
70 will not change.
n= =8
8.75 8. (b) : The numbers are 2, 4, 6, 8, ...,100
4. (b) : Let n1 and n2 be the number of males and 1 50 2 2
Variance = ∑ x −x
females. Then total weekly salary = (n1 + n2) × 5000 n i =1 i
\ 5000(n1 + n2) = 5200n1 + 4200n2
Now ∑ xi = 2 + 4 + 6 + ... + 100
⇒ 800n2 = 200n1 i
⇒ n1 : n2 = 800 : 200 = 4 : 1 50 50 × 102
4 =
(2 + 100) =
\ Percentage of males = × 100 = 80 2 2
5
1 1 50 × 102 102
1 \ x = ∑ xi = × = = 51
and percentage of females = × 100 = 20 50 i 50 2 2
5
2 2 2 2 2
5. (c): The frequency distribution is Now ∑ xi = 2 + 4 + 6 + .... + 100
i
xi fi xi f i
= 22(12 + 22 + 32 + .. + 502)
1 1 12 = 22 × {50(50 + 1)(2 × 50 + 1)}/6
2 2 22
3 3 32 = 22 × {50 × 51 × 101}/6
. . . 1 1 22 × 50 × 51 × 101 4 × 51 × 101
2
. . . \ ∑ xi = × =
i i i2 n i 50 6 6
. . .
. . . 2
= × 51 × 101
n n n2 3

mathematics today | december ‘15 77


2 14. (a) : Given that, x = 40
1 1 
\ Variance = ∑ xi2 −  ∑ xi  and co-efficient of mean deviation about mean is
n  n 
30%.
2 1 n
= × 51 × 101 − (51)2 = 3434 − 2601 = 833 ∑ |x − x |
3 n i =1 i
\ × 100 = 30
9. (c): Given that sx = 15, n = 50 x
1 1 n 30 × 40
Let yi = − xi , i = 1, 2, ..., 50 ⇒ ∑ | xi − x | = = 12
3 n i =1 100
1 1
⇒ var( y ) = var(x ) = × 152 [ s x = 15] Thus, the mean deviation about mean is 12.
9 9
15. (d) : Given that,
var(y) = 25 ⇒ sy = 5 1 n
\ Standard deviation of the new set of 50 values of median = me = 60 and ∑ | x − me | = 24
n i =1 i
x will be 5.
\ The co-efficient of mean deviation is
7 + 9 + 24 + 14 + 26 80
10. (c): Here x = = = 16 24
5 5 × 100 % = 40 %
\ Mean deviation about mean 60
1 5 9 + 7 + 8 + 2 + 10 36 16. (c): Let A be the event that the girls occupy seats
= ∑ | xi − 16 | = = = 7. 2
5 i =1 5 5 side by side.
Now 4 boys and 2 girls can be arranged in a row at
25
2 random in 6! ways. Therefore sample space contains
11. (c): Given that, ∑ (xi − 40) = 900
i =1 6! equally likely event points. Considering 2 girls as
1 25 2 1 one member, we shall have 5 members in all. These
⇒ ∑ (xi − 40) = × 900 5 members can be arranged in a row in 5! ways and
25 i =1 25
for each such arrangement, 2 girls can arrange among
⇒ sx2 = 36 \ sx = 6 themselves in 2! ways. Therefore, the number of
Now, co-efficient of variation arrangements of 4 boys and 2 girls in which the girls
s 6 occupy seats side by side is 5! × 2!.
= x × 100 = × 100 = 15% Therefore, by classical definition of probability, we get
x 40
12. (d) : Given that n = 10, Sx = 20, Sx2 = 200 5! × 2 ! 5! × 2 ! 1
P ( A) = = =
6! 6 × 5! 3
2 2
1 2 1  1  20  17. (b) : The equation ax2 + bx + 1 = 0 has real roots.
\ sx = ∑x −  ∑x = × 200 −  
n  n  10  10  \ b2 – 4a · 1 ≥ 0 ⇒ b2 ≥ 4a
Now each a and b can take values 1 or 2 with equal
= 20 − 4 = 16 = 4
probability.
13. (d) : The relation between two variables x and y is a b b2 4a
y = 8 + 2x
1 1 1 4
Let yi = 8 + 2xi, (i = 1, 2, .... n)
\ y = 8 + 2x 1 2 4 4
⇒ yi − y = 2(xi − x ) 2 1 1 8
2 2 4 8
⇒ ( yi − y )2 = 4(xi − x )2
The above table shows the possible combination of
1 n 2 1 n 2
⇒ ∑ ( y i − y ) = 4 ∑ ( xi − x ) values of a and b. Among the 4 cases, only 1 case,
n i =1 n i =1 i.e. 2 nd combination will be favourable for the
⇒ sy2 = 4sx2 = 4 × 9 = 36 [Q sx = 3] required event.
\ sy = 6 Hence, the probability is 1/4.

78 mathematics today | december ‘15


18. (c): Let A be the event that each face turns up twice. Again, number of favourable cases for the event B is
Since a six faced die is rolled 12 times therefore, sample 5 i.e., {(2, 6), (3, 5), (4, 4), (5, 3), (6, 2)}
space contains 612 equally likely event points. 5
So, the no. of favourable cases is \ P ( B) =
12C × 10C × 8C × .... × 2C 36
2 2 2 2
12 Now, favourable cases for the event A ∩ B are 3
C2 × 10C2 × 8C2 × ... × 2C2
\ P ( A) = i.e., {(2, 6), (4, 4), (6, 2)}
612 \ P(A ∪ B) = P(A) + P(B) – P(A ∩ B)
12 ! 10 ! 8! 6! 4! 2 18 5 3 20 5
× × × × × = + − = =
= 2 × 10 ! 2 × 8 ! 2 × 6 ! 2 × 4 ! 2 ×2 2 36 36 36 36 9
612 22. (c): By the problem, we have
12 !
⇒ P ( A) = P(A ∪ B) = 0.6 and P(A ∩ B) = 0.3
2 × 612
6
Since P(A ∪ B) = P(A) + P(B) – P(A ∩ B)
19. (b) : Total number of balls in the urn is 3 + 4 + 2 = 9. ⇒ 0.6 + 0.3 = P(A) + P(B)
Since 3 balls can be selected from 9 balls in 9C3 ways ⇒ 1 – P(A′) + 1 – P(B′) = 0.9
therefore, sample space contains 9C3 equally likely event ⇒ P(A′) + P(B′) = 2 – 0.9 = 1.1
points. Let A denote the event that the 3 balls drawn
23. (d) : Let A be the event that among 3 balls drawn
from the urn at random without replacement are of
from the urn 2, are red and 1 is white.
different colours.
Again B be the event that among 3 balls drawn from
\ Favourable cases = 3C1 × 4C1 × 2C1
3
the urn 1, is red and 2 are white.
C1 × 4C1 × 2C1 3 × 4 × 2 × 3! 2 Since, 3 balls can be drawn from the urn containing
\ P ( A) = = =
9 9×8×7 7 8 + 5 = 13 balls, in 13C3 ways. Therefore, there are 13C3
C3
equally likely sample points in sample space.
20. (d) : Let A denotes the event that all the 26 cards
8
are distinct. C2 × 5C1 8
C1 × 5C2
\ P ( A) = and P (B) =
Since, from 2 decks of playing cards, 26 cards can be 13
C3 13
C3
drawn randomly in 104C26 ways, therefore the sample
space will contain 104C26 equally likely sample points. \ Probability that balls of both colours are drawn is
Now, since, there are 52 distinct cards in a deck and P(A ∪ B) = P(A) + P(B)
each distinct card is 2 in number. [Q A, B events are mutually exclusive]
\ 2 decks will also contain 52 distinct cards, two 8
C2 × 5C1 8
C1 × 5C2 10
each. = + =
13 13 13
C3 C3
From these, 26 distinct cards are drawn in 226 × 52C26
ways. 24. (c): Given that, for two events A and B
\ The number of favourable cases = 226 × 52C26 3 1 2
P ( A ∪ B) = , P ( A ∩ B) = and P ( A) =
226 × 52C26 4 4 3
\ P ( A) = 3 1
104 \ = P ( A) + P (B) −
C26
4 4
21. (d) : Let A be the event of getting an even number 3 1 2 4 1
⇒ P ( B) = + − = =
on the first die and B be the event of getting a total of 4 4 3 12 3
1 1 1
8. Total number of sample points in sample space is Now, P(Ac ∩ B) = P(B) – P(A ∩ B) = − =
6 × 6 = 36 3 4 12
5
Now, number of favourable cases for the event A is 25. (d) : Here P ( A ∩ B) =
13
3 × 6 = 18 i.e., {(2, 1), (2, 2), (2, 3), ...., (2, 6), (4, 1),
(4, 2), (4, 3), ...., (4, 6), (6, 1), (6, 2), ...., (6, 6)} \ P(Ac ∪ Bc) = P(A ∩ B)c = 1 – P(A ∩ B)
18 5 8
\ P ( A) = = 1− =
36 13 13

mathematics today | december ‘15 79


26. (d) : Let A denote the event that out of three 1 1 1
chosen numbers 6 is maximum and B is the event that 3 29. (a) : Here, P ( A) = , P (B) = and P (C ) =
2 3 4
is minimum. Since 6 is maximum, therefore we have to
The problem is solved, when any one of them can solve
select the remaining two numbers from 1, 2, 3, 4, 5 the problem.
5
C2 5 \ P(A ∪ B ∪ C) = 1 – P(A ∪ B ∪ C)c
\ P ( A) = =
8
C3 28 = 1 – P(Ac ∩ Bc ∩ Cc)
Again, 3 is minimum and 6 is maximum. Therefore, we = 1 – P(Ac) P(Bc) P(Cc)
have to select the remaining numbers from 4, 5  1   2   3  1
= 1 −       = 1 −
2 1   
 2 3 4     4
\ P ( A ∩ B) = = 3
8
C3 28 ⇒ P(A ∪ B ∪ C) =
4
1
P (B ∩ A) 28 1 Thus, the probability that the problem will be solved
\ P (B | A) = = = is 3/4.
P ( A) 5 5
28 30. (c): Let E1 be the event that the plane departs on
1 1 time and E2 be the event that the plane arrives on time.
27. (c): Given that P ( A ∪ B)c = , P ( A ∩ B) = \ P(E1) = 0.9, P(E2) = 0.8 and P(E1 ∩ E2) = 0.7
6 4
1 P (E2 ∩ E1 ) 0.7 7
and P ( Ac ) = Now, P (E2 | E1 ) = = =
4 P (E1 ) 0. 9 9
1 31. (c): Let us define the events as
Now, P ( A ∪ B)c =
6 E1 : bolt manufactured from machine A
1 E2 : bolt manufactured from machine B
⇒ P ( Ac ∩ B c ) = ... (i) E3 : bolt manufactured from machine C
6
Let A be the event that the drawn bolt is defective.
c 1 1
Also, P ( A ∪ B) = ⇒ 1 − P ( A ∪ B) = 2000 2000 4
6 6 P (E1 ) = = =
2000 + 2500 + 4000 8500 17
5
⇒ P ( A ∪ B) = 2500 2500 5
6 P (E2 ) = = =
2000 + 2500 + 4000 8500 17
5
⇒ P ( A) + P (B) − P ( A ∩ B) = 4000 4000 8
6 P (E3 ) = = =
3 1 5 2000 + 2500 + 4000 8500 17
⇒ + P ( B) − =
4 4 6 3 4 2.5
P ( A | E1 ) = , P ( A | E2 ) = , P ( A | E3 ) =
5 1 5−3 1 100 100 100
⇒ P ( B) = − = =
6 2 6 3 P (E3 ) ⋅ P ( A | E3 )
2 \ P (E3 | A) =
\ P (Bc ) = P (E1 ) ⋅ P ( A | E1 ) + P (E2 ) ⋅ P ( A | E2 )
3 + P (E3 ) ⋅ P ( A | E3 )
1 1 2
From (i), P ( Ac ∩ Bc ) = = ⋅ = P ( Ac ) ⋅ P (Bc ) 8 2⋅5
6 4 3 ×
= 17 100
Thus, the events are independent. Since the probability 4 3 5 4 8 2. 5
of A and B are different, so they are not equally likely × + × + ×
17 100 17 100 17 100
events.
8 × 2. 5 20 5
P (C ∩ D) P (C ) = = =
28. (d) : P (C | D) = = [ C ⊆ D] 12 + 20 + 20 52 13
P (D ) P (D )
32. (c): Let E denote the event that the student knows
⇒ P (C | D) ≥ P (C )
the answer.
 P (C )  Therefore Ec denote the event that he does not know
 0 < P (D) ≤ 1 \ P (D) ≥ P (C ) the answer.
 

80 mathematics today | december ‘15


38. (a, b, c) : We know that P(A ∪ B) ≤ 1 2
1
⇒ P(A) + P(B) – P(A ∩ B) ≤ 1 \ {P ( A) − P (B)}2 =  
\ P(A ∩ B) ≥ P(A) + P(B) – 1  12 
Again, A ∩ B ⊆ A \ P(A ∩ B) ≤ P(A) 1 ... (ii)
\ P ( A) − P (B) =
Also, P(Ac ∩ Bc) ≤ 1 12
⇒ P(A ∪ B)c ≤ 1 ⇒ 1 – P(A ∪ B) ≤ 1 (i) + (ii) gives
⇒ 1 – {P(A) + P(B) – P(A ∩ B)} ≤ 1 8 2 1
2P ( A) = = \ P ( A) =
⇒ 1 – P(A) – P(B) + P(A ∩ B) ≤ 1 12 3 3
⇒ {1 – P(A)} + {1 – P(B)} – 1 ≤ 1 – P(A ∩ B) 7 1 1
From (i), P (B) = − =
⇒ P(Ac) + P(Bc) – 1 ≤ P(A ∩ B)c 12 3 4
⇒ P(Ac) + P(Bc) – 1 ≤ 1 [Q P(A ∩ B)c ≤ 1] 1 1
\ P ( A) = and P (B) =
Also, P(Ac) + P(Bc) – 1 ≤ P(Ac ∩ Bc) 3 4
2 3 1
i.e. P(Ac ∩ Bc) ≥ P(Ac) + P(Bc) – 1 Now, P(Ac ∩ Bc) = P(Ac) · P(Bc) = ⋅ =
Now, P(A ∩ B) = P(A) · P(B) is only possible when A 3 4 2
and B are independent. 40. (a, b, c) : Clearly A ∩ Bc and B ∩ Ac are mutually
39. (a, c) : Given that A and B are two independent exclusive.
events. A B S
Since, P(A ∪ B) = P(A) + P(B) – P(A ∩ B)
1 1
⇒ = P ( A) + P (B) −
2 12
7
⇒ P ( A) + P (B) = ... (i) A  Bc B  Ac
12
1 1
But P ( A ∩ B) = ⇒ P ( A) ⋅ P (B) = \ P{(A ∩ Bc) ∪ (B ∩ Ac)}
12 12 = P(A ∩ Bc) + P(B ∩ Ac)
Now, {P(A) – P(B)}2 = {P(A) + P(B)}2 – 4P(A)P(B) From the diagram, it is clear that
2 P{(A ∩ Bc) ∪ (B ∩ Ac)} = P(A ∪ B) – P(A ∩ B)
7 1 49 4 49 − 48
=   −4× = − = = P(A) + P(B) – 2P(A ∩ B)
 12  12 122 12 122
nn

82 mathematics today | december ‘15


π 3± 5 5 +1 5 −1
θ= → r2 = →r = ,
6 2 2 2

Solution Set-155 8. (a, d) : θ = π → r 2 = 5 ± 21 , r = 7 ± 3


a +b 3 2 2
1. (b) : = ab + 2 → b = a + 2 9. (6) :
2 D
2 12
(a, b) = (1, 9), (4, 16), (9, 25), (16, 36), (25, 49),
E /3 x C
(36, 64), (49, 81), (64, 100), (81, 121) 9 pairs.
2. (d) : Required area 8 8
1 2 
= 2  ∫ ( 5 − x ) dx + ∫ ( 5 − x ) dx  /3 /3
A B
0 1 
EC = x
(
1
) ( )
2  1
−2  ∫ 1 − x 2 dx + ∫ x 2 − 1 dx  In triangle CDE, 144 = x 2 + 4 − 2 x ⋅ 2 ⋅
0 1  2
= 2(8) – 2(2) = 12 1
→ x = 1 + 141, AB = x + 2 ⋅ 8 ⋅ = 9 + 141
3. (d) : 1 6 11 .............. 96 2
2 7 12 .............. 97 m = 9, n = 141 , m + n = 150, with digit sum 6.
10. (a) : P → 2, Q → 2, R → 5, S → 4
3 8 13 .............. 98
f (x ) f (x )
4 9 14 .............. 99 (P) lim 1 + 2 = 3 \ lim 2 = 2
x →0 x x →0 x
5 10 15 ............. 100 2 1
a2 – b2 = (a + b) (a – b) (Q) 2 sin x + sin x – 1 = 0 → sin x = –1,
2
  20   20 2   100  35 1 π 5π
sin x = → x = ,
Probability = 5   + 2    ÷ 
 = 2 6 6
  2   1    2  99 3π
  sin x = −1 → x = , not valid.
π 2π 4π 2
4. (d) : A = , B = , C =
7 7 7 (R) Area of ABCD = 8
StanB tanC = 1 + sec A sec B sec C D
1 2 4
= 1+
π 2π 4π A C
cos cos cos (4, 5) 2 4
7 7 7
π B
8 sin 2
= 1+ 7 = 1 – 8 = – 7.
(S)
x 2
+
( y − 3) = 1 ⇒ S = (0, 5), S = (0, 1)
π π 2π 4π 1
8 sin cos cos cos 5 9
7 7 7 7
 P = (3, 3) ⇒ DPSS1 = 6
  
5. (a) : PQ = a(2i + 7 j − 5k ) → nn
ERRATA
2a = – 8 – 3t – 3s, 7a = – 4 + 2t + s, November Issue Math Musing Problem set 155 page no. 8 Q.No. 2
Q2. The area bounded by the curves y = |1 – x2| and y = 5 – |x| is
–5a = 8 + 4t – s  (a) 18 (b) 19 (c) 20 (d) 12
\ s = t = a = – 1, | PQ |2 = 4 + 49 + 25 = 78
 Solution Sender of Maths Musing
| PQ | = 78
Set-154
6. (c, d): The number of functions
1. Jayanthi (Hyderabad)
 8 − 1  7   7  2. Khokon Kumar Nandi (W. B.)
=   =   =   = 35 = 5 × 7.
 5 − 1  4   3 
3. Gouri Sankar Adhikari (W. B.)
4. Satya Dev (Bangalore)
2 Set-155
7. (c, d) : reiθ + 1 e −iθ = 4 →
r 1. Rajdeep Choudhary (W.B.)
2. N. Jayanthi (Hyderabad)
r 2 = 2 − cos 2θ ± 3 − 4 cos 2θ + cos2 2θ 3. Manmohan Krishna (Bihar)

84 mathematicS today | DeCeMBeR ‘15


Y U ASK Ans. Equation of a tangent to the given ellipse can be
written as
y – mx = a2m2 + b2 …(1)

WE ANSWER
Do you have a question that you just can’t get
If P(h, k) be a point lying on the above tangent,
then, we have
k – mh = a2m2 + b2
answered?
i.e., (h2 – a2)m2 – 2hkm + k2 – b2 = 0 …(2)
Use the vast expertise of our mtg team to get to the
Let the slope of two tangents or roots of equation
bottom of the question. From the serious to the silly,
(2) be m1 and m2.
the controversial to the trivial, the team will tackle the
m − m2
questions, easy and tough. Then tan q = 1
1 + m1m2
the best questions and their solutions will be printed in
this column each month. i.e., (1 + m1m2)2tan2q = (m1 – m2)2
= (m1 + m2)2 – 4m1m2
1. If for 0 < x < p/2, exp [(sin2x + sin4x + sin6x + … ∞) 2
 2 2 4h 2 k 2 4(k 2 − b2 )
loge2] satisfies the quadratic equation x2 – 9x + 8 = 0, i.e., 1 + k − b  tan2 q = −
sin x − cos x
 h2 − a 2  (h2 − a2 )2 (h2 − a2 )
find the value of . i.e., (h2 + k2 – a2 – b2)2tan2q = 4(b2h2 + a2k2 – a2b2)
sin x + cos x
– Madhav, Assam Putting (x, y) in place of (h, k) gives the equation
Ans. Q 0 < x < p/2 \ 0 < sin2x
<1 of the required locus as
2 4 6
Then sin x + sin x + sin x + … ∞ (x2 + y2 – a2 – b2)2 tan2q = 4(b2x2 + a2y2 – a2b2).
sin2 x 3. Evaluate ∫ ln( 1 − x + 1 + x ) dx
= = tan2x
2
1 − sin x – Swastika, A.P
\ exp [(sin2x + sin4x + sin6x + … ∞) loge2] Ans. We have
= exp {tan2x loge2} I = ∫ ln( 1 − x + 1 + x ) dx
tan 2x
2 2
= exp {log e 2tan x } = e log e 2 = 2tan x x  1 1 
= x ln( 1 − x + 1 + x ) − ∫  − 
 11− x + 1 +1x 2 1 + x 2 1 − x
2
Let y = 2tan x x ln( 1 − x + 1 + x ) − ∫
x

  dx
Q y satisfies the quadratic equation 1− x + 1+ x  2 1+ x 2 1− x 
y2 – 9y + 8 = 0 \ y = 1, 8 x 1− x − 1+ x
= x ln( 1 − x + 1 + x ) − ∫ ⋅ dx
tan2 x tan2 x
if y = 1 = 2 then 2 = 20 21− x + 1+ x
x 2 1 −1x− x − 1 + x
2
\ tan x = 0 x ln( 1 − x + 1 + x ) − ∫ ⋅ dx
\ x = 0 (impossible) (Q x > 0) 2 1 − x2 1 − x + 1 + x

Now if y = 8 = 2 then 2tan2 x tan2 x = 23 x 2 − 2 1 − x2


= x ln( 1 − x + 1 + x ) − ∫ ⋅ dx
2
⇒ tan x = 3 \ tan x = 3 2 1 − x2 −2 x
2
x 2 − 2 1− x
sin x − cos x tan x − 1 3 −1 3 −1 x ln( 1 − x + 1 + x ) − ∫ ⋅ dx
\ = = × −2 x
sin x + cos x tan x + 1 3 +1 3 −1 2 1 − x2
2
( 3 − 1) 3 +1− 2 3 1 − x2 − 1
= = = 2 − 3. = x ln( 1 − x + 1 + x ) − ∫ dx
3 −1 2 2 1 − x2
2. Find the locus of the point from which the two 1 1 dx
x2 y2 = x ln( 1 − x + 1 + x ) − ∫ 1 dx + ∫
tangents to the ellipse + = 1 are inclined at 2 2 1 − x2
a 2 b2 1 1
an angle q. = x ln( 1 − x + 1 + x ) − x + sin −1 x + C
2 2
– Rakesh Sethi, Punjab nn

mathematics today | DECEmbEr ‘15 85


86 mathematics today | DECEmbEr ‘15

You might also like